*NURSING > TEST BANK > (Answered) Health Assessment All Combined —Jarvis Final Exam_2022. (All)

(Answered) Health Assessment All Combined —Jarvis Final Exam_2022.

Document Content and Description Below

Health Assessment All Combined —Jarvis Final Exam_2022. The nurse is preparing to conduct a health history. Which of these statements best describes the purpose of a health history? a. To p... rovide an opportunity for interaction between the patient and the nurse b. To provide a form for obtaining the patients biographic information c. To document the normal and abnormal findings of a physical assessment d. To provide a database of subjective information about the patients past and current health When the nurse is evaluating the reliability of a patients responses, which of these statements would be correct? The patient: a. Has a history of drug abuse and therefore is not reliable. b. Provided consistent information and therefore is reliable. c. Smiled throughout interview and therefore is assumed reliable. d. Would not answer questions concerning stress and therefore is not reliable. A 59-year-old patient tells the nurse that he has ulcerative colitis. He has been having black stools for the last 24 hours. How would the nurse best document his reason for seeking care? a. J.M. is a 59-year-old man seeking treatment for ulcerative colitis. b. J.M. came into the clinic complaining of having black stools for the past 24 hours. c. J.M. is a 59-year-old man who states that he has ulcerative colitis and wants it checked. d. J.M. is a 59-year-old man who states that he has been having black stools for the past 24 hours. A patient tells the nurse that she has had abdominal pain for the past week. What would be the nurses best response? a. Can you point to where it hurts? b. Well talk more about that later in the interview. c. What have you had to eat in the last 24 hours? d. Have you ever had any surgeries on your abdomen? A 29-year-old woman tells the nurse that she has excruciating pain in her back. Which would be the nurses appropriate response to the womans statement? a. How does your family react to your pain? b. The pain must be terrible. You probably pinched a nerve. c. Ive had back pain myself, and it can be excruciating. d. How would you say the pain affects your ability to do your daily activities? In recording the childhood illnesses of a patient who denies having had any, which note by the nurse would be most accurate? a. Patient denies usual childhood illnesses. b. Patient states he was a very healthy child. c. Patient states his sister had measles, but he didnt. d. Patient denies measles, mumps, rubella, chickenpox, pertussis, and strep throat. A female patient tells the nurse that she has had six pregnancies, with four live births at term and two spontaneous abortions. Her four children are still living. How would the nurse record this information? a. P-6, B-4, (S)Ab-2 b. Grav 6, Term 4, (S)Ab-2, Living 4 c. Patient has had four living babies. d. Patient has been pregnant six times. A patient tells the nurse that he is allergic to penicillin. What would be the nurses best response to this information? a. Are you allergic to any other drugs? b. How often have you received penicillin? c. Ill write your allergy on your chart so you wont receive any penicillin. d. Describe what happens to you when you take penicillin. The nurse is taking a family history. Important diseases or problems about which the patient should be specifically asked include: a. Emphysema. b. Head trauma. c. Mental illness. d. Fractured bones. The review of systems provides the nurse with: a. Physical findings related to each system. b. Information regarding health promotion practices. c. An opportunity to teach the patient medical terms. d. Information necessary for the nurse to diagnose the patients medical problem. Which of these statements represents subjective data the nurse obtained from the patient regarding the patients skin? a. Skin appears dry. b. No lesions are obvious. c. Patient denies any color change. d. Lesion is noted on the lateral aspect of the right arm. The nurse is obtaining a history from a 30-year-old male patient and is concerned about health promotion activities. Which of these questions would be appropriate to use to assess health promotion activities for this patient? a. Do you perform testicular self-examinations? b. Have you ever noticed any pain in your testicles? c. Have you had any problems with passing urine? d. Do you have any history of sexually transmitted diseases? Which of these responses might the nurse expect during a functional assessment of a patient whose leg is in a cast? a. I broke my right leg in a car accident 2 weeks ago. b. The pain is decreasing, but I still need to take acetaminophen. c. I check the color of my toes every evening just like I was taught. d. Im able to transfer myself from the wheelchair to the bed without help. In response to a question about stress, a 39-year-old woman tells the nurse that her husband and mother both died in the past year. Which response by the nurse is most appropriate? a. This has been a difficult year for you. b. I dont know how anyone could handle that much stress in 1 year! c. What did you do to cope with the loss of both your husband and mother? d. That is a lot of stress; now lets go on to the next section of your history. In response to a question regarding the use of alcohol, a patient asks the nurse why the nurse needs to know. What is the reason for needing this information? a. This information is necessary to determine the patients reliability. b. Alcohol can interact with all medications and can make some diseases worse. c. The nurse needs to be able to teach the patient about the dangers of alcohol use. d. This information is not necessary unless a drinking problem is obvious. The mother of a 16-month-old toddler tells the nurse that her daughter has an earache. What would be an appropriate response? a. Maybe she is just teething. b. I will check her ear for an ear infection. c. Are you sure she is really having pain? d. Describe what she is doing to indicate she is having pain. During an assessment of a patients family history, the nurse constructs a genogram. Which statement best describes a genogram? a. List of diseases present in a persons near relatives b. Graphic family tree that uses symbols to depict the gender, relationship, and age of immediate family members c. Drawing that depicts the patients family members up to five generations back d. Description of the health of a persons children and grandchildren A 5-year-old boy is being admitted to the hospital to have his tonsils removed. Which information should the nurse collect before this procedure? a. Childs birth weight b. Age at which he crawled c. Whether the child has had the measles d. Childs reactions to previous hospitalizations As part of the health history of a 6-year-old boy at a clinic for a sports physical examination, the nurse reviews his immunization record and notes that his last measles-mumps-rubella (MMR) vaccination was at 15 months of age. What recommendation should the nurse make? a. No further MMR immunizations are needed. b. MMR vaccination needs to be repeated at 4 to 6 years of age. c. MMR immunization needs to be repeated every 4 years until age 21 years. d. A recommendation cannot be made until the physician is consulted. In obtaining a review of systems on a healthy 7-year-old girl, the health care provider knows that it would be important to include the: a. Last glaucoma examination. b. Frequency of breast self-examinations. c. Date of her last electrocardiogram. d. Limitations related to her involvement in sports activities. When the nurse asks for a description of who lives with a child, the method of discipline, and the support system of the child, what part of the assessment is being performed? a. Family history b. Review of systems c. Functional assessment d. Reason for seeking care The nurse is obtaining a health history on an 87-year-old woman. Which of the following areas of questioning would be most useful at this time? a. Obstetric history b. Childhood illnesses c. General health for the past 20 years d. Current health promotion activities The nurse is performing a review of systems on a 76-year-old patient. Which of these statements is correct for this situation? a. The questions asked are identical for all ages. b. The interviewer will start incorporating different questions for patients 70 years of age and older. c. Questions that are reflective of the normal effects of aging are added. d. At this age, a review of systems is not necessarythe focus should be on current problems. A 90-year-old patient tells the nurse that he cannot remember the names of the medications he is taking or for what reason he is taking them. An appropriate response from the nurse would be: a. Can you tell me what they look like? b. Dont worry about it. You are only taking two medications. c. How long have you been taking each of the pills? d. Would you have a family member bring in your medications? The nurse is performing a functional assessment on an 82-year-old patient who recently had a stroke. Which of these questions would be most important to ask? a. Do you wear glasses? b. Are you able to dress yourself? c. Do you have any thyroid problems? d. How many times a day do you have a bowel movement? The nurse is preparing to do a functional assessment. Which statement best describes the purpose of a functional assessment? a. The functional assessment assesses how the individual is coping with life at home. b. It determines how children are meeting developmental milestones. c. The functional assessment can identify any problems with memory the individual may be experiencing. d. It helps determine how a person is managing day-to-day activities. The nurse is asking a patient for his reason for seeking care and asks about the signs and symptoms he is experiencing. Which of these is an example of a symptom? a. Chest pain b. Clammy skin c. Serum potassium level at 4.2 mEq/L d. Body temperature of 100 F A patient is describing his symptoms to the nurse. Which of these statements reflects a description of the setting of his symptoms? a. It is a sharp, burning pain in my stomach. b. I also have the sweats and nausea when I feel this pain. c. I think this pain is telling me that something bad is wrong with me. d. This pain happens every time I sit down to use the computer. During an assessment, the nurse uses the CAGE test. The patient answers yes to two of the questions. What could this be indicating? a. The patient is an alcoholic. b. The patient is annoyed at the questions. c. The patient should be thoroughly examined for possible alcohol withdrawal symptoms. d. The nurse should suspect alcohol abuse and continue with a more thorough substance abuse assessment. The nurse is incorporating a persons spiritual values into the health history. Which of these questions illustrates the community portion of the FICA (faith and belief, importance and influence, community, and addressing or applying in care) questions? a. Do you believe in God? b. Are you a part of any religious or spiritual congregation? c. Do you consider yourself to be a religious or spiritual person? d. How does your religious faith influence the way you think about your health? The nurse is preparing to complete a health assessment on a 16-year-old girl whose parents have brought her to the clinic. Which instruction would be appropriate for the parents before the interview begins? a. Please stay during the interview; you can answer for her if she does not know the answer. b. It would help to interview the three of you together. c. While I interview your daughter, will you please stay in the room and complete these family health history questionnaires? d. While I interview your daughter, will you step out to the waiting room and complete these family health history questionnaires? The nurse is assessing a new patient who has recently immigrated to the United States. Which question is appropriate to add to the health history? a. Why did you come to the United States? b. When did you come to the United States and from what country? c. What made you leave your native country? d. Are you planning to return to your home? The nurse is conducting an interview with a woman who has recently learned that she is pregnant and who has come to the clinic today to begin prenatal care. The woman states that she and her husband are excited about the pregnancy but have a few questions. She looks nervously at her hands during the interview and sighs loudly. Considering the concept of communication, which statement does the nurse know to be most accurate? The woman is: a. Excited about her pregnancy but nervous about the labor. b. Exhibiting verbal and nonverbal behaviors that do not match. c. Excited about her pregnancy, but her husband is not and this is upsetting to her. d. Not excited about her pregnancy but believes the nurse will negatively respond to her if she states this. Receiving is a part of the communication process. Which receiver is most likely to misinterpret a message sent by a health care professional? a. Well-adjusted adolescent who came in for a sports physical b. Recovering alcoholic who came in for a basic physical examination c. Man whose wife has just been diagnosed with lung cancer d. Man with a hearing impairment who uses sign language to communicate and who has an interpreter with him The nurse makes which adjustment in the physical environment to promote the success of an interview? a. Reduces noise by turning off televisions and radios b. Reduces the distance between the interviewer and the patient to 2 feet or less c. Provides a dim light that makes the room cozy and helps the patient relax d. Arranges seating across a desk or table to allow the patient some personal space In an interview, the nurse may find it necessary to take notes to aid his or her memory later. Which statement is true regarding note-taking? a. Note-taking may impede the nurses observation of the patients nonverbal behaviors. b. Note-taking allows the patient to continue at his or her own pace as the nurse records what is said. c. Note-taking allows the nurse to shift attention away from the patient, resulting in an increased comfort level. d. Note-taking allows the nurse to break eye contact with the patient, which may increase his or her level of comfort. The nurse asks, I would like to ask you some questions about your health and your usual daily activities so that we can better plan your stay here. This question is found at the __________ phase of the interview process. a. Summary b. Closing c. Body d. Opening or introduction A woman has just entered the emergency department after being battered by her husband. The nurse needs to get some information from her to begin treatment. What is the best choice for an opening phase of the interview with this patient? a. Hello, Nancy, my name is Mrs. C. b. Hello, Mrs. H., my name is Mrs. C. It sure is cold today! c. Mrs. H., my name is Mrs. C. How are you? d. Mrs. H., my name is Mrs. C. Ill need to ask you a few questions about what happened. During an interview, the nurse states, You mentioned having shortness of breath. Tell me more about that. Which verbal skill is used with this statement? a. Reflection b. Facilitation c. Direct question d. Open-ended question A patient has finished giving the nurse information about the reason he is seeking care. When reviewing the data, the nurse finds that some information about past hospitalizations is missing. At this point, which statement by the nurse would be most appropriate to gather these data? a. Mr. Y., at your age, surely you have been hospitalized before! b. Mr. Y., I just need permission to get your medical records from County Medical. c. Mr. Y., you mentioned that you have been hospitalized on several occasions. Would you tell me more about that? d. Mr. Y., I just need to get some additional information about your past hospitalizations. When was the last time you were admitted for chest pain? In using verbal responses to assist the patients narrative, some responses focus on the patients frame of reference and some focus on the health care providers perspective. An example of a verbal response that focuses on the health care providers perspective would be: a. Empathy. b. Reflection. c. Facilitation. d. Confrontation. When taking a history from a newly admitted patient, the nurse notices that he often pauses and expectantly looks at the nurse. What would be the nurses best response to this behavior? a. Be silent, and allow him to continue when he is ready. b. Smile at him and say, Dont worry about all of this. Im sure we can find out why youre having these pains. c. Lean back in the chair and ask, You are looking at me kind of funny; there isnt anything wrong, is there? d. Stand up and say, I can see that this interview is uncomfortable for you. We can continue it another time. A woman is discussing the problems she is having with her 2-year-old son. She says, He wont go to sleep at night, and during the day he has several fits. I get so upset when that happens. The nurses best verbal response would be: a. Go on, Im listening. b. Fits? Tell me what you mean by this. c. Yes, it can be upsetting when a child has a fit. d. Dont be upset when he has a fit; every 2 year old has fits. A 17-year-old single mother is describing how difficult it is to raise a 3-year-old child by herself. During the course of the interview she states, I cant believe my boyfriend left me to do this by myself! What a terrible thing to do to me! Which of these responses by the nurse uses empathy? a. You feel alone. b. You cant believe he left you alone? c. It must be so hard to face this all alone. d. I would be angry, too; raising a child alone is no picnic. A man has been admitted to the observation unit for observation after being treated for a large cut on his forehead. As the nurse works through the interview, one of the standard questions has to do with alcohol, tobacco, and drug use. When the nurse asks him about tobacco use, he states, I quit smoking after my wife died 7 years ago. However, the nurse notices an open pack of cigarettes in his shirt pocket. Using confrontation, the nurse could say: a. Mr. K., I know that you are lying. b. Mr. K., come on, tell me how much you smoke. c. Mr. K., I didnt realize your wife had died. It must be difficult for you at this time. Please tell me more about that. d. Mr. K., you have said that you dont smoke, but I see that you have an open pack of cigarettes in your pocket. The nurse has used interpretation regarding a patients statement or actions. After using this technique, it would be best for the nurse to: a. Apologize, because using interpretation can be demeaning for the patient. b. Allow time for the patient to confirm or correct the inference. c. Continue with the interview as though nothing has happened. d. Immediately restate the nurses conclusion on the basis of the patients nonverbal response. During an interview, a woman says, I have decided that I can no longer allow my children to live with their fathers violence, but I just cant seem to leave him. Using interpretation, the nurses best response would be: a. You are going to leave him? b. If you are afraid for your children, then why cant you leave? c. It sounds as if you might be afraid of how your husband will respond. d. It sounds as though you have made your decision. I think it is a good one. A pregnant woman states, I just know labor will be so painful that I wont be able to stand it. I know it sounds awful, but I really dread going into labor. The nurse responds by stating, Oh, dont worry about labor so much. I have been through it, and although it is painful, many good medications are available to decrease the pain. Which statement is true regarding this response? The nurses reply was a: a. Therapeutic response. By sharing something personal, the nurse gives hope to this woman. b. Nontherapeutic response. By providing false reassurance, the nurse actually cut off further discussion of the womans fears. c. Therapeutic response. By providing information about the medications available, the nurse is giving information to the woman. d. Nontherapeutic response. The nurse is essentially giving the message to the woman that labor cannot be tolerated without medication. . During a visit to the clinic, a patient states, The doctor just told me he thought I ought to stop smoking. He doesnt understand how hard Ive tried. I just dont know the best way to do it. What should I do? The nurses most appropriate response in this case would be: a. Id quit if I were you. The doctor really knows what he is talking about. b. Would you like some information about the different ways a person can quit smoking? c. Stopping your dependence on cigarettes can be very difficult. I understand how you feel. d. Why are you confused? Didnt the doctor give you the information about the smoking cessation program we offer? As the nurse enters a patients room, the nurse finds her crying. The patient states that she has just found out that the lump in her breast is cancer and says, Im so afraid of, um, you know. The nurses most therapeutic response would be to say in a gentle manner: a. Youre afraid you might lose your breast? b. No, Im not sure what you are talking about. c. Ill wait here until you get yourself under control, and then we can talk. d. I can see that you are very upset. Perhaps we should discuss this later. A nurse is taking complete health histories on all of the patients attending a wellness workshop. On the history form, one of the written questions asks, You dont smoke, drink, or take drugs, do you? This question is an example of: a. Talking too much. b. Using confrontation. c. Using biased or leading questions. d. Using blunt language to deal with distasteful topics. When observing a patients verbal and nonverbal communication, the nurse notices a discrepancy. Which statement is true regarding this situation? The nurse should: a. Ask someone who knows the patient well to help interpret this discrepancy. b. Focus on the patients verbal message, and try to ignore the nonverbal behaviors. c. Try to integrate the verbal and nonverbal messages and then interpret them as an average. d. Focus on the patients nonverbal behaviors, because these are often more reflective of a patients true feelings. During an interview, a parent of a hospitalized child is sitting in an open position. As the interviewer begins to discuss his sons treatment, however, he suddenly crosses his arms against his chest and crosses his legs. This changed posture would suggest that the parent is: a. Simply changing positions. b. More comfortable in this position. c. Tired and needs a break from the interview. d. Uncomfortable talking about his sons treatment. A mother brings her 28-month-old daughter into the clinic for a well-child visit. At the beginning of the visit, the nurse focuses attention away from the toddler, but as the interview progresses, the toddler begins to warm up and is smiling shyly at the nurse. The nurse will be most successful in interacting with the toddler if which is done next? a. Tickle the toddler, and get her to laugh. b. Stoop down to her level, and ask her about the toy she is holding. c. Continue to ignore her until it is time for the physical examination. d. Ask the mother to leave during the examination of the toddler, because toddlers often fuss less if their parent is not in view. During an examination of a 3-year-old child, the nurse will need to take her blood pressure. What might the nurse do to try to gain the childs full cooperation? a. Tell the child that the blood pressure cuff is going to give her arm a big hug. b. Tell the child that the blood pressure cuff is asleep and cannot wake up. c. Give the blood pressure cuff a name and refer to it by this name during the assessment. d. Tell the child that by using the blood pressure cuff, we can see how strong her muscles are. A 16-year-old boy has just been admitted to the unit for overnight observation after being in an automobile accident. What is the nurses best approach to communicating with him? a. Use periods of silence to communicate respect for him. b. Be totally honest with him, even if the information is unpleasant. c. Tell him that everything that is discussed will be kept totally confidential. d. Use slang language when possible to help him open up. A 75-year-old woman is at the office for a preoperative interview. The nurse is aware that the interview may take longer than interviews with younger persons. What is the reason for this? a. An aged person has a longer story to tell. b. An aged person is usually lonely and likes to have someone with whom to talk. c. Aged persons lose much of their mental abilities and require longer time to complete an interview. d. As a person ages, he or she is unable to hear; thus the interviewer usually needs to repeat much of what is said The nurse is interviewing a male patient who has a hearing impairment. What techniques would be most beneficial in communicating with this patient? a. Determine the communication method he prefers. b. Avoid using facial and hand gestures because most hearing-impaired people find this degrading. c. Request a sign language interpreter before meeting with him to help facilitate the communication. d. Speak loudly and with exaggerated facial movement when talking with him because doing so will help him lip read. During a prenatal check, a patient begins to cry as the nurse asks her about previous pregnancies. She states that she is remembering her last pregnancy, which ended in miscarriage. The nurses best response to her crying would be: a. Im so sorry for making you cry! b. I can see that you are sad remembering this. It is all right to cry. c. Why dont I step out for a few minutes until youre feeling better? d. I can see that you feel sad about this; why dont we talk about something else? A female nurse is interviewing a man who has recently immigrated. During the course of the interview, he leans forward and then finally moves his chair close enough that his knees are nearly touching the nurses knees. The nurse begins to feel uncomfortable with his proximity. Which statement most closely reflects what the nurse should do next? a. The nurse should try to relax; these behaviors are culturally appropriate for this person. b. The nurse should discreetly move his or her chair back until the distance is more comfortable, and then continue with the interview. c. These behaviors are indicative of sexual aggression, and the nurse should confront this person about his behaviors. d. The nurse should laugh but tell him that he or she is uncomfortable with his proximity and ask him to move away. A female American Indian has come to the clinic for follow-up diabetic teaching. During the interview, the nurse notices that she never makes eye contact and speaks mostly to the floor. Which statement is true regarding this situation? a. The woman is nervous and embarrassed. b. She has something to hide and is ashamed. c. The woman is showing inconsistent verbal and nonverbal behaviors. d. She is showing that she is carefully listening to what the nurse is saying. The nurse is performing a health interview on a patient who has a language barrier, and no interpreter is available. Which is the best example of an appropriate question for the nurse to ask in this situation? a. Do you take medicine? b. Do you sterilize the bottles? c. Do you have nausea and vomiting? d. You have been taking your medicine, havent you? A man arrives at the clinic for his annual wellness physical. He is experiencing no acute health problems. Which question or statement by the nurse is most appropriate when beginning the interview? a. How is your family? b. How is your job? c. Tell me about your hypertension. d. How has your health been since your last visit? . The nurse makes this comment to a patient, I know it may be hard, but you should do what the doctor ordered because she is the expert in this field. Which statement is correct about the nurses comment? a. This comment is inappropriate because it shows the nurses bias. b. This comment is appropriate because members of the health care team are experts in their area of patient care. c. This type of comment promotes dependency and inferiority on the part of the patient and is best avoided in an interview situation. d. Using authority statements when dealing with patients, especially when they are undecided about an issue, is necessary at times. A female patient does not speak English well, and the nurse needs to choose an interpreter. Which of the following would be the most appropriate choice? a. Trained interpreter b. Male family member c. Female family member d. Volunteer college student from the foreign language studies department . During a follow-up visit, the nurse discovers that a patient has not been taking his insulin on a regular basis. The nurse asks, Why havent you taken your insulin? Which statement is an appropriate evaluation of this question? a. This question may place the patient on the defensive. b. This question is an innocent search for information. c. Discussing his behavior with his wife would have been better. d. A direct question is the best way to discover the reasons for his behavior. The nurse is nearing the end of an interview. Which statement is appropriate at this time? a. Did we forget something? b. Is there anything else you would like to mention? c. I need to go on to the next patient. Ill be back. d. While Im here, lets talk about your upcoming surgery. During the interview portion of data collection, the nurse collects __________ data. a. Physical b. Historical c. Objective d. Subjective During an interview, the nurse would expect that most of the interview will take place at what distance? a. Intimate zone b. Personal distance c. Social distance d. Public distance A female nurse is interviewing a male patient who is near the same age as the nurse. During the interview, the patient makes an overtly sexual comment. The nurses best reaction would be: a. Stop that immediately! b. Oh, you are too funny. Lets keep going with the interview. c. Do you really think I would be interested? d. It makes me uncomfortable when you talk that way. Please stop. The nurse is reviewing the development of culture. Which statement is correct regarding the development of ones culture? Culture is: a. Genetically determined on the basis of racial background. b. Learned through language acquisition and socialization. c. A nonspecific phenomenon and is adaptive but unnecessary. d. Biologically determined on the basis of physical characteristics. During a class on the aspects of culture, the nurse shares that culture has four basic characteristics. Which statement correctly reflects one of these characteristics? a. Cultures are static and unchanging, despite changes around them. b. Cultures are never specific, which makes them hard to identify. c. Culture is most clearly reflected in a persons language and behavior. d. Culture adapts to specific environmental factors and available natural resources. During a seminar on cultural aspects of nursing, the nurse recognizes that the definition stating the specific and distinct knowledge, beliefs, skills, and customs acquired by members of a society reflects which term? a. Mores b. Norms c. Culture d. Social learning When discussing the use of the term subculture, the nurse recognizes that it is best described as: a. Fitting as many people into the majority culture as possible. b. Defining small groups of people who do not want to be identified with the larger culture. c. Singling out groups of people who suffer differential and unequal treatment as a result of cultural variations. d. Identifying fairly large groups of people with shared characteristics that are not common to all members of a culture. When reviewing the demographics of ethnic groups in the United States, the nurse recalls that the largest and fastest growing population is: a. Hispanic. b. Black. c. Asian. d. American Indian. During an assessment, the nurse notices that a patient is handling a small charm that is tied to a leather strip around his neck. Which action by the nurse is appropriate? a. Ask the patient about the item and its significance. b. Ask the patient to lock the item with other valuables in the hospitals safe. c. Tell the patient that a family member should take valuables home. d. No action is necessary. The nurse manager is explaining culturally competent care during a staff meeting. Which statement accurately describes the concept of culturally competent care? The caregiver: a. Is able to speak the patients native language. b. Possesses some basic knowledge of the patients cultural background. c. Applies the proper background knowledge of a patients cultural background to provide the best possible health care. d. Understands and attends to the total context of the patients situation. The nurse recognizes that an example of a person who is heritage consistent would be a: a. Woman who has adapted her clothing to the clothing style of her new country. b. Woman who follows the traditions that her mother followed regarding meals. c. Man who is not sure of his ancestors country of origin. d. Child who is not able to speak his parents native language. After a class on culture and ethnicity, the new graduate nurse reflects a correct understanding of the concept of ethnicity with which statement? a. Ethnicity is dynamic and ever changing. b. Ethnicity is the belief in a higher power. c. Ethnicity pertains to a social group within the social system that claims shared values and traditions. d. Ethnicity is learned from birth through the processes of language acquisition and socialization. The nurse is comparing the concepts of religion and spirituality. Which of the following is an appropriate component of ones spirituality? a. Belief in and the worship of God or gods b. Attendance at a specific church or place of worship c. Personal effort made to find purpose and meaning in life d. Being closely tied to ones ethnic background . A woman who has lived in the United States for a year after moving from Europe has learned to speak English and is almost finished with her college studies. She now dresses like her peers and says that her family in Europe would hardly recognize her. This nurse recognizes that this situation illustrates which concept? a. Assimilation b. Heritage consistency c. Biculturalism d. Acculturation The nurse is conducting a heritage assessment. Which question is most appropriate for this assessment? a. What is your religion? b. Do you mostly participate in the religious traditions of your family? c. Do you smoke? d. Do you have a history of heart disease? In the majority culture of America, coughing, sweating, and diarrhea are symptoms of an illness. For some individuals of Mexican-American origin, however, these symptoms are a normal part of living. The nurse recognizes that this difference is true, probably because Mexican-Americans: a. Have less efficient immune systems and are often ill. b. Consider these symptoms part of normal living, not symptoms of ill health. c. Come from Mexico, and coughing is normal and healthy there. d. Are usually in a lower socioeconomic group and are more likely to be sick The nurse is reviewing theories of illness. The germ theory, which states that microscopic organisms such as bacteria and viruses are responsible for specific disease conditions, is a basic belief of which theory of illness? a. Holistic b. Biomedical c. Naturalistic d. Magicoreligious An Asian-American woman is experiencing diarrhea, which is believed to be cold or yin. The nurse expects that the woman is likely to try to treat it with: a. Foods that are hot or yang. b. Readings and Eastern medicine meditations. c. High doses of medicines believed to be cold. d. No treatment is tried because diarrhea is an expected part of life. Many Asians believe in the yin/yang theory, which is rooted in the ancient Chinese philosophy of Tao. Which statement most accurately reflects health in an Asian with this belief? a. A person is able to work and produce. b. A person is happy, stable, and feels good. c. All aspects of the person are in perfect balance. d. A person is able to care for others and function socially. Illness is considered part of lifes rhythmic course and is an outward sign of disharmony within. This statement most accurately reflects the views about illness from which theory? a. Naturalistic b. Biomedical c. Reductionist d. Magicoreligious An individual who takes the magicoreligious perspective of illness and disease is likely to believe that his or her illness was caused by: a. Germs and viruses. b. Supernatural forces. c. Eating imbalanced foods. d. An imbalance within his or her spiritual nature. If an American Indian woman has come to the clinic to seek help with regulating her diabetes, then the nurse can expect that she: a. Will comply with the treatment prescribed. b. Has obviously given up her belief in naturalistic causes of disease. c. May also be seeking the assistance of a shaman or medicine man. d. Will need extra help in dealing with her illness and may be experiencing a crisis of faith. An older Mexican-American woman with traditional beliefs has been admitted to an inpatient care unit. A culturally sensitive nurse would: a. Contact the hospital administrator about the best course of action. b. Automatically get a curandero for her, because requesting one herself is not culturally appropriate. c. Further assess the patients cultural beliefs and offer the patient assistance in contacting a curandero or priest if she desires. d. Ask the family what they would like to do because Mexican-Americans traditionally give control of decision making to their families. A 63-year-old Chinese-American man enters the hospital with complaints of chest pain, shortness of breath, and palpitations. Which statement most accurately reflects the nurses best course of action? a. The nurse should focus on performing a full cardiac assessment. b. The nurse should focus on psychosomatic complaints because the patient has just learned that his wife has cancer. c. This patient is not in any danger at present; therefore, the nurse should send him home with instructions to contact his physician. d. It is unclear what is happening with this patient; consequently, the nurse should perform an assessment in both the physical and the psychosocial realms. Symptoms, such as pain, are often influenced by a persons cultural heritage. Which of the following is a true statement regarding pain? a. Nurses attitudes toward their patients pain are unrelated to their own experiences with pain. b. Nurses need to recognize that many cultures practice silent suffering as a response to pain. c. A nurses area of clinical practice will most likely determine his or her assessment of a patients pain. d. A nurses years of clinical experience and current position are strong indicators of his or her response to patient pain. The nurse is reviewing concepts of cultural aspects of pain. Which statement is true regarding pain? a. All patients will behave the same way when in pain. b. Just as patients vary in their perceptions of pain, so will they vary in their expressions of pain. c. Cultural norms have very little to do with pain tolerance, because pain tolerance is always biologically determined. d. A patients expression of pain is largely dependent on the amount of tissue injury associated with the pain. During a class on religion and spirituality, the nurse is asked to define spirituality. Which answer is correct? Spirituality: a. Is a personal search to discover a supreme being. b. Is an organized system of beliefs concerning the cause, nature, and purpose of the universe. c. Is a belief that each person exists forever in some form, such as a belief in reincarnation or the afterlife. d. Arises out of each persons unique life experience and his or her personal effort to find purpose in life. The nurse recognizes that working with children with a different cultural perspective may be especially difficult because: a. Children have spiritual needs that are influenced by their stages of development. b. Children have spiritual needs that are direct reflections of what is occurring in their homes. c. Religious beliefs rarely affect the parents perceptions of the illness. d. Parents are often the decision makers, and they have no knowledge of their childrens spiritual needs. . A 30-year-old woman has recently moved to the United States with her husband. They are living with the womans sister until they can get a home of their own. When company arrives to visit with the womans sister, the woman feels suddenly shy and retreats to the back bedroom to hide until the company leaves. She explains that her reaction to guests is simply because she does not know how to speak perfect English. This woman could be experiencing: a. Culture shock. b. Cultural taboos. c. Cultural unfamiliarity. d. Culture disorientation. After a symptom is recognized, the first effort at treatment is often self-care. Which of the following statements about self-care is true? Self-care is: a. Not recognized as valuable by most health care providers. b. Usually ineffective and may delay more effective treatment. c. Always less expensive than biomedical alternatives. d. Influenced by the accessibility of over-the-counter medicines. The nurse is reviewing the hot/cold theory of health and illness. Which statement best describes the basic tenets of this theory? a. The causation of illness is based on supernatural forces that influence the humors of the body. b. Herbs and medicines are classified on their physical characteristics of hot and cold and the humors of the body. c. The four humors of the body consist of blood, yellow bile, spiritual connectedness, and social aspects of the individual. d. The treatment of disease consists of adding or subtracting cold, heat, dryness, or wetness to restore the balance of the humors of the body. In the hot/cold theory, illnesses are believed to be caused by hot or cold entering the body. Which of these patient conditions is most consistent with a cold condition? a. Patient with diabetes and renal failure b. Teenager with an abscessed tooth c. Child with symptoms of itching and a rash d. Older man with gastrointestinal discomfort . When providing culturally competent care, nurses must incorporate cultural assessments into their health assessments. Which statement is most appropriate to use when initiating an assessment of cultural beliefs with an older American-Indian patient? a. Are you of the Christian faith? b. Do you want to see a medicine man? c. How often do you seek help from medical providers? d. What cultural or spiritual beliefs are important to you? . During a class on cultural practices, the nurse hears the term cultural taboo. Which statement illustrates the concept of a cultural taboo? a. Believing that illness is a punishment of sin b. Trying prayer before seeking medical help c. Refusing to accept blood products as part of treatment d. Stating that a childs birth defect is the result of the parents sins The nurse recognizes that categories such as ethnicity, gender, and religion illustrate the concept of: a. Family. b. Cultures. c. Spirituality. d. Subcultures. After completing an initial assessment of a patient, the nurse has charted that his respirations are eupneic and his pulse is 58 beats per minute. These types of data would be: a. Objective. b. Reflective. c. Subjective. d. Introspective. A patient tells the nurse that he is very nervous, is nauseated, and feels hot. These types of data would be: a. Objective. b. Reflective. c. Subjective. d. Introspective. The patients record, laboratory studies, objective data, and subjective data combine to form the: a. Data base. b. Admitting data. c. Financial statement. d. Discharge summary. When listening to a patients breath sounds, the nurse is unsure of a sound that is heard. The nurses next action should be to: a. Immediately notify the patients physician. b. Document the sound exactly as it was heard. c. Validate the data by asking a coworker to listen to the breath sounds. d. Assess again in 20 minutes to note whether the sound is still present. The nurse is conducting a class for new graduate nurses. During the teaching session, the nurse should keep in mind that novice nurses, without a background of skills and experience from which to draw, are more likely to make their decisions using: a. Intuition. b. A set of rules. c. Articles in journals. d. Advice from supervisors. Expert nurses learn to attend to a pattern of assessment data and act without consciously labeling it. These responses are referred to as: a. Intuition. b. The nursing process. c. Clinical knowledge. d. Diagnostic reasoning. The nurse is reviewing information about evidence-based practice (EBP). Which statement best reflects EBP? a. EBP relies on tradition for support of best practices. b. EBP is simply the use of best practice techniques for the treatment of patients. c. EBP emphasizes the use of best evidence with the clinicians experience. d. The patients own preferences are not important with EBP. The nurse is conducting a class on priority setting for a group of new graduate nurses. Which is an example of a first-level priority problem? a. Patient with postoperative pain b. Newly diagnosed patient with diabetes who needs diabetic teaching c. Individual with a small laceration on the sole of the foot d. Individual with shortness of breath and respiratory distress When considering priority setting of problems, the nurse keeps in mind that second-level priority problems include which of these aspects? a. Low self-esteem b. Lack of knowledge c. Abnormal laboratory values d. Severely abnormal vital signs Which critical thinking skill helps the nurse see relationships among the data? a. Validation b. Clustering related cues c. Identifying gaps in data d. Distinguishing relevant from irrelevant The nurse knows that developing appropriate nursing interventions for a patient relies on the appropriateness of the __________ diagnosis. a. Nursing b. Medical c. Admission d. Collaborative The nursing process is a sequential method of problem solving that nurses use and includes which steps? a. Assessment, treatment, planning, evaluation, discharge, and follow-up b. Admission, assessment, diagnosis, treatment, and discharge planning c. Admission, diagnosis, treatment, evaluation, and discharge planning d. Assessment, diagnosis, outcome identification, planning, implementation, and evaluation A newly admitted patient is in acute pain, has not been sleeping well lately, and is having difficulty breathing. How should the nurse prioritize these problems? a. Breathing, pain, and sleep b. Breathing, sleep, and pain c. Sleep, breathing, and pain d. Sleep, pain, and breathing Which of these would be formulated by a nurse using diagnostic reasoning? a. Nursing diagnosis b. Medical diagnosis c. Diagnostic hypothesis d. Diagnostic assessment Barriers to incorporating EBP include: a. Nurses lack of research skills in evaluating the quality of research studies. b. Lack of significant research studies. c. Insufficient clinical skills of nurses. d. Inadequate physical assessment skills. What step of the nursing process includes data collection by health history, physical examination, and interview? a. Planning b. Diagnosis c. Evaluation d. Assessment During a staff meeting, nurses discuss the problems with accessing research studies to incorporate evidence-based clinical decision making into their practice. Which suggestion by the nurse manager would best help these problems? a. Form a committee to conduct research studies. b. Post published research studies on the units bulletin boards. c. Encourage the nurses to visit the library to review studies. d. Teach the nurses how to conduct electronic searches for research studies. When reviewing the concepts of health, the nurse recalls that the components of holistic health include which of these? a. Disease originates from the external environment. b. The individual human is a closed system. c. Nurses are responsible for a patients health state. d. Holistic health views the mind, body, and spirit as interdependent. The nurse recognizes that the concept of prevention in describing health is essential because: a. Disease can be prevented by treating the external environment. b. The majority of deaths among Americans under age 65 years are not preventable. c. Prevention places the emphasis on the link between health and personal behavior. d. The means to prevention is through treatment provided by primary health care practitioners. The nurse is performing a physical assessment on a newly admitted patient. An example of objective information obtained during the physical assessment includes the: a. Patients history of allergies. b. Patients use of medications at home. c. Last menstrual period 1 month ago. d. 2 5 cm scar on the right lower forearm. A visiting nurse is making an initial home visit for a patient who has many chronic medical problems. Which type of data base is most appropriate to collect in this setting? a. A follow-up data base to evaluate changes at appropriate intervals b. An episodic data base because of the continuing, complex medical problems of this patient c. A complete health data base because of the nurses primary responsibility for monitoring the patients health d. An emergency data base because of the need to collect information and make accurate diagnoses rapidly Which situation is most appropriate during which the nurse performs a focused or problem-centered history? a. Patient is admitted to a long-term care facility. b. Patient has a sudden and severe shortness of breath. c. Patient is admitted to the hospital for surgery the following day. d. Patient in an outpatient clinic has cold and influenza-like symptoms. A patient is at the clinic to have her blood pressure checked. She has been coming to the clinic weekly since she changed medications 2 months ago. The nurse should: a. Collect a follow-up data base and then check her blood pressure. b. Ask her to read her health record and indicate any changes since her last visit. c. Check only her blood pressure because her complete health history was documented 2 months ago. d. Obtain a complete health history before checking her blood pressure because much of her history information may have changed. A patient is brought by ambulance to the emergency department with multiple traumas received in an automobile accident. He is alert and cooperative, but his injuries are quite severe. How would the nurse proceed with data collection? a. Collect history information first, then perform the physical examination and institute life-saving measures. b. Simultaneously ask history questions while performing the examination and initiating life-saving measures. c. Collect all information on the history form, including social support patterns, strengths, and coping patterns. d. Perform life-saving measures and delay asking any history questions until the patient is transferred to the intensive care unit. A 42-year-old patient of Asian descent is being seen at the clinic for an initial examination. The nurse knows that including cultural information in his health assessment is important to: a. Identify the cause of his illness. b. Make accurate disease diagnoses. c. Provide cultural health rights for the individual. d. Provide culturally sensitive and appropriate care. In the health promotion model, the focus of the health professional includes: a. Changing the patients perceptions of disease. b. Identifying biomedical model interventions. c. Identifying negative health acts of the consumer. d. Helping the consumer choose a healthier lifestyle. The nurse has implemented several planned interventions to address the nursing diagnosis of acute pain. Which would be the next appropriate action? a. Establish priorities. b. Identify expected outcomes. c. Evaluate the individuals condition, and compare actual outcomes with expected outcomes. d. Interpret data, and then identify clusters of cues and make inferences. Which statement best describes a proficient nurse? A proficient nurse is one who: a. Has little experience with a specified population and uses rules to guide performance. b. Has an intuitive grasp of a clinical situation and quickly identifies the accurate solution. c. Sees actions in the context of daily plans for patients. d. Understands a patient situation as a whole rather than a list of tasks and recognizes the long-term goals for the patient. . The nurse is reviewing data collected after an assessment. Of the data listed below, which would be considered related cues that would be clustered together during data analysis? Select all that apply. a. Inspiratory wheezes noted in left lower lobes b. Hypoactive bowel sounds c. Nonproductive cough d. Edema, +2, noted on left hand e. Patient reports dyspnea upon exertion f. Rate of respirations 16 breaths per minute C E F a. A patient newly diagnosed with type 2 diabetes mellitus does not know how to check his own blood glucose levels with a glucometer. b. A teenager who was stung by a bee during a soccer match is having trouble breathing. c. An older adult with a urinary tract infection is also showing signs of confusion and agitation. problem - 1. a = First-level priority problem 2. b = Second-level priority problem 3. c = Third-level priority 1. The nurse is assessing an older adult's functional ability. Which definition correctly describes one's functional ability? Functional ability: A. Is the measure of the expected changes of aging that one is experiencing. B. Refers to the individual's motivation to live independently. C. Refers to the level of cognition present in an older person. D. Refers to one's ability to perform activities necessary to live in modern society. Functional ability refers to one's ability to perform activities necessary to live in modern society and can include driving, using the telephone, or performing personal tasks such as bathing and toileting. 2. The nurse is preparing to perform a functional assessment of an older patient and knows that a good approach would be to: A. Observe the patient's ability to perform the tasks. B. Ask the patient's wife how he does when performing tasks. C. Review the medical record for information on the patient's abilities. D. Ask the patient's physician for information on the patient's abilities. There are two approaches for performing a functional assessment, asking individuals about their abilities to perform the tasks (using self-reports) or actually observing their ability to perform the tasks. For persons with memory problems, the use of surrogate reporters (proxy reports) such as family members or caregivers may be necessary, keeping in mind that they may either overestimate or underestimate their actual abilities. 3. The nurse needs to assess a patient's ability to perform activities of daily living and should choose which tool for this assessment? A. Direct Assessment of Functional Abilities (DAFA) B. Lawton IADL C. Barthel Index D. Older Americans Resources and Services Multidimensional Functional Assessment Questionnaire-IADL (OARS-IADL) The Barthel Index is used to assess activities of daily living. The other options are used to measure instrumental activities of daily living 4. The nurse is preparing to use the Lawton IADL instrument as part of an assessment. Which statement about the Lawton IADL instrument is true? A. The nurse uses direct observation to implement this tool. B. It is designed as a self-report measure of performance rather than ability. C. It is not useful in the acute hospital setting. D. It is best used for those residing in an institutional setting - The Lawton IADL instrument is designed as a self-report measure of performance rather than ability. Direct testing is often not feasible, such as demonstrating the ability to prepare food while a hospital inpatient. Attention to the final score is less important than identifying a person's strengths and areas where assistance is needed. The instrument is useful in acute hospital settings for discharge planning and continuously in outpatient settings. It would not be useful for those residing in institutional settings because many of these tasks are already being managed for the resident. 5. The nurse is assessing an older adult's advanced activities of daily living, which would include: A. Recreational activities. B. Meal preparation. C. Balancing the checkbook. D. Self-grooming activities. Advanced activities of daily living (AADL) are activities that an older adult performs such as occupational and recreational activities. Self-grooming activities are basic activities of daily living (ADLs); meal preparation and balancing the checkbook are considered instrumental activities of daily living (IADLs) 6. When using the various instruments to assess an older person's activities of daily living (ADLs), the nurse needs to remember that a disadvantage of these instruments includes: A. The reliability of the tools. B. Self or proxy report of functional activities. C. Lack of confidentiality during the assessment. D. Insufficient detail about the deficiencies identified. - A disadvantage of many of the ADL and IADL instruments is the self or proxy report of functional activities. The other responses are not correct. 7. The nurse is administering a test that is timed over 15 minutes and assesses a patient's upper body fine motor and coarse motor activities, balance, mobility, coordination, and endurance. During this test, activities such as dressing and stair climbing are timed. Which test is described by these activities? A. The Get Up and Go Test B. The Performance Activities of Daily Living C. The Physical Performance Test D. Tinetti Gait and Balance Evaluation The Physical Performance Test is appropriate for use with community-dwelling older adults. The test requires approximately 15 minutes to complete and assesses upper body fine motor and coarse motor activities, balance, mobility, coordination, and endurance. Activities such as eating, dressing and transferring, and stair climbing are simulated and timed. 8. A patient will be ready to be discharged from the hospital soon, and the patient's family membersare concerned about whether the patient is able to walk outside alone safely. The nurse will perform which test to assess this? A. The Get Up and Go Test B. The Performance Activities of Daily Living C. The Physical Performance Test D. Tinetti Gait and Balance Evaluation The Get Up and Go Test is a reliable and valid test to quantify functional mobility. The test is quick, requires little training and no special equipment, and is appropriate to use in many settingsincluding hospitals and clinics. This instrument has been shown to predict a person's ability to gooutside alone safely. The Performance of Activities of Daily Living test has a trained observer actually observing as a patient performs various ADLs. The Physical Performance Test assesses upper body fine motor and coarse motor activities, as well as balance, mobility, coordination, andendurance. The Tinetti Gait and Balance Evaluation assesses gait and balance and provides information about fall risk 9. The nurse is assessing the forms of support an older patient has before she is discharged. Which of these examples is an informal source of support? A. The local senior center B. Her Medicare check C. Meals on Wheels meal delivery service D. Her neighbor, who visits with her daily Informal support includes family and close long-time friends and is usually provided free of charge. Another example of informal support is a neighbor who has daily contact with the client and shares food and company. Formal supports include programs such as social welfare and other social service and health care delivery agencies such as home health care. Semi formal supports such as church societies, neighborhood groups, and senior centers also form an important role in social support. 10. An 85-year-old man has been hospitalized after a fall at home, and his 86-year-old wife is at his bedside. She tells the nurse that she is his primary caregiver. The nurse should assess the caregiver for signs of possible caregiver burnout, such as: A. Depression. B. Weight gain. C. Hypertension. D. Social phobias Caregiver burden is the perceived strain by the person who cares for an elderly, chronically ill, or disabled person. Caregiver burnout is linked to the caregiver's ability to cope and handle stress. Signs of possible caregiver burnout include multiple somatic complaints, increased stress and anxiety, social isolation, depression, and weight loss. Screening caregivers for depression may also be appropriate. 11. During a morning assessment, the nurse notices that an older patient is less attentive and is unable to recall yesterday's events. Which test is appropriate for assessing the patient's mental status? A. Geriatric Depression Scale, Short Form B. The Physical Performance Test C. Mini-Cog D. The Get Up and Go Test For nurses in various settings, cognitive assessments provide continuing comparisons to the individual's baseline to detect any acute changes in mental status. The Mini-Cog is a mental status test that tests immediate and delayed recall and visuospatial ability. The Geriatric Depression Scale, Short Form assess for depression and changes in the level of depression, not mental status. The Physical Performance Test assesses activities such as eating, dressing, transferring, and stair climbing, but not mental status. The Get Up and Go Test assesses functional mobility, not mental status. 12. An elderly patient has been admitted to the intensive care unit (ICU) after falling at home. Within 8 hours, his condition has stabilized and he is transferred to a medical unit. The family is wondering whether he will be able to go back home. Which assessment instrument is most appropriate for the nurse to choose at this time? A. The Lawton IADL instrument B. Hospital Admission Risk Profile (HARP) C. The Mini-Cog D. The NEECHAM Confusion Scale - Hospital-acquired functional decline may occur within two days of a hospital admission. The HARP helps to identify older adults who are at greatest risk for loss of ADLs or mobility at this critical time. The Lawton IADL measures instrumental activities of daily living, which may be difficult to observe in the hospital setting. The Mini-Cog is an assessment of mental status. The NEECHAM Confusion Scale is used to assess for delirium. 13. During a functional assessment of an older person's home environment, which statement or question by the nurse is most appropriate regarding common environmental hazards? A. "These low toilet seats are safe because they are nearer to the ground in case of falls." B. "Do you have a relative or friend who can help to install grab bars in your shower?" C. "These small rugs are ideal for preventing you from slipping on the hard floor." D. "It would be safer to keep the lighting low in this room to avoid glare in your eyes." - Environmental hazards within the home can be a potential constraint on the older person's day-to-day functioning. Common environmental hazards including inadequate lighting, loose throw rugs, curled carpet edges, obstructed hallways, cords in walkways, lack of grab bars in tub and shower, and low and loose toilet seats are hazards that could lead to an increased risk of falls and fractures. Environmental modifications can promote mobility and reduce the likelihood of the older adult falling. 14. When beginning to assess a person's spirituality, which question by the nurse would be most appropriate? A. "Do you believe in God?" B. "How does your spirituality relate to your health care decisions?" C. "What religious faith do you follow?" D. "Do you believe in the power of prayer?" - Open-ended questions provide a foundation for future dialog. The other responses are easily answered by one-word replies, and they are closed questions. 15. The nurse is preparing to assess an older adult and discovers that the older adult is in severe pain.Which statement about pain and the older adult is true? A. Pain is inevitable with aging. B. Older adults with cognitive impairments feel less pain. C. Alleviating pain should be a priority over other aspects of the assessment. D. The assessment should take priority so that care decisions can be made. If the older adult is experiencing pain or discomfort, then the depth of knowledge gathered through the assessments will suffer. Alleviating pain should be a priority over other aspects of the assessment. It is paramount to remember that older adults with cognitive impairment do not feel less pain. 16. The nurse is assessing the abilities of an older adult. Which of these following activities are considered instrumental activities of daily living? Select all that apply. A. Feeding oneself B. Preparing a meal C. Balancing a checkbook D. Walking E. Toileting F. Grocery shopping - , C, F Typically, instrumental activities of daily living tasks include shopping, meal preparation, housekeeping, laundry, managing finances, taking medications, and using transportation. The others listed are activities of daily living related to self-care. An 85-year-old man has come in for a physical examination, and the nurse notices that he uses a cane. When documenting general appearance, the nurse should document this information under the section that covers: A) posture. B) mobility. C) mood and affect. D) physical deformity. - Use of assistive devices would be documented under the mobility section. The other responses are all other categories of the general appearance section of the health history. Page: 764 The nurse is performing a vision examination. Which of these charts is most widely used for vision examinations? A) Snellen B) Shetllen C) Smoollen D) Schwellon The Snellen eye chart is most widely used for vision examinations. The other options are not tests for vision examinations. Page: 764 After the health history has been obtained, and before beginning the physical examination, the nurse should ask the patient to first: A) empty the bladder. B) completely disrobe. C) lie on the examination table. D) walk around the room. Before beginning the examination, the nurse should ask the person to empty the bladder (save the specimen if needed), disrobe except for underpants, put on a gown, and sit with legs dangling off side of the bed or table. Page: 764 During a complete health assessment, how would the nurse test the patient's hearing? A) By observing how the patient participates in normal conversation B) Using the whispered voice test C) Using the Weber and Rinne tests D) Testing with an audiometer - During the complete health assessment, the nurse should test hearing with the whispered voice test. The other options are not correct. Page: 765 A patient states, "Whenever I open my mouth real wide, I feel this popping sensation in front of my ears." To further examine this, the nurse would: A) place the stethoscope over the temporomandibular joint and listen for bruits. B) place the hands over his ears and ask him to open his mouth "really wide." C) place one hand on his forehead and the other on his jaw and ask him to try to open his mouth. D) place a finger on his temporomandibular joint and ask him to open and close his mouth. The nurse should palpate the temporomandibular joint by placing your fingers over it as the person opens and closes the mouth. Page: 765 The nurse has just completed an examination of a patient's extraocular muscles. When documenting the findings, the nurse should document the assessment of which cranial nerves? A) II, III, VI B) II, IV, V C) III, IV, V D) III, IV, VI Extraocular muscles are innervated by cranial nerves III, IV, and VI. Page: 765 A patient's uvula rises midline when she says "ahh" and she has a positive gag reflex. The nurse has just tested which cranial nerves? A) IX, X B) IX, XII C) X, XII D) XI, XII Cranial nerves IX and X are being tested by having the patient say "ahh," noting the mobility of the uvula, and when assessing the patient's gag reflex. Page: 766 During an examination, the nurse notices that a patient is unable to stick out his tongue. Which cranial nerve is involved with successful performance of this action? A) I B) V C) XI D) XII Cranial nerve XII enables the person to stick out his or her tongue. Page: 766 A patient is unable to shrug her shoulders against the nurse's resistant hands. What cranial nerve is involved with successful shoulder shrugging? A) VII B) IX C) XI D) XII Cranial nerve XI enables the patient to shrug her shoulders against resistance. Page: 766 During an examination, a patient has just successfully completed the finger-to-nose and the rapid-alternating-movements tests and is able to run each heel down the opposite shin. The nurse will conclude that the patient's ____ function is intact. A) occipital B) cerebral C) temporal D) cerebellar The nurse should test cerebellar function of the upper extremities by using the finger-to-nose test or rapid-alternating-movements test. The nurse should test cerebellar function of the lower extremities by asking the person to run each heel down the opposite shin. Pages: 769-770 A 5-year old child is in the clinic for a checkup. The nurse would expect him to: A) have to be held on his mother's lap. B) be able to sit on the examination table. C) be able to stand on the floor for the examination. D) be able to remain alone in the examination room - At 4 or 5 years old, a child usually feels comfortable on the examination table. Older infants and young children aged 6 months to 2 or 3 years should be positioned in the parent's lap. Pages: 777-778 When the nurse performs the confrontation test, the nurse has assessed: A) extraocular eye muscles (EOMs). B) pupils (PERRLA). C) near vision. D) visual fields. The confrontation test assesses visual fields. The other options are not tested with the confrontation test. Page: 765 Which of these statements is true regarding the complete physical assessment? A) The male genitalia should be examined in the supine position. B) The patient should be in the sitting position for examination of the head and neck. C) The vital signs, height, and weight should be obtained at the end of the examination. D) To promote consistency between patients, the examiner should not vary the order of the assessment. - The head and neck should be examined in the sitting position to best palpate the thyroid and lymph nodes. The male patient should stand during examination of the genitalia. Vital signs are measured early in the assessment. The sequence of the assessment may need to vary according to different patient situations. Page: 764 Which of these statements is true regarding the recording of data from the history and physical examination? A) Use long, descriptive sentences to document findings. B) Record the data as soon as possible after the interview and physical examination. C) If the information is not documented, then it can be assumed that it was done as a standard of care. D) The examiner should avoid taking any notes during the history and examination because of the possibility of decreasing rapport with the patient. - The data from the history and physical examination should be recorded as soon after the event as possible. From a legal perspective, if it is not documented, it was not done. Brief notes should be taken during the examination. When documenting, the nurse should use short clear phrases and should avoid redundant phrases and descriptions. Page: 781 Which of these is included in assessment of general appearance? A) Height B) Weight C) Skin color D) Vital signs General appearance includes items such as level of consciousness, skin color, nutritional status, posture, mobility, facial expression, mood and affect, speech, hearing, and personal hygiene. Height, weight, and vital signs are considered measurements. Page: 764 The nurse should wear gloves for which of these examinations? A) Measuring vital signs B) Palpation of the sinuses C) Palpation of the mouth and tongue D) Inspection of the eye with an ophthalmoscope Gloves should be worn when the examiner is exposed to the patient's body fluids. Page: 766 The nurse should use which location for eliciting deep tendon reflexes? A) Achilles B) Femoral C) Scapular D) Abdominal Deep tendon reflexes are elicited in the biceps, triceps, brachioradialis, patella, and Achilles. Pages: 769-770 During inspection of a patient's face, the nurse notices that the facial features are symmetric. This finding indicates that which cranial nerve is intact? A) VII B) IX C) XI D) XII Cranial nerve VII is responsible for facial symmetry. Page: 766 During inspection of the posterior chest, the nurse should assess for: A) symmetric expansion. B) symmetry of shoulders and muscles. C) tactile fremitus. D) diaphragmatic excursion. - During inspection of the posterior chest, the nurse should inspect for symmetry of shoulders and muscles, configuration of the thoracic cage, and skin characteristics. Symmetric expansion and tactile fremitus are assessed with palpation; diaphragmatic excursion is assessed with percussion. Page: 766 When assessing the neonate, the nurse should test for hip stability with which method? A) Eliciting the Moro reflex B) Performing the Romberg's test C) Checking for the Ortolani's sign D) Assessing the stepping reflex The nurse should test for hip stability in the neonate by testing for the Ortolani's sign. The other tests are not appropriate for testing hip stability. Pages: 775-776 A female patient tells the nurse that she has four children and has had three pregnancies. How should the nurse document this? A) Gravida 3, para 4 B) Gravida 4, para 3 C) This information cannot be documented using the terms gravida and para. D) "The patient seems to be confused about how many times she has been pregnant." Gravida refers to the number of pregnancies, and para refers to the number of children. One pregnancy was with twins. Page: 781 During an examination, the patient tells the nurse that she sometimes feels as if objects are spinning around her. The nurse would document that she occasionally experiences: A) vertigo. B) tinnitus. C) syncope. D) dizziness. Vertigo is the sensation of moving around in space (subjective) or of having objects move about the person (objective) and is a result of a disturbance of equilibratory apparatus. See Chapter 23. Pages: 621-678 A patient tells the nurse that "sometimes I wake up at night and I have real trouble breathing. I have to sit up in bed to get a good breath." When documenting this information, the nurse would note: A) orthopnea. B) acute emphysema. C) paroxysmal nocturnal dyspnea. D) acute shortness of breath episode. Paroxysmal nocturnal dyspnea occurs when the patient awakens from sleep with shortness of breath and needs to be upright to achieve comfort. See Chapter 18. Pages: 411-454 During the examination of a patient, the nurse notices that the patient has several small, flat macules on the posterior portion of her thorax. These macules are less than 1 cm wide. Another name for these macules is: A) warts. B) bullae. C) freckles. D) papules. A macule is solely a lesion with color change, flat and circumscribed, less than 1 cm. Macules are also known as freckles. See Chapter 12. Pages: 203-250 During an examination, the nurse notices that a patient's legs turn white when they are raised above the patient's head. The nurse should suspect: A) lymphedema. B) Raynaud's disease. C) chronic arterial insufficiency. D) chronic venous insufficiency. Elevational pallor (marked) indicates arterial insufficiency. See Chapter 20. Pages: 499-525 The nurse documents that a patient has coarse, thickened skin and brown discoloration over the lower legs. Pulses are present. This finding is probably the result of: A) lymphedema. B) Raynaud's disease. C) chronic arterial insufficiency. D) chronic venous insufficiency. Chronic venous insufficiency would present as firm brawny edema, coarse thickened skin, normal pulses, and brown discoloration. See Chapter 20. Pages: 499-525 The nurse notices that a patient has ulcerations on the tips of the toes and on the lateral aspect of the ankles. This finding indicates: A) lymphedema. B) Raynaud's disease. C) arterial insufficiency. D) venous insufficiency. Ulcerations on the tips of the toes and lateral aspect of the ankles are indicative of arterial insufficiency. See Chapter 20. Pages: 499-525 When the nurse flexes the patient's knee and gently compresses the gastrocnemius muscle anteriorly against the tibia, the patient indicates that he is having calf pain. The nurse should document _____ sign. A) positive Allen's B) negative Allen's C) positive Homans' D) negative Homans' Calf pain with these maneuvers is a positive Homans' sign, which occurs in some cases of deep vein thrombosis. See Chapter 20. Pages: 499-525 The nurse has just recorded a positive obturator test on a patient who has abdominal pain. This test is used to confirm a(n): A) inflamed liver. B) perforated spleen. C) perforated appendix. D) enlarged gallbladder. A perforated appendix irritates the obturator muscle, producing pain. See Chapter 21. Pages: 527-564 The nurse is documenting the assessment of an infant. During the abdominal assessment, the nurse noticed a very loud splash auscultated over the upper abdomen when the nurse rocked her from side to side. This finding would indicate: A) epigastric hernia. B) pyloric obstruction. C) hypoactive bowel sounds. D) hyperactive bowel sounds. A succussion splash, which is unrelated to peristalsis, is a very loud splash auscultated over the upper abdomen when the infant is rocked side to side. It indicates increased air and fluid in the stomach as seen with pyloric obstruction or large hiatus hernia. See Chapter 21. Pages: 527-564 The nurse will measure a patient's near vision with which tool? A) Snellen eye chart with letters B) Snellen "E" chart C) Jaeger card D) Ophthalmoscope The Jaeger card is used to measure near vision. See Chapter 14. Pages: 279-322 If the nurse records the results to the Hirschberg test, the nurse has: A) tested the patellar reflex. B) assessed for appendicitis. C) tested the corneal light reflex. D) assessed for thrombophlebitis. The Hirschberg test assesses the corneal light reflex. See Chapter 14. Pages: 279-322 During the examination of a patient's mouth, the nurse observes a nodular bony ridge down the middle of the hard palate. The nurse would chart this finding as: A) cheilosis. B) leukoplakia. C) ankyloglossia. D) torus palatinus. A normal variation of the hard palate is a nodular bony ridge down the middle of the hard palate, a torus palatinus. See Chapter 16. Pages: 351-382 During examination, the nurse finds that a patient is unable to distinguish objects placed in his hand. The nurse would document: A) stereognosis. B) astereognosis. C) graphesthesia. D) agraphesthesia. - Astereognosis is the inability to identify correctly an object placed in the hand. See Chapter 23. Pages: 621-678 After the examination of an infant, the nurse documents opisthotonos. The nurse recognizes that this finding often occurs with: A) cerebral palsy. B) meningeal irritation. C) a lower motor neuron lesion. D) a upper motor neuron lesion. - Opisthotonos is a form of spasm in which the head is arched back, and there is stiffness of the neck and extension of the arms and legs. It occurs with meningeal or brainstem irritation. See Chapter 23 Pages: 621-678 After assessing a female patient, the nurse notices flesh-colored, soft, pointed, moist, papules in a cauliflower-like patch around her introitus. This finding is most likely: A) urethral caruncle. B) syphilitic chancre. C) herpes. D) human papillomavirus. Human papillomavirus appears in a flesh-colored, soft, moist, cauliflower-like patch of papules. See Chapter 26. Pages: 725-762 While recording in a patient's medical record, the nurse notices that a patient's Hematest results are positive. This means that there: A) are crystals in his urine. B) are parasites in his stool. C) is occult blood in his stool. D) are bacteria in his sputum. If a stool is Hematest positive, then it indicates the presence of occult blood. See Chapter 21. Pages: 527-564 While examining a 48-year-old patient's eyes, the nurse notices that he had to move the handheld vision screener farther away from his face. The nurse would suspect: A) myopia. B) omniopia. C) hyperopia. D) presbyopia. Presbyopia, the decrease in power of accommodation with aging, is suggested when the handheld vision screener card is moved farther away. See Chapter 14. Pages: 279-322 Which of these actions is most appropriate to perform on a 9-month-old infant at a well-child checkup? A) Testing for Ortolani's sign B) Assessment for stereognosis C) Blood pressure measurement D) Assessment for the presence of the startle reflex Until the age of 12 months, the infant should be assessed for Ortolani's sign. If Ortolani's sign is present, it could indicate the presence of a dislocated hip. The other tests are not appropriate for a 9-month-old child. Pages: 565-620 For the abdominal assessment, place these assessment techniques in the correct order, with A being performed first and E being performed last. A. Deep palpation, all quadrants B. Light palpation, all quadrants C Auscultate bowel sounds D. Inspect abdomen for contour, skin characteristics, and pulsations E. Percuss all quadrants D, C, E, B, A After inspection, first perform auscultation of bowel sounds so that the sounds are not altered by percussion and palpation. Follow auscultation by percussion, then light palpation, then deep palpation. See Chapter 21. Pages: 527-564 Which of these statements about the anal canal is true? The anal canal: A) is about 2 cm long in the adult. B) slants backward toward the sacrum. C) contains hair and sebaceous glands. D) is the outlet for the gastrointestinal tract. The anal canal is the outlet for the gastrointestinal tract, and it is about 3.8 cm long in the adult. It is lined with a modified skin that does not contain hair or sebaceous glands, and it slants forward toward the umbilicus. Which of these statements about the sphincters is correct? A) The internal sphincter is under voluntary control. B) The external sphincter is under voluntary control. C) Both sphincters remain slightly relaxed at all times. D) The internal sphincter surrounds the external sphincter. - The external sphincter surrounds the internal sphincter but also has a small section overriding the tip of the internal sphincter at the opening. The external sphincter is under voluntary control. Except for the passing of feces and gas, the sphincters keep the anal canal tightly closed. The nurse is performing an examination of the anus and rectum. Which of these statements is correct and important to remember during this examination? A) The rectum is about 8 cm long. B) The anorectal junction cannot be palpated. C) Above the anal canal, the rectum turns anteriorly. D) There are no sensory nerves in the anal canal or rectum. - The anal columns are folds of mucosa that extend vertically down from the rectum and end in the anorectal junction. This junction is not palpable, but it is visible on proctoscopy. The rectum is 12 cm long, and just above the anal canal, the rectum dilates and turns posteriorly. The structure that secretes a thin, milky alkaline fluid to enhance the viability of sperm is the: A) Cowper's gland. B) prostate gland. C) median sulcus. D) bulbourethral gland. - In men, the prostate gland secretes a thin milky alkaline fluid that enhances sperm viability. The Cowper's glands (also known as bulbourethral glands) secrete a clear, viscid mucus. The median sulcus is a groove dividing the lobes of the prostate gland and does not secrete fluid. A 46-year-old man requires assessment of his sigmoid colon. The nurse is aware that which of these is most appropriate for this examination? A) Proctoscope B) Ultrasound C) Colonoscope D) Rectal exam with an examining finger The sigmoid colon is 40 cm long and is accessible to examination only with the colonoscope. The other responses are not appropriate for examination of the entire sigmoid colon. The nurse is caring for a newborn infant. Thirty hours after birth, the infant passes a dark green meconium stool. The nurse recognizes that this is important because: A) this stool would indicate anal patency. B) the dark green color could indicate occult blood in the stool. C) meconium stool can be reflective of distress in the newborn. D) the newborn should have passed the first stool within 12 hours after birth. The first stool passed by the newborn is dark green meconium and occurs within 24 to 48 hours of birth, indicating anal patency. The other responses are not correct During the assessment of an 18-month-old child, the mother expresses concern to the nurse about the infant's inability to toilet train. What would be the nurse's best response? A) "Some children are just more difficult to train, so I wouldn't worry about it yet." B) "Have you considered reading any of the books on toilet training? They can be very helpful." C) "This could mean there is a problem in your baby's development. We'll watch her closely for the next few months." D) "The nerves that will allow your baby to have control over the passing of stools are not developed until at least 18 to 24 months of age." The infant passes stools by reflex. Voluntary control of the external anal sphincter cannot occur until the nerves supplying the area have become fully myelinated, usually around 1 1/2 to 2 years of age. Toilet training usually starts after the age of 2. A 60-year-old man has just been told he has benign prostatic hypertrophy. He has a friend who just died from cancer of the prostate, and he is concerned this will happen to him. How should the nurse respond? A) "The swelling in your prostate is only temporary and will go away." B) "We will treat you with chemotherapy so we can control the cancer." C) "It would be very unusual for a man your age to have cancer of the prostate." D) "The enlargement of your prostate is caused by hormone changes and not cancer." The prostate gland commonly starts to enlarge during the middle adult years. This benign prostatic hypertrophy (BPH) is present in 1 out of 10 males at the age of 40 years and increases with age. It is thought that the hypertrophy is caused by hormonal imbalance that leads to the proliferation of benign adenomas. The other responses are not appropriate. A 30-year-old woman is visiting the clinic because of "pain in my bottom when I have a bowel movement." The nurse should assess for which problem? A) Pinworms B) Hemorrhoids C) Colon cancer D) Fecal incontinence - Having painful bowel movements, known as dyschezia, may be due to a local condition (hemorrhoid or fissure) or constipation. The other responses are not correct. A patient who is visiting the clinic complains of having "stomach pains for 2 weeks" and describes his stools as being "soft and black" for about the last 10 days. He denies taking any medications. The nurse is aware that these symptoms are most indicative of: A) excessive fat caused by malabsorption. B) increased iron intake resulting from a change in diet. C) occult blood resulting from gastrointestinal bleeding. D) absent bile pigment from liver problems. Black stools may be tarry due to occult blood (melena) from gastrointestinal bleeding or nontarry from ingestion of iron medications (not diet). Excessive fat causes the stool to become frothy; absence of bile pigment causes clay-colored stools. After completing an assessment of a 60-year-old man with a family history of colon cancer, the nurse discusses with him early detection measures for colon cancer. The nurse should mention the need for a(n): A) annual proctoscopy. B) colonoscopy every 10 years. C) fecal test for blood every 6 months. D) digital rectal examinations every 2 years. - Early detection measures for colon cancer include a digital rectal examination performed annually after age 50 years, a fecal occult blood test annually after age 50 years, sigmoidoscopy every 5 years or colonoscopy every 10 years after age 50 years; and a PSA blood test annually for men over 50 years old, except black men beginning at age 45 years (American Cancer Society, 2006). The mother of a 5-year-old girl tells the nurse that she has noticed her daughter "scratching at her bottom a lot the last few days." During the assessment, the nurse finds redness and raised skin in the anal area. This most likely indicates: A) pinworms. B) chickenpox. C) constipation. D) bacterial infection. In children, pinworms are a common cause of intense itching and irritated anal skin. The other options are not correct. The nurse is examining only the rectal area of a woman and should place the woman in what position? A) Lithotomy position B) Prone position C) Left lateral decubitus position D) Bending over the table while standing The nurse should place the female patient in lithotomy position if examining genitalia as well; use the left lateral decubitus position for the rectal area alone. While doing an assessment of the perianal area of a patient, the nurse notices that the pigmentation of anus is darker than surrounding skin, the anal opening is closed, and there is a skin sac that is shiny and blue. The patient mentioned that he has had pain with bowel movements and has noted some spots of blood occasionally. What would this assessment and history be most likely to indicate? A) Anal fistula B) Pilonidal cyst C) Rectal prolapse D) Thrombosed hemorrhoid The anus normally looks moist and hairless, with coarse folded skin that is more pigmented than the perianal skin. The anal opening is tightly closed. The shiny blue skin sac indicates a thrombosed hemorrhoid. The nurse is preparing to palpate the rectum and should use which of these techniques? A) Flex the finger and insert slowly toward the umbilicus. B) Instruct the patient first that this will be a painful procedure. C) Insert an extended index finger at a right angle to the anus. D) Place the finger directly into the anus to overcome the tight sphincter. The nurse should place the pad of the index finger gently against the anal verge. The nurse will feel the sphincter tighten and then relax. As it relaxes, the nurse should flex the tip of the finger and slowly insert it into the anal canal in a direction toward the umbilicus. The nurse should never approach the anus at right angles with the index finger extended—this would cause pain. The nurse should instruct the patient that palpation is not painful but may feel like needing to move the bowels. While performing a rectal examination, the nurse notices a firm, irregularly shaped mass. What should the nurse do next? A) Continue with the examination and document the finding in the chart. B) Instruct patient to return for repeat assessment in 1 month. C) Tell the patient that a mass was felt but it is nothing to worry about. D) Report the finding and refer the patient to a specialist for further examination. A firm or hard mass with irregular shape or rolled edges may signify carcinoma. Promptly report any mass that is discovered for further examination. The other responses are not correct When testing stool for occult blood, the nurse is aware that a false-positive result may occur with: A) absent bile pigment. B) increased fat content. C) increased ingestion of iron medication. D) a large amount of red meat within the last 3 days. When testing for occult blood, a false-positive finding may occur if the person has ingested significant amounts of red meat within 3 days of the test. Absent bile pigment causes the stools to be gray or tan in color. Increased fat content causes the stool to be pale, yellow, and greasy. Increased ingestion of iron medication causes the stool to be black in color. During an assessment of the newborn, the nurse expects to see which finding when the anal area is slightly stroked? A) A jerking of the legs B) Flexion of the knees C) A quick contraction of the sphincter D) Relaxation of the external sphincter To assess sphincter tone, the nurse should check the anal reflex by gently stroking the anal area and noticing a quick contraction of the sphincter. The other responses are not correct. A 13-year-old girl is visiting the clinic for a sports physical. The nurse should remember to include which of these tests in the examination? A) Test for occult blood B) The Valsalva maneuver C) Internal palpation of the anus D) Inspection of the perianal area Inspect the perianal region of the school-aged child and adolescent during examination of the genitalia. Internal palpation is not performed routinely at this age. Testing for occult blood and doing the Valsalva maneuver are also not necessary. During an assessment of a 20-year-old man, the nurse finds a small palpable lesion with a tuft of hair located directly over the coccyx. The nurse knows that this lesion would most likely be a: A) polyp. B) pruritus ani. C) carcinoma. D) pilonidal cyst. A pilonidal cyst or sinus is a hair-containing cyst or sinus located in the midline over the coccyx or lower sacrum. It often opens as a dimple with a visible tuft of hair and, possibly, an erythematous halo. See Table 25-1 for more information, and also for description of pruritus ani. See Table 25-2 for descriptions of rectal polyps and carcinoma. During an examination, the nurse asks the patient to perform the Valsalva maneuver and notices that the patient has a moist, red, doughnut-shaped protrusion from the anus. The nurse knows that this would be consistent with: A) a rectal polyp. B) hemorrhoids. C) a rectal fissure. D) rectal prolapse. In rectal prolapse, the rectal mucous membrane protrudes through the anus, appearing as a moist red doughnut with radiating lines. It occurs following a Valsalva maneuver, such as straining at stool, or with exercise. See Table 25-1. For a description of rectal polyps, see Table 25-2. See Table 25-1 for descriptions of rectal fissure and hemorrhoids. A 70-year-old man is visiting the clinic for difficulty in passing urine. In the history he indicates he has to urinate frequently, especially at night. He has burning when he urinates and has noticed pain in his back. Given this history, what might the nurse expect to find during the physical assessment? A) Asymmetric, hard, fixed prostate gland B) Occult blood and perianal pain to palpation C) Symmetrically enlarged, soft prostate gland D) A soft nodule protruding from rectal mucosa Subjective symptoms of carcinoma of the prostate include frequency, nocturia, hematuria, weak stream, hesitancy, pain or burning on urination, and continuous pain in lower back, pelvis, and thighs. Objective symptoms of carcinoma of the prostate include a malignant neoplasm often starts as a single hard nodule on the posterior surface, producing asymmetry and a change in consistency. As it invades normal tissue, multiple hard nodules appear, or the entire gland feels stone hard and fixed. A 40-year-old black man is in the office for his annual physical. Which statement regarding the prostate-specific antigen (PSA) blood test is true, according to the American Cancer Society? The PSA: A) should be done with this visit. B) should be done at age 45 years. C) should be done at age 50 years. D) is only necessary if there is a family history of prostate cancer. - According to the American Cancer Society (2006) the PSA blood test should be done annually for black men beginning at age 45 years, and annually for all other men over age 50 years. A 62-year-old man is experiencing fever, chills, malaise, urinary frequency, and urgency. He also reports urethral discharge and a dull aching pain in the perineal and rectal area. These symptoms are most consistent with which of the following? A) Prostatitis B) A polyp C) Carcinoma of the prostate D) Benign prostatic hypertrophy (BPH) The common presenting symptoms of prostatitis are fever, chills, malaise, and urinary frequency and urgency. The individual may also have dysuria, urethral discharge, and a dull aching pain in the perineal and rectal area. See Table 25-3 for descriptions of carcinoma of the prostate and BPH. These are not the symptoms of a polyp. During a discussion for a men's health group, the nurse relates that the group with the highest incidence of prostate cancer is: A) Asian Americans. B) African-Americans. C) American Indians. D) Hispanics. - According to the American Cancer Society (2010), African-American men have a higher rate of prostate cancer than other racial groups. The nurse is palpating the prostate gland through the rectum and notices an abnormal finding if which of these is present? A) Palpable central groove B) Tenderness to palpation C) Heart shape D) Elastic and rubbery consistency - The normal prostate gland should feel smooth, elastic, and rubbery; should be slightly movable; should be heart-shaped with a palpable central groove; and should not be tender to palpation. The nurse notices that a patient has had a pale, yellow, greasy stool, or steatorrhea, and recalls that this is caused by: A) occult bleeding. B) absent bile pigment. C) increased fat content. D) ingestion of bismuth preparations. Steatorrhea (pale, yellow, greasy stool) is caused by increased fat content in the stools, as in malabsorption syndrome. Occult bleeding and ingestion of bismuth products cause black stool, and absent bile pigment causes gray, tan stool. During a health history of a patient who complains of chronic constipation, the patient asks the nurse about high-fiber foods. The nurse relates that an example of a high-fiber food would be: A) broccoli. B) hamburger. C) iceberg lettuce. D) yogurt. High-fiber foods are either soluble type (i.e., beans, prunes, barley, broccoli) and insoluble type (i.e., cereals, wheat germ). The other examples are not considered high-fiber foods. While assessing a hospitalized, bedridden patient, the nurse notices that the patient has been incontinent of stool. The stool is loose and gray-tan in color. The nurse recognizes that this finding indicates which of the following? A) Occult blood B) Inflammation C) Absent bile pigment D) Ingestion of iron preparations The presence of gray, tan stool indicates absent bile pigment, which can occur with obstructive jaundice. Ingestion of iron preparations and presence of occult blood would turn the stools to a black color. Jelly-like mucus shreds mixed in the stool would indicate inflammation. During a digital examination of the rectum, the nurse notices that the patient has hard feces in the rectum. The patient complains of feeling "full," has a distended abdomen, and states that she has not had a bowel movement "for several days." The nurse suspects which condition? A) Rectal polyp B) Fecal impaction C) Rectal abscess D) Rectal prolapse - A fecal impaction is a collection of hard, desiccated feces in the rectum. The obstruction often results from decreased bowel motility, in which more water is reabsorbed from the stool. See Table 25-2 for descriptions of rectal polyp and abscess; See Table 25-1 for description of rectal prolapse During a history, the patient states, "It really hurts back there, and sometimes it itches, too. I have even seen blood on the tissue when I have a bowel movement. Is there something there?" The nurse should expect to see which of these upon examination of the anus? A) Rectal prolapse B) Internal hemorrhoid C) External hemorrhoid that has resolved D) External hemorrhoid that is thrombosed These symptoms are consistent with an external hemorrhoid. An external hemorrhoid, when thrombosed, contains clotted blood and becomes a painful, swollen, shiny blue mass that itches and bleeds with defecation. When the external hemorrhoid resolves, it leaves a flabby, painless skin sac around the anal orifice. An internal hemorrhoid is not palpable, but may appear as a red mucosal mass when the person performs a Valsalva maneuver. A rectal prolapse appears as a moist, red doughnut with radiating lines The nurse is performing a digital examination of a patient's prostate gland and notices that characteristics of a normal prostate gland include which of the following? Select all that apply. A) The gland protruding 1 cm into the rectum B) Heart-shaped with a palpable central groove C) Flat with no groove palpable D) Boggy and soft consistency E) Smooth surface, elastic, or rubbery consistency F) Fixed mobility , B, E The size should be 2.5 cm long by 4 cm wide, and it should not protrude more than 1 cm into the rectum. The prostate should be heart-shaped, with a palpable central groove, a smooth surface, and elastic, rubbery consistency. Abnormal findings include a flat shape with no palpable groove, boggy with a soft consistency, and fixed mobility. During a health history, a 22-year old woman asks, "Can I get that vaccine for HPV? I have genital warts and I'd like them to go away!" What is the nurse's best response? A) "The HPV vaccine is for girls and women ages 9 to 26, so we can start that today." B) "This vaccine is only for girls who have not started to have intercourse yet." C) "Let's check with the physician to see if you are a candidate for this vaccine." D) "The vaccine cannot protect you if you already have an HPV infection." The HPV (human papillomavirus) vaccine is appropriate for girls and women age 9 to 26 and is given to prevent cervical cancer by preventing HPV infections before girls become sexually active. However, it cannot protect the woman if an HPV infection is already present. During an examination the nurse observes a female patient's vestibule and expects to see the: A) urethral meatus and vaginal orifice. B) vaginal orifice and vestibular (Bartholin) glands. C) urethral meatus and paraurethral (Skene) glands. D) paraurethral (Skene) and vestibular (Bartholin) glands. The labial structures encircle a boat-shaped space, or cleft, termed the vestibule. Within it are numerous openings. The urethral meatus and vaginal orifice are visible. The ducts of the paraurethral (Skene) glands and the vestibular (Bartholin) glands are present but not visible. During a speculum inspection of the vagina, the nurse would expect to see what at the end of the vaginal canal? A) Cervix B) Uterus C) Ovaries D) Fallopian tubes At the end of the canal, the uterine cervix projects into the vagina The uterus is usually positioned tilting forward and superior to the bladder. This position is known as: A) anteverted and anteflexed. B) retroverted and anteflexed. C) retroverted and retroflexed. D) superiorverted and anteflexed. The uterus is freely movable, not fixed, and usually tilts forward and superior to the bladder (a position labeled as anteverted and anteflexed). An 11-year-old girl is in the clinic for a sports physical. The nurse notices that she has begun to develop breasts, and during the conversation the girl reveals that she is worried about her development. The nurse should use which of these techniques to best assist the young girl in understanding the expected sequence for development? The nurse should: A) use the Tanner's table on the five stages of sexual development. B) describe her development and compare it with that of other girls her age. C) use Jacobsen's table on expected development on the basis of height and weight data. D) reassure her that her development is within normal limits and should tell her not to worry about the next step. Tanner's table on the five stages of pubic hair development is helpful in teaching girls the expected sequence of sexual development (see Table 26-1). The other responses are not appropriate. A woman who is 8 weeks pregnant is in the clinic for a checkup. The nurse reads on her chart that her cervix is softened and looks cyanotic. The nurse knows that the woman is exhibiting _____ sign and _____ sign. A) Tanner's; Hegar's B) Hegar's; Goodell's C) Chadwick's; Hegar's D) Goodell's; Chadwick's Shortly after the first missed menstrual period, the female genitalia show signs of the growing fetus. The cervix softens (Goodell's sign) at 4 to 6 weeks, and the vaginal mucosa and cervix look cyanotic (Chadwick's sign) at 8 to 12 weeks. These changes occur because of increased vascularity and edema of the cervix and hypertrophy and hyperplasia of the cervical glands. Hegar's sign occurs when the isthmus of the uterus softens at 6 to 8 weeks. Tanner's sign is not a correct response. A woman who is 22 weeks pregnant has a vaginal infection. She tells the nurse that she is afraid that this infection will hurt the fetus. The nurse knows that which of these statements is true? A) If intercourse is avoided, then the risk for infection is minimal. B) A thick mucus plug forms that protects the fetus from infection. C) The acidic pH of vaginal secretions promotes the growth of pathogenic bacteria. D) The mucus plug that forms in the cervical canal is a good medium for bacterial growth. - A clot of thick, tenacious mucus forms in the spaces of the cervical canal (the mucus plug), which protects the fetus from infection. Cervical and vaginal secretions increase during pregnancy and are thick, white, and more acidic. The acidic pH keeps pathogenic bacteria from multiplying in the vagina, but the increase in glycogen increases the risk of candidiasis (commonly called a yeast infection) during pregnancy. The changes normally associated with menopause occur generally because the cells in the reproductive tract are: A) aging. B) becoming fibrous. C) estrogen dependent. D) able to respond to estrogen. Because cells in the reproductive tract are estrogen dependent, decreased estrogen levels during menopause bring dramatic physical changes. The other options are not correct. The nurse is reviewing the changes that occur with menopause. Which of these are changes associated with menopause? A) Uterine and ovarian atrophy along with thinning vaginal epithelium B) Ovarian atrophy, increased vaginal secretions, and increasing clitoral size C) Cervical hypertrophy, ovarian atrophy, and increased acidity of vaginal secretions D) Vaginal mucosa fragility, increased acidity of vaginal secretions, and uterine hypertrophy The uterus shrinks because of its decreased myometrium. The ovaries atrophy to 1 to 2 cm and are not palpable after menopause. The sacral ligaments relax, and the pelvic musculature weakens, so the uterus droops. The cervix shrinks and looks paler with a thick glistening epithelium. The vaginal epithelium atrophies, becoming thinner, drier, and itchy. The vaginal pH becomes more alkaline, and secretions are decreased. This results in a fragile mucosal surface that is at risk for bleeding and vaginitis. A 54-year-old woman who has just completed menopause is in the clinic today for a yearly physical examination. Which of these statements should the nurse include in patient education? "A postmenopausal woman: A) is not at any greater risk for heart disease than a younger woman is." B) should be aware that she is at increased risk for dyspareunia because of decreased vaginal secretions." C) has only stopped menstruating; there really are no other significant changes with which she should be concerned." D) is likely to have difficulty with sexual pleasure as a result of drastic changes in the female sexual response cycle." - Decreased vaginal secretions leave the vagina dry and at risk for irritation and pain with intercourse (dyspareunia). The other statements are incorrect. A woman is in the clinic for an annual gynecologic examination. The nurse should plan to begin the interview with the: A) menstrual history because it is generally nonthreatening. B) obstetric history because it is the most important information. C) urinary system history because there may be problems in this area as well. D) sexual history because it will build rapport to discuss this first. Menstrual history is usually nonthreatening; thus it is a good place to start. Obstetric, urinary, and sexual histories are also part of the interview but not necessarily the best topics with which to start. A patient has had three pregnancies and two live births. The nurse would record this information as gravida _____, para _____, AB _____. A) 2; 2; 1 B) 3; 2; 0 C) 3; 2; 1 D) 3; 3; 1 Gravida is number of pregnancies. Para is number of births. Abortions are interrupted pregnancies, including elective abortions and spontaneous miscarriages. During the interview with a female patient, the nurse gathers data that indicate that the patient is perimenopausal. Which of these statements made by this patient leads to this conclusion? A) "I have noticed that my muscles ache at night when I go to bed." B) "I will be very happy when I can stop worrying about having a period." C) "I have been noticing that I sweat a lot more than I used to, especially at night." D) "I have only been pregnant twice, but both times I had breast tenderness as my first symptom." Hormone shifts occur during the perimenopausal period, and associated symptoms of menopause may occur, such as hot flashes, night sweats, numbness and tingling, headache, palpitations, drenching sweats, mood swings, vaginal dryness, and itching. The other responses are not correct. A 50-year-old woman calls the clinic because she has noticed some changes in her body and breasts and wonders if they could be due to the hormone replacement therapy (HRT) she started 3 months ago. The nurse should tell her: A) "Hormone replacement therapy is at such a low dose that side effects are very unusual." B) "Hormone replacement therapy has several side effects, including fluid retention, breast tenderness, and vaginal bleeding." C) "It would be very unusual to have vaginal bleeding with hormone replacement therapy, and I suggest you come in to the clinic immediately to have this evaluated." D) "It sounds as if your dose of estrogen is too high; I think you may need to decrease the amount you are taking and then call back in a week." - Side effects of hormone replacement therapy include fluid retention, breast pain, and vaginal bleeding. The other responses are not correct. A 52-year-old patient states that when she sneezes or coughs she "wets herself a little." She is very concerned that something may be wrong with her. The nurse suspects that the problem is: A) dysuria. B) stress incontinence. C) hematuria. D) urge incontinence. - Stress incontinence is involuntary urine loss with physical strain, sneezing, or coughing. Dysuria is pain or burning with urination. Hematuria is bleeding with urination. Urge incontinence is involuntary urine loss but it occurs due to an overactive detrusor muscle in the bladder that contracts and causes an urgent need to void During the interview a patient reveals that she has some vaginal discharge. She is worried that it may be a sexually transmitted infection. The nurse's most appropriate response to this would be: A) "Oh, don't worry. Some cyclic vaginal discharge is normal." B) "Have you been engaging in unprotected sexual intercourse?" C) "I'd like some information about the discharge. What color is it?" D) "Have you had any urinary incontinence associated with the discharge?" Ask questions that help the patient reveal more information about her symptoms in a nonthreatening manner. Assess vaginal discharge further by asking about the amount, color, and odor. Normal vaginal discharge is small, clear or cloudy, and always nonirritating. A woman states that 2 weeks ago she had a urinary tract infection that was treated with an antibiotic. As a part of the interview, the nurse should ask, "Have you noticed: A) a change in your urination patterns?" B) any excessive vaginal bleeding?" C) any unusual vaginal discharge or itching?" D) any changes in your desire for intercourse?" Several medications may increase the risk of vaginitis. Broad-spectrum antibiotics alter the balance of normal flora, which may lead to the development of vaginitis. The other questions are not correct Which statement would be most appropriate when the nurse is introducing the topic of sexual relationships during an interview? A) "Now it is time to talk about your sexual history. When did you first have intercourse?" B) "Women often feel dissatisfied with their sexual relationships. Would it be okay to discuss this now?" C) "Often women have questions about their sexual relationship and how it affects their health. Do you have any questions?" D) "Most women your age have had more than one sexual partner. How many would you say you have had?" The nurse should begin with an open-ended question to assess individual needs. The nurse should include appropriate questions as a routine part of the history, because doing so communicates that the nurse accepts the individual's sexual activity and believes it is important. The nurse's comfort with discussion prompts the patient's interest and possibly relief that the topic has been introduced. This establishes a database for comparison with any future sexual activities and provides an opportunity to screen sexual problems. A 22-year-old woman has been considering using oral contraceptives. As a part of her history, the nurse should ask: A) "Do you have a history of heart murmurs?" B) "Will you be in a monogamous relationship?" C) "Have you thought this choice through carefully?" D) "If you smoke, how many cigarettes do you smoke per day?" Oral contraceptives, together with cigarette smoking, increase the risk for cardiovascular side effects. If cigarettes are used, then the nurse should assess smoking history. The other questions are not appropriate. A married couple has come to the clinic seeking advice on pregnancy. They have been trying to conceive for 4 months and have not been successful. What should the nurse do first? A) Ascertain whether either of them has been using broad-spectrum antibiotics. B) Explain that couples are considered infertile after 1 year of unprotected intercourse. C) Immediately refer the woman to an expert in pelvic inflammatory disease—the most common cause of infertility. D) Explain that couples are considered infertile after 3 months of engaging in unprotected intercourse and that they will need a referral to a fertility expert. - Infertility is considered after 1 year of engaging in unprotected sexual intercourse without conceiving. The other actions are not appropriate A nurse is assessing a patient's risk of contracting a sexually transmitted infection (STI). An appropriate question to ask would be: A) "You know that it's important to use condoms for protection, right?" B) "Do you use a condom with each episode of sexual intercourse?" C) "Do you have a sexually transmitted infection?" D) "You are aware of the dangers of unprotected sex, aren't you?" - In reviewing a patient's risk for sexually transmitted infections, the nurse should ask, in a nonconfrontational manner, whether condoms are used at each episode of sexual intercourse. Asking a person whether he or she has an infection does not address the risk. When the nurse is interviewing a preadolescent girl, which opening statement would be least threatening? A) "Do you have any questions about growing up?" B) "What has your mother told you about growing up?" C) "When did you notice that your body was changing?" D) "I remember being very scared when I got my period. How do you think you'll feel?" Try the open-ended, "When did you ... ?" rather than "Do you ... ?" This is less threatening because it implies that the topic is normal and unexceptional. When the nurse is discussing sexuality and sexual issues with adolescents, a permission statement helps to convey that it is normal to think or feel a certain way. Which of these is the best example of a permission statement? A) "It is okay that you have become sexually active." B) "Often girls your age have questions about sexual activity. Have you any questions?" C) "If it is okay with you, I'd like to ask you some questions about your sexual history." D) "Often girls your age engage in sexual activity. It is okay to tell me if you have had intercourse." - Start with a permission statement, "Often girls your age experience . . . ." This conveys that it is normal to think or feel a certain way, and it is important to relay that the topic is normal and unexceptional. The nurse is preparing to interview a postmenopausal woman. Which of these statements is true with regard to the history of a postmenopausal woman? A) The nurse should ask a postmenopausal woman if she ever has vaginal bleeding. B) Once a woman reaches menopause, the nurse does not need to ask any further history questions. C) The nurse should screen for monthly breast tenderness. D) Postmenopausal women are not at risk for contracting sexually transmitted infections and thus these questions can be omitted. Postmenopausal bleeding warrants further workup and referral. The other statements are not true. During the examination portion of a patient's visit, she will be in lithotomy position. Which statement below reflects some things that the nurse can do to make this more comfortable for her? A) Ask her to place her hands and arms behind her head. B) Elevate her head and shoulders to maintain eye contact. C) Allow her to choose to have her feet in the stirrups or have them resting side by side on the edge of the table. D) Allow her to keep her buttocks about 6 inches from the edge of the table to prevent her from feeling as if she will fall off. - The nurse should elevate her head and shoulders to maintain eye contact. The patient's arms should be placed at her sides or across the chest, not behind the head, because this position only tightens the abdominal muscles. The feet should be placed into the stirrups, knees apart, and buttocks at the edge of the examining table. Place the stirrups so the legs are not abducted too far An 18-year-old patient is having her first pelvic examination. Which action by the nurse is appropriate? A) Invite her mother to be present during the examination. B) Avoid the lithotomy position this first time because it can be uncomfortable and embarrassing. C) Raise the head of the examination table and give her a mirror so that she can view the exam. D) Drape her fully, leaving the drape between her legs elevated to avoid embarrassing her with eye contact. Use the techniques of the educational or mirror pelvic examination. This is a routine examination with some modifications in attitude, position, and communication. First, the woman is considered an active participant, one who is interested in learning and in sharing decisions about her own health care. The woman props herself up on one elbow, or the head of the table is raised. Her other hand holds a mirror between her legs, above the examiner's hands. The woman can see all that the examiner is doing and has a full view of her genitalia. The mirror works well for teaching normal anatomy and its relationship to sexual behavior. You can ask her if she would like to have a family member, friend, or chaperone present for the examination. The drape should be pushed down between the woman's legs so that the nurse can see her face. The nurse has just completed an inspection of a nulliparous woman's external genitalia. Which of these would be a description of a finding within normal limits? A) Redness of the labia majora B) Multiple nontender sebaceous cysts C) Discharge that is sticky and yellow-green D) Gaping and slightly shriveled labia majora - There should be no lesions, except for occasional sebaceous cysts. These are yellowish 1-cm nodules that are firm, nontender, and often multiple. The labia majora are dark pink, moist, and symmetrical; redness indicates inflammation or lesions. Discharge that is sticky and yellow-green may indicate infection. In the nulliparous woman, the labia majora meet in the midline, are symmetric and plump. The nurse is preparing for an internal genitalia examination of a woman. Which order of the examination is correct? A) Bimanual, speculum, rectovaginal B) Speculum, rectovaginal, bimanual C) Speculum, bimanual, rectovaginal D) Rectovaginal, bimanual, speculum The correct sequence is speculum examination, then bimanual examination after removing the speculum, and then rectovaginal examination. The examiner should change gloves before performing the rectovaginal examination to avoid spreading any possible infection During an internal examination of a woman's genitalia, the nurse will use which technique for proper insertion of the speculum? A) Instruct the woman to bear down, open the speculum blades, and apply in a swift, upward movement. B) Insert the blades of the speculum on a horizontal plane, turning them to a 30-degree angle while continuing to insert them. Ask the woman to bear down after the speculum is inserted. C) Instruct the woman to bear down, turn the width of the blades horizontally, and insert the speculum at a 45-degree angle downward toward the small of the woman's back. D) Lock the blades open by turning the thumbscrew. Once the blades are open, apply pressure to the introitus and insert the blades at a 45-degree angle downward to bring the cervix into view. The examiner should instruct the woman to bear down, turn the width of the blades horizontally, and insert the speculum at a 45-degree angle downward toward the small of the woman's back. See the text under "Speculum Examination" for more detail. The nurse is examining a 35-year-old female patient. During the history, the nurse notices that she has had two term pregnancies, and both babies were delivered vaginally. During the internal examination the nurse observes that the cervical os is a horizontal slit with some healed lacerations and that the cervix has some nabothian cysts that are small, smooth, and yellow. In addition, the nurse notices that the cervical surface is granular and red, especially around the os. Finally, the nurse notices the presence of stringy, opaque, odorless secretions. Which of these findings are abnormal? A) Nabothian cysts are present. B) The cervical os is a horizontal slit. C) The cervical surface is granular and red. D) Stringy and opaque secretions are present. Normal findings: Nabothian cysts may be present on the cervix after childbirth. The cervical os is a horizontal, irregular slit in the parous woman. Secretions vary according to the day of the menstrual cycle, and may be clear and thin or thick, opaque, and stringy. The surface is normally smooth, but cervical eversion, or ectropion may occur where the endocervical canal is "rolled out." Abnormal finding: The cervical surface should not be reddened or granular, which may indicate a lesion. A patient calls the clinic for instructions before having a Papanicolaou (Pap) smear. The most appropriate instructions from the nurse are: A) "If you are menstruating, please use pads to avoid placing anything into the vagina." B) "Avoid intercourse, inserting anything into the vagina, or douching within 24 hours of your appointment." C) "If you suspect that you have a vaginal infection, please gather a sample of the discharge to bring with you." D) "We would like you to use a mild saline douche before your examination. You may pick this up in our office." - When instructing a patient before a Papanicolaou (Pap) smear is obtained, the nurse should follow these guidelines: Do not obtain during the woman's menses or if a heavy infectious discharge is present. Instruct the woman not to douche, have intercourse, or put anything into the vagina within 24 hours before collecting the specimens. Any specimens will be obtained during the visit, not beforehand. During an examination, which tests will the nurse collect to screen for cervical cancer? A) Endocervical specimen, cervical scrape, and vaginal pool B) Endocervical specimen, vaginal pool, and acetic acid wash C) Endocervical specimen, KOH preparation, and acetic acid wash D) Cervical scrape, acetic acid wash, saline mount ("wet prep") Laboratories may vary in method, but usually the test consists of three specimens: endocervical specimen, cervical scrape, and vaginal pool. The other tests (acetic acid wash, KOH preparation, and saline mount) are used to test for sexually transmitted infections When performing the bimanual examination, the nurse notices that the cervix feels smooth and firm, is round, and is fixed in place (does not move). When cervical palpation is performed, the patient complains of some pain. The nurse's interpretation of these results should be which of these? A) These findings are all within normal limits. B) The cervical consistency should be soft and velvety—not firm. C) The cervix should move when palpated; an immobile cervix may indicate malignancy. D) Pain may occur during palpation of the cervix. Normally the cervix feels smooth and firm, as the consistency of the tip of the nose. It softens and feels velvety at 5 to 6 weeks of pregnancy (Goodell's sign). The cervix should be evenly rounded. With a finger on either side, the examiner should be able to move the cervix gently from side to side, and doing so should produce no pain for the patient. Hardness of the cervix may occur with malignancy. Immobility may occur with malignancy, and pain may occur with inflammation or ectopic pregnancy. The nurse is palpating a female patient's adnexa. The findings include a firm, smooth uterine wall; the ovaries are palpable and feel smooth and firm. The fallopian tube is firm and pulsating. The nurse's most appropriate course of action would be to: A) tell the patient that her examination was normal. B) give her an immediate referral to a gynecologist. C) suggest that she return in a month for a recheck to verify the findings. D) tell the patient that she may have an ovarian cyst that should be evaluated further. - Normally the uterine wall feels firm and smooth, with the contour of the fundus rounded. Ovaries are not often palpable, but when they are, they normally feel smooth, firm, and almond shaped and are highly movable, sliding through the fingers. The fallopian tube is not palpable normally. No other mass or pulsation should be felt. Pulsation or palpable fallopian tube suggests ectopic pregnancy, which warrants immediate referral A 65-year-old woman is in the office for routine gynecologic care. She had a complete hysterectomy 3 months ago after cervical cancer was detected. The nurse knows that which of these statements is true with regard to this visit? A) Her cervical mucosa will be red and dry looking. B) She will not need to have a Pap smear done. C) The nurse can expect to find that her uterus will be somewhat enlarged and her ovaries small and hard. D) The nurse should plan to lubricate the instruments and the examining hand well to avoid a painful examination. In the aging adult woman, natural lubrication is decreased; to avoid a painful examination, the nurse should take care to lubricate instruments and the examining hand adequately. Menopause, with the resulting decrease in estrogen production, shows numerous physical changes. The cervix shrinks and looks pale and glistening. With the bimanual examination, the uterus feels smaller and firmer and the ovaries are not palpable normally. Women should continue cervical cancer screening up to age 70 years if they have an intact cervix and are in good health. Women who have had a total hysterectomy for benign gynecologic disease do not need cervical cancer screening, but if the hysterectomy was done for cervical cancer, then Pap tests should continue until the patient has a 10-year history of no abnormal results. The nurse is preparing to examine the external genitalia of a school-age girl. Which of these positions would be most appropriate in this situation? A) In the parent's lap B) In a frog-leg position on the examining table C) In the lithotomy position with the feet in stirrups D) Lying flat on the examining table with legs extended - For school-age children it is best to place them on the examining table in a frog-leg position. With toddlers and preschoolers, it is best to have the child on the parent's lap in a frog-leg position. When assessing a newborn infant's genitalia, the nurse notices that the genitalia are somewhat engorged. The labia majora are swollen, the clitoris looks large, and the hymen is thick. The vaginal opening is difficult to visualize. The infant's mother states that she is worried about the labia being swollen. The nurse should reply: A) "This is a normal finding in newborns and should resolve within a few weeks." B) "This could indicate an abnormality and may need to be evaluated by a physician." C) "We will need to have estrogen levels evaluated to make sure that they are within normal limits." D) "We will need to keep close watch over the next few days to see if the genitalia decrease in size." It is normal for a newborn's genitalia to be somewhat engorged. A sanguineous vaginal discharge or leukorrhea is normal during the first few weeks because of the maternal estrogen effect. During the early weeks, the genital engorgement resolves, and the labia minora atrophy and remain small until puberty. During a vaginal examination of a 38-year-old woman, the nurse notices that the vulva and vagina are erythematous and edematous with thick, white, curdlike discharge adhering to the vaginal walls. The woman reports intense pruritus and thick white discharge from her vagina. The nurse knows that these history and physical examination findings are most consistent with which of these conditions? A) Candidiasis B) Trichomoniasis C) Atrophic vaginitis D) Bacterial vaginosis The woman with candidiasis often reports intense pruritus and thick white discharge. The vulva and vagina are erythematous and edematous. The discharge is usually thick, white, and curdlike. Infection with trichomoniasis causes a profuse, watery, gray-green, and frothy discharge. Bacterial vaginosis causes a profuse discharge that has a "foul, fishy, rotten" odor. Atrophic vaginitis may have a mucoid discharge. See Table 26-5 for complete descriptions of each option. A 22-year-old woman is being seen at the clinic for problems with vulvar pain, dysuria, and fever. On physical examination, the nurse notices clusters of small, shallow vesicles with surrounding erythema on the labia. There is also inguinal lymphadenopathy present. The most likely cause of these lesions is: A) pediculosis pubis. B) contact dermatitis. C) human papillomavirus. D) herpes simplex virus type 2. Herpes simplex virus type 2 presents with clusters of small, shallow vesicles with surrounding erythema that erupt on the genital areas. There is also the presence of inguinal lymphadenopathy. The individual reports local pain, dysuria, and fever. See Table 26-2 for more information and descriptions of the other conditions. When performing an external genitalia examination of a 10-year-old girl, the nurse notices that there is no pubic hair, and the mons and the labia are covered with fine vellus hair. These findings are consistent with stage _____ of sexual maturity, according to the Sexual Maturity Rating scale. A) 1 B) 2 C) 3 D) 4 Sexual Maturity Rating stage 1 is the preadolescent stage. There is no pubic hair. The mons and labia are covered with fine, vellus hair as on the abdomen. See Table 26-1. A 46-year-old woman is in the clinic for her annual gynecologic examination. She voices a concern about ovarian cancer because her mother and sister died of it. The nurse knows that which of these statements is correct regarding ovarian cancer? A) Ovarian cancer rarely has any symptoms. B) The Pap smear detects the presence of ovarian cancer. C) Women at high risk for ovarian cancer should have annual transvaginal ultrasonography for screening. D) Women over age 40 years should have a thorough pelvic examination every 3 years. With ovarian cancer, the patient may have abdominal pain, pelvic pain, increased abdominal size, bloating, and nonspecific gastrointestinal symptoms, or she may be asymptomatic. The Pap smear does not detect the presence of ovarian cancer. Annual transvaginal ultrasonography may detect ovarian cancer at an earlier stage in women who are at high risk for it. During a bimanual examination, the nurse detects a solid tumor on the ovary that is heavy and fixed, with a poorly defined mass. This finding is suggestive of: A) an ovarian cyst. B) endometriosis. C) ovarian cancer. D) an ectopic pregnancy. Ovarian tumors that are solid, heavy, and fixed, with poorly defined mass are suggestive of malignancy. Benign masses may feel mobile and solid. An ovarian cyst may feel smooth, round, fluctuant, mobile, and nontender. With an ectopic pregnancy, the examiner may feel a palpable, tender pelvic mass that is solid, mobile, and unilateral. Endometriosis may have masses (in various locations in the pelvic area) that are small, firm, nodular and tender to palpation, with enlarged ovaries. A 25-year-old woman comes to the emergency department with a sudden fever of 101° F and abdominal pain. Upon examination, the nurse notices that she has rigid, boardlike lower abdominal musculature. When the nurse tries to perform a vaginal examination, the patient has severe pain when the uterus and cervix are moved. The nurse knows that these signs and symptoms are suggestive of: A) endometriosis. B) uterine fibroids. C) ectopic pregnancy. D) pelvic inflammatory disease. These signs and symptoms are suggestive of acute pelvic inflammatory disease, also known as acute salpingitis. See Table 26-7. For description of endometriosis and uterine fibroids, see Table 26-6; for description of ectopic pregnancy, see Table 26-7. During an external genitalia examination of a woman, the nurse notices several lesions around the vulva. The lesions are pink, moist, soft, and pointed papules. The patient states that she is not aware of any problems in that area. The nurse recognizes that these lesions may be: A) syphilitic chancre. B) herpes simplex virus type 2 (herpes genitalis). C) human papillomavirus (HPV), or genital warts. D) pediculosis pubis (crab lice). HPV lesions are painless, warty growths that the woman may not notice. Lesions are pink or flesh colored, soft, pointed, moist, warty papules that occur in single or multiple cauliflower-like patches around the vulva, introitus, anus, vagina, or cervix. Herpetic lesions are painful clusters of small, shallow vesicles with surrounding erythema. Syphilitic chancres begin as a solitary silvery papule that erodes to a red, round or oval, superficial ulcer with a yellowish discharge. Pediculosis pubis causes severe perineal itching and excoriations and erythematous areas. See Table 26-2. During an examination, the nurse would expect the cervical os of a woman who has never had children to appear: A) stellate. B) small and round. C) as a horizontal irregular slit. D) everted. - The cervical os in a nulliparous woman is small and round. In the parous woman, it is a horizontal, irregular slit that also may show healed lacerations on the sides. See Figure 26-13. A woman has just been diagnosed with HPV, or genital warts. The nurse should counsel her to receive regular examinations because this virus makes her at a higher risk for _____ cancer. A) uterine B) cervical C) ovarian D) endometrial - HPV is the virus responsible for most cases of cervical cancer, not the other options. During an internal examination, the nurse notices that the cervix bulges outside the introitus when the patient is asked to strain. The nurse will document this as: A) uterine prolapse, graded first degree. B) uterine prolapse, graded second degree. C) uterine prolapse, graded third degree. D) a normal finding. - The cervix should not be found to bulge into the vagina. Uterine prolapse is graded as follows: first degree—cervix appears at introitus with straining; second degree—cervix bulges outside introitus with straining; and third degree—whole uterus protrudes, even without straining (essentially, uterus is inside out). A 35-year-old woman is at the clinic for a gynecologic examination. During the examination, she asks the nurse, "How often do I need to have this Pap test done?" Which reply by the nurse is correct? A) "It depends. Do you smoke?" B) "This will need to be done annually until you are 65." C) "If you have 2 consecutive normal Pap tests, then you can wait 5 years between tests." D) "After age 30, if you have 3 consecutive normal Pap tests, then you may be screened every 2 to 3 years." Cervical cancer screening with the Pap test continues annually until age 30. After age 30, if the woman has 3 consecutive normal Pap tests, then women may be screened every 2 to 3 years. The nurse is palpating an ovarian mass during an internal examination of a 63-year-old woman. Which findings of the mass's characteristics would suggest the presence of an ovarian cyst? Select all that apply. A) Heavy and solid B) Mobile and fluctuant C) Mobile and solid D) Fixed E) Smooth and round F) Poorly defined - , E An ovarian cyst (fluctuant ovarian mass) is usually asymptomatic, and would feel like a smooth, round, fluctuant, mobile, nontender mass on the ovary. A mass that is heavy, solid, fixed, and poorly defined suggests malignancy. A benign mass may feel mobile and solid. When examining the eye, the nurse is aware that the bulbar conjunctiva: 1. overlies the sclera. 2. covers the iris and pupil. 3. is visible at the inner canthus of the eye. 4. is a thin mucous membrane that lines the lids. 1 The bulbar conjunctiva overlies the eyeball with the white sclera showing through. During ocular examinations, the nurse keeps in mind that movement of the extraocular muscles is: 1. decreased in the elderly. 2. impaired in a patient with cataracts. 3. stimulated by cranial nerves I and II. 4. stimulated by cranial nerves III, IV, and VI. 4 Movement of the extraocular muscles is stimulated by three cranial nerves: III, IV, and VI. Which of the following statements regarding the outer layer of the eye is true? 1. The outer layer of the eye is very sensitive to touch. 2. The outer layer of the eye is darkly pigmented to prevent light from reflecting internally. 3. The trigeminal (CN V) and the trochlear (CN IV) nerves are stimulated when the outer surface of the eye is stimulated. 4. The visual receptive layer of the eye in which light waves are changed into nerve impulses is located in the outer layer of the eye. 1 The cornea and the sclera make up the outer layer of the eye. The cornea is very sensitive to touch. When examining a patient's eyes, the nurse recalls that stimulation of the sympathetic branch of the autonomic nervous system: 1. causes pupillary constriction. 2. adjusts the eye for near vision. 3. elevates the eyelid and dilates the pupil. 4. causes contraction of the ciliary body. 3 Stimulation of the sympathetic branch dilates the pupil and elevates the eyelid. Intraocular pressure is determined by the: 1. thickness or bulging of the lens. 2. posterior chamber as it accommodates for an increase in fluid. 3. contraction of the ciliary body in response to the aqueous within the eye. 4. amount of aqueous produced and resistance to its outflow at the angle of the anterior chamber. 4 Intraocular pressure is determined by a balance between the amount of aqueous produced and resistance to its outflow at the angle of the anterior chamber. The nurse is conducting a visual examination. Which of the following statements regarding visual pathways and visual fields is true? 1. The right side of the brain interprets vision for the right eye. 2. The image formed on the retina is upside down and reversed from its actual appearance in the outside world. 3. Light rays are refracted through the transparent media of the eye before striking the pupil. 4. The light impulses are conducted through the optic nerve to the temporal lobes of the brain. 2 The image formed on the retina is upside down and reversed from its actual appearance in the outside world. The nurse is testing a patient's visual accommodation, which refers to: 1. pupillary constriction when looking at a near object. 2. pupillary dilation when looking at a far object. 3. changes in peripheral vision in response to light. 4. involuntary blinking in the presence of bright light. 1 The muscle fibers of the iris contract the pupil in bright light and accommodate for near vision. A patient has a normal pupillary light reflex. The nurse recognizes that this indicates that: 1. the eyes converge to focus on the light. 2. light is reflected at the same spot in both eyes. 3. the eye focuses the image in the center of the pupil. 4. constriction of both pupils occurs in response to bright light. 4 The pupillary light reflex is the normal constriction of the pupils when bright light shines on the retina. A mother asks when her newborn infant's eyesight will be developed. The nurse should reply: 1. "Vision is not totally developed until 2 years of age." 2. "Infants develop the ability to focus on an object at around 8 months." 3. "By about 3 months, infants develop more coordinated eye movements and can fixate on an object." 4. "Most infants have uncoordinated eye movements for the first year of life." 3 By 3 to 4 months of age, the infant establishes binocularity and can fixate on a single image with both eyes simultaneously. Which of the following physiological changes is responsible for presbyopia? 1. Degeneration of the cornea 2. Loss of lens elasticity 3. Decreased adaptation to darkness 4. Decreased distance vision abilities 2 The lens loses elasticity and decreases its ability to change shape to accommodate for near vision. This condition is called presbyopia. Which of the following would the nurse expect to find when examining the eyes of a black patient? 1. Increased night vision 2. A dark retinal background 3. Increased photosensitivity 4. Narrowed palpebral fissures 2 There is an ethnically based variability in the color of the iris and in retinal pigmen- tation, with darker irides having darker retinas behind them. A 52-year-old patient describes the presence of occasional "floaters or spots" moving in front of his eyes. The nurse should: 1. examine the retina to determine the number of floaters. 2. presume the patient has glaucoma and refer him for further testing. 3. consider this an abnormal finding and refer him to an ophthalmologist. 4. know that "floaters" are usually not significant and are caused by condensed vitreous fibers. 4 Floaters are a common sensation with myopia or after middle age owing to condensed vitreous fibers. Usually, they are not significant. The nurse is preparing to assess the visual acuity of a 16-year-old patient. How would the nurse proceed? 1. Perform the confrontation test. 2. Ask the patient to read the print on a hand-held Jaeger card. 3. Use the Snellen chart positioned 20 feet away from the patient. 4. Determine the patient's ability to read newsprint at a distance of 12 to 14 inches. 3 The Snellen alphabet chart is the most commonly used and accurate measure of visual acuity. A patient's vision is recorded as 20/30 when the Snellen eye chart is used. The nurse recognizes that these results indicate that: 1. at 30 feet the patient can read the entire chart. 2. the patient can read at 20 feet what a person with normal vision can read at 30 feet. 3. the patient can read the chart from 20 feet in the left eye and 30 feet in the right eye. 4. the patient can read from 30 feet what a person with normal vision can read from 20 feet. 2 The top number indicates the distance the person is standing from the chart; the denominator gives the distance at which a normal eye can see. A patient is unable to read the 20/100 line on the Snellen chart. The nurse would: 1. refer the patient to an ophthalmologist or optometrist for further evaluation. 2. assess whether the patient can count the nurse's fingers when they are placed in front of his or her eyes. 3. ask the patient to put on his or her reading glasses and attempt to read the Snellen chart again. 4. shorten the distance between the patient and the chart and ask him or her to read the smallest line of print possible. 1 If vision is poorer than 20/30, refer the person to an ophthalmologist or optometrist. A patient's vision is recorded as 20/80 in each eye. The nurse recognizes that this finding indicates that: 1. the patient has poor vision. 2. the patient has acute vision. 3. the patient has normal vision. 4. the patient is presbyopic. 1 Normal visual acuity is 20/20 in each eye. The larger the denominator, the poorer the vision. When performing the corneal light reflex assessment, the nurse notes that the light is reflected at 2 o'clock in each eye. The nurse would: 1. consider this a normal finding. 2. refer the individual for further evaluation. 3. document this as an asymmetric light reflex. 4. perform the confrontation test to validate the findings. 1 Reflection of the light on the corneas should be in exactly the same spot on each eye, or symmetric. Which of the following is an expected normal finding when performing the diagnostic positions test? 1. Convergence of the eyes 2. Parallel movement of both eyes 3. Nystagmus in extreme superior gaze 4. A slight amount of lid lag when moving the eyes from a superior to inferior position 2 A normal response for the diagnostic positions test is parallel tracking of the object with both eyes. In assessing the sclera of a black patient, which of the following would be an expected finding? 1. Yellow fatty deposits over the cornea 2. Pallor near the outer canthus of the lower lid 3. Yellow color of the sclera that extends up to the iris 4. The presence of small brown macules on the sclera 4 In dark-skinned people, one normally may see small brown macules in the scle A 60-year-old man is at the clinic for an eye examination. The nurse suspects that he has a ptosis of one eye. How would the nurse check for this? 1. Perform the confrontation test. 2. Assess the individual's near vision. 3. Observe the distance between the palpebral fissures. 4. Perform the corneal light test and look for symmetry of the light reflex. 3 Ptosis is drooping of the upper eyelid that would be apparent by observing the distance between the upper and lower eyelids. During an examination of the eye, the nurse would expect what normal finding when assessing the lacrimal apparatus? 1. The presence of tears along the inner canthus 2. A blocked nasolacrimal duct in a newborn infant 3. A slight swelling over the upper lid and along the bony orbit if the individual has a cold 4. The absence of drainage from the puncta when pressing against the inner orbital rim 4 There should be no swelling, redness, or drainage from the puncta. When assessing the pupillary light reflex, the nurse should use which technique? 1. Shine a penlight from directly in front of the patient and inspect for pupillary constriction. 2. Ask the patient to follow the penlight in eight directions and observe for bilateral pupil constriction. 3. Shine a light across the pupil from the side and observe for direct and consensual pupillary constriction. 4. Ask the patient to focus on a distant object. Then ask the patient to follow the penlight to about 7 cm from the nose. 3 To test the pupillary light reflex, advance a light in from the side and note the direct and consensual pupillary constriction. The nurse is assessing a patient's eyes for the accommodation response and would expect to see: 1. dilation of the pupils. 2. a consensual light reflex. 3. conjugate movement of the eyes. 4. convergence of the axes of the eyes. 4 The accommodation reaction includes pupillary constriction and convergence of the axes of the eyes. In using the ophthalmoscope to assess a patient's eyes, the nurse notes a red glow in the patient's pupils. On the basis of this finding, the nurse would: 1. suspect that there is an opacity in the lens or cornea. 2. check the light source of the ophthalmoscope to verify that it is functioning. 3. consider this a normal reflection of the ophthalmoscope light off the inner retina. 4. continue with the ophthalmoscopic examination and refer the patient for further evaluation. 3 The red glow filling the person's pupil is the red reflex. This is caused by the reflection of the ophthalmoscope light off the inner retina. When the retina is examined, which of the following is considered a normal finding? 1. An optic disc that is a yellow-orange color 2. Optic disc margins that are blurred around the edges 3. The presence of pigmented crescents in the macular area 4. The presence of the macula located on the nasal side of the retina 1 The optic disc is located on the nasal side of the retina. It is a creamy yellow-orange to pink color. A 2-week-old infant can fixate on an object but not follow a light or bright toy. The nurse would: 1. consider this a normal finding. 2. assess the pupillary light reflex for possible blindness. 3. continue with the examination and assess visual fields. 4. expect that a 2-week-old infant should be able to fixate and follow an object. 1 By 2 to 4 weeks an infant can fixate on an object. By 1 month, the infant should fixate and follow a bright light or toy. To assess color vision on a male child, the nurse would: 1. check color vision annually until the age of 18 years. 2. ask the child to identify the color of his or her clothing. 3. test for color vision once between the ages of 4 and 8. 4. begin color vision screening at the child's 2-year check-up. 3 Test only boys for color vision once between the ages of 4 and 8 years. The nurse is performing an eye-screening clinic at the day care center. When examining a 2-year-old child, the nurse suspects that the child has "lazy eye" and would: 1. examine the external structures of the eye. 2. assess visual acuity with the Snellen eye chart. 3. assess the child's visual fields with the confrontation test. 4. test for strabismus by performing the corneal light reflex test. 4 Testing for strabismus is done by performing the corneal light reflex test. The light should be reflected at exactly the same spot in both eyes. The nurse is performing an eye assessment on an 80-year-old patient. Which of the following findings is considered abnormal? 1. A decrease in tear production 2. Unequal pupillary constriction in response to light 3. The presence of arcus senilis seen around the cornea 4. Loss of the outer hair on the eyebrows due to a decrease in hair follicles 2 Pupils are small in old age, and the pupillary light reflex may be slowed, but pupillary constriction should be symmetric. The nurse notes the presence of periorbital edema when performing an eye assessment on a 70-year-old patient. The nurse will: 1. check for the presence of exophthalmos. 2. suspect that the patient has hyperthyroidism. 3. ask the patient if he or she has a history of heart failure. 4. assess for blepharitis because this is often associated with periorbital edema. 3 Periorbital edema occurs with local infections, crying, and systemic conditions such as heart failure, renal failure, allergy, and hypothyroidism. When a light is directed across the iris of the eye from the temporal side, the examiner is assessing for: 1. drainage from dacryocystitis. 2. the presence of conjunctivitis over the iris. 3. the presence of shadows, which may indicate glaucoma. 4. a scattered light reflex, which may be indicative of cataracts. 3 The presence of shadows in the anterior chamber may be a sign of acute anglelosure glaucoma. In a patient who has anisocoria, the nurse would expect to observe: 1. dilated pupils. 2. excessive tearing. 3. pupils of unequal size. 4. an uneven curvature of the lens. 3 Unequal pupil size is termed anisocoria. It exists normally in 5% of the population but may also be indicative of central nervous system disease. A patient comes to the emergency department after a boxing match, and his left eye is swollen almost shut. He has bruises on his face and neck. He says he is worried because he "can't see well" from his left eye. The physician suspects retinal damage. The nurse recognizes that signs of retinal detachment include 1. loss of central vision. 2. shadow or diminished vision in one quadrant or one half visual field. 3. loss of peripheral vision. 4. sudden loss of pupillary constriction and accommodation. 2 With retinal detachment, the person has shadow or diminished vision in one quadrant or one half visual field. A patient comes into the clinic complaining of pain in her right eye. On examination, the nurse sees a pustule at the lid margin that is painful to touch, red, and swollen. The nurse recognizes that this is 1. a chalazion. 2. a hordeolum (stye). 3. dacryocystitis. 4. blepharitis. 3 A hordeolum, or stye, is a painful, red, and swollen pustule at the lid margin. A 68-year-old woman is in the eye clinic for a checkup. She tells the nurse that she has been having trouble with reading the paper, sewing, and even seeing the faces of her grandchildren. On examination, the nurse notes that she has some loss of central vision but her peripheral vision is normal. These findings suggest that: 1. she may have macular degeneration. 2. her vision is normal for someone her age. 3. she has the beginning stages of cataract formation. 4. she has increased intraocular pressure or glaucoma. 1 Macular degeneration is the most common cause of blindness. It is characterized by loss of central vision. A patient comes into the emergency department after an accident at work. A machine blew dust into his eyes and he was not wearing safety glasses. The nurse examines his corneas by shining a light from the side across the cornea. What findings would suggest that he has suffered a corneal abrasion? 1. Smooth and clear corneas 2. Opacity of the lens behind the cornea 3. Bleeding from the areas across the cornea 4. A shattered look to the light rays reflecting off the cornea 4 A corneal abrasion causes irregular ridges in reflected light, producing a shattered look to light rays. An ophthalmic examination reveals papilledema. The nurse is aware that this finding indicates: 1. retinal detachment. 2. diabetic retinopathy. 3. acute-angle glaucoma. 4. increased intracranial pressure. 4 Papilledema, or choked disk, is caused by increased intracranial pressure, which is caused by a space-occupying mass such as a brain tumor or hematoma. This pressure causes venous stasis in the globe, redness, congestion, and elevation of the optic disc, blurred margins, hemorrhages, and absent venous pulsations. During an ophthalmscopic examination of the eye, the examiner notes areas of exudate that look like "cotton wool" or fluffy gray-white cumulus clouds. This finding indicates which possible problem? 1. Diabetes 2. Hyperthyroidism 3. Glaucoma 4. Hypotension 1 Soft exudates or "cotton wool" areas look like fluffy gray-white cumulus clouds, They occur with diabetes, hypertension, subacute bacterial endocarditis, lupus, and papilledema of any cause. During a physical education class, a student is hit in the eye with the end of a baseball bat. When examined in the emergency department, the nurse notes the presence of blood in the anterior chamber of the eye. This finding indicates the presence of: 1. hypopyon. 2. hyphema. 3. corneal abrasion. 4. iritis. 2 Hyphema is the term for blood in anterior chamber is a serious result of blunt trauma (a fist or a baseball) or spontaneous hemorrhage and may indicate scleral rupture or major intraocular trauma. During an examination, a patient states that she was diagnosed with open-angle glaucoma 2 years ago. The nurse recalls that there are various types of glaucoma, such as open-angle glaucoma and closed-angle glaucoma. Which of the following are characteristics of open-angle glaucoma? Select all that apply. 1. The patient may experience sensitivity to light, nausea, and halos around lights. 2. It is the most common type of glaucoma. 3. Immediate treatment is needed. 4. Vision loss begins with peripheral vision. 5. It causes sudden attacks of increased pressure that causes blurred vision. 6. There are virtually no symptoms 2, 4, 6 Open-angle glaucoma is the most common type of glaucoma; there are virtually no symptoms. Vision loss begins with the peripheral vision, which often goes unnoticed because individuals learn to compensate intuitively by turning their heads. The other characteristics are those of closed-angle glaucoma. A physician tells the nurse that a patient's vertebra prominens is tender and asks the nurse to re-evaluate the area in 1 hour. The area of the body the nurse will assess is the area: 1.just above the diaphragm. 2.just lateral to the knee cap. 3.at the level of the C7 vertebra. 4.at the level of the T11 vertebra. 3 The C7 vertebra has a long spinous process, called the vertebra prominens, that is palpable when the head is flexed. A mother brings her 2-month-old daughter in for an examination and says, "My daughter rolled over against the wall and now I have noticed that she has this spot that is soft on the top of her head. Is there something terribly wrong?" The nurse's response would be: 1."Perhaps that could be a result of your dietary intake during pregnancy." 2."Your baby may have craniosynostosis, a disease of the sutures of the brain." 3."That 'soft spot' you are referring to may be an indication of cretinism or congeni- tal hypothyroidism." 4."That 'soft spot' is normal, and actually allows for growth of the brain during the first year of your baby's life." 4 Membrane-covered "soft spots" allow for growth of the brain during the first year. They gradually ossify; the triangular-shaped posterior fontanel is closed by 1 to 2 months, and the diamond-shaped anterior fontanel closes between 9 months and 2 years. The nurse notices that a patient's palpebral fissures are not symmetrical. On examination, the nurse may find that there has been damage to:1. 1.CN III. 2.CN V. 3.CN VII. 4.CN VIII. 3 Facial muscles are mediated by CN VII; asymmetry of palpebral fissures may be due to CN VII damage. A patient is unable to differentiate between sharp and dull stimulation to both sides of her face. The nurse suspects: 1. Bell's palsy. 2. damage to the trigeminal nerve. 3. frostbite with resultant paresthesia to the cheeks. 4. scleroderma with a pronounced proliferation of connective tissue in the face and cheeks. 2 Facial sensations of pain or touch are mediated by CN V, the trigeminal nerve. When examining the face, the nurse is aware that the two pairs of salivary glands that are accessible to examination are the _____ glands. 1. occipital and submental 2. parotid and jugulodigastric 3. parotid and submandibular 4. submandibular and occipital 3 Two pairs of salivary glands accessible to examination on the face are the parotid glands in the cheeks over the mandible, anterior to and below the ear, and the submandibular glands, beneath the mandible at the angle of the jaw. The parotid glands are not normally palpable. A patient comes to the clinic complaining of neck and shoulder pain and is unable to turn her head. The nurse suspects damage to CN _____ and proceeds with the examination by: 1. XI; palpating the anterior and posterior triangles. 2. XI; asking the patient to shrug her shoulders against resistance. 3. XII; percussing the sternomastoid and submandibular neck muscles. 4. XII; assessing for a positive Romberg's sign. 2 The major neck muscles are the sternomastoid and the trapezius. They are innervated by CN XI, the spinal accessory. The innervated muscles assist with head rotation and head flexion, movement of the shoulders, and extension and turning of the head. The muscles in the neck that are innervated by CN XI are the: 1. sternomastoid and trapezius. 2. spinal accessory and omohyoid. 3. trapezius and sternomandibular. 4. sternomandibular and spinal accessory. 1 The major neck muscles are the sternomastoid and the trapezius. They are innervated by CN XI, the spinal accessory. A patient's laboratory data reveal an elevated thyroxine level. The nurse would proceed with an examination of the: 1. thyroid gland. 2. parotid gland. 3. adrenal gland. 4. thyroxine gland. 1 The thyroid gland is a highly vascular endocrine gland that secretes thyroxine (T4) and tri-iodothyronine (T3). A patient says that she has recently noticed a lump in the front of her neck below her "Adam's apple" that seems to be getting bigger. During the assessment, the finding that reassures the nurse that this may not be a cancerous thyroid nodule is that the lump (nodule): 1. is tender. 2. is mobile and not hard. 3. disappears when the patient smiles. 4. is hard and fixed to the surrounding structures. 2 Suspect any painless, rapidly growing nodule, especially the appearance of a single nodule in a young person. Cancerous nodules tend to be hard and are fixed to surrounding structures. The nurse notices that a patient's submental lymph nodes are enlarged. In an effort to identify the cause of the node enlargement, the nurse would assess the: 1. infraclavicular area. 2. supraclavicular area. 3. area distal to the enlarged node. 4. area proximal to the enlarged node. 4 When nodes are abnormal, check the area they drain for the source of the problem. Explore the area proximal (upstream) to the location of the abnormal node. The nurse is aware that the four areas in the body where lymph nodes are accessible are the: 1. head, breasts, groin, and abdomen. 2. arms, breasts, inguinal area, and legs. 3. head and neck, arms, breasts, and axillae. 4. head and neck, arms, inguinal area, and axillae. 4 Nodes are located throughout the body, but are accessible to examination only in four areas: head and neck, arms, axillae, and inguinal region. A mother brings her newborn in for an assessment and asks, "Is there something wrong with my baby? His head seems so big." The nurse knows the following about relative proportions of the head and trunk of the newborn: 1. At birth, the head is one fifth the total length. 2. Head circumference should be greater than chest circumference at birth. 3. The head size reaches 90% of its final size when the child is 3 years old. 4. When the anterior fontanel closes at 2 months, the head will be more proportioned to the body. 2 During the fetal period, head growth predominates. Head size is greater than chest circumference at birth, and the head size grows during childhood, reaching 90% of its final size when the child is age 6 years. A patient, an 85-year-old woman, is complaining about the fact that the bones in her face have become more noticeable. What explanation should the nurse give to her? 1. Diets low in protein and high in carbohydrates may cause enhanced facial bones. 2. It is probably because she doesn't use a dermatologically approved moisturizer. 3. It is probably due to a combination of factors such as decreased elasticity, subcutaneous fat, and moisture in her skin. 4. Facial skin becomes more elastic with age. This increased elasticity causes the skin to be more taught, drawing attention to the facial bones. 3 The facial bones and orbits appear more prominent in the aging adult, and the facial skin sags owing to decreased elasticity, decreased subcutaneous fat, and decreased moisture in the skin. A patient presents with excruciating headache pain on one side of his head, especially around his eye, forehead, and cheek that lasts about 1/2 to 2 hours, occurring once or twice each day. The nurse suspects: 1. hypertension. 2. cluster headaches. 3. tension headaches. 4. migraine headaches. 2 Cluster headaches produce pain around the eye, temple, forehead, and cheek and are unilateral and always on the same side of the head. They are excruciating and occur once or twice per day and last 1/2 to 2 hours each. A patient complains that while studying for an examination he began to notice a severe headache in the frontotemporal area of his head that is throbbing and is somewhat relieved when he lies down. He tells the nurse that his mother also had these headaches. The nurse suspects that he may be suffering from: 1. hypertension. 2. cluster headaches. 3. tension headaches. 4. migraine headaches. 4 Migraine headaches tend to be supraorbital, retro-orbital, or frontotemporal with a throbbing quality. They are of a severe quality and are relieved by lying down. Migraines are associated with family history of migraine. A 19-year-old college student is brought to the emergency department with a severe headache he describes as "Like nothing I've ever had before." His temperature is 104° F, and he has a stiff neck. What do these signs and symptoms suggest? 1. Head injury 2. Cluster headache 3. Migraine headache 4. Meningeal inflammation 4 Acute onset of neck stiffness and pain along with headache and fever occurs with meningeal inflammation. A severe headache in an adult or child who has never had it before is a red flag. During a well-baby check, the nurse notices that a 1-week-old infant's face looks small compared with his cranium, which seems enlarged. On further examination, the nurse also notes dilated scalp veins and downcast, or "setting sun," eyes. The nurse suspects which condition? 1. Craniotabes 2. Microcephaly 3. Hydrocephalus 4. Caput succedaneum 3 Hydrocephalus occurs with obstruction of drainage of cerebrospinal fluid that results in excessive accumulation, increasing intracranial pressure, and enlargement of the head. The face looks small compared with the enlarged cranium, and dilated scalp veins and downcast, or "setting sun," eyes are noted. The temporomandibular joint is just below the temporal artery and anterior to the:1. hyoid. 2. vagus. 3. tragus. 4. mandible. 3 The temporomandibular joint is just below the temporal artery and anterior to the tragus. A patient has come in for an examination and states, "I have this spot in front of my ear lobe here on my cheek that seems to be getting bigger and is real tender. What do you think it is?" The nurse notes swelling below the angle of the jaw and suspects that it could be an inflammation of his: 1. thyroid gland. 2. parotid gland. 3. occipital lymph node. 4. submental lymph node. 2 Swelling with the parotid gland occurs below the angle of the jaw and is most visible when the head is extended. Painful inflammation occurs with mumps, and swelling also occurs with abscesses or tumors. Swelling occurs anterior to the lower ear lobe. A male patient with a history of AIDS has come in for an examination and he states, "I think that I have the mumps." The nurse would begin by examining the: 1. thyroid gland. 2. parotid gland. 3. cervical lymph nodes. 4. mouth and skin for lesions. 2 The parotid gland may become swollen with the onset of mumps, and parotid enlargement has been found with HIV. The nurse suspects that a patient has hyperthyroidism and laboratory data indicate that the patient's thyroxine and tri-iodothyronine hormone levels are elevated. Which of the following findings would the nurse most likely find on examination? 1. Tachycardia 2. Constipation 3. Rapid dyspnea 4. Atrophied nodular thyroid 1 Thyroxine and tri-iodothyronine are thyroid hormones that stimulate the rate of cellular metabolism, thus resulting in tachycardia. With an enlarged thyroid as in hyperthyroidism, you might expect to find diffuse enlargement (goiter) or a nodular lump. A visitor from Poland who does not speak English seems to be somewhat apprehensive about the nurse examining his neck. He would probably be most comfortable with the nurse examining his thyroid: 1. from behind with the nurse's hands placed firmly around his neck. 2. from the side with the nurse's eyes averted toward the ceiling and thumbs on his neck. 3. from the front with the nurse's thumbs placed on either side of his trachea and his head tilted forward. 4. from the front with the nurse's thumbs placed on either side of his trachea and his head tilted backward. 3 Examining this patient's thyroid from the back may be unsettling for him. It would be best to examine his thyroid using the anterior approach, asking him to tip his head forward and to the right and then the left. A patient's thyroid is enlarged, and the nurse is preparing to auscultate the thyroid for the presence of a bruit. A bruit is a: 1. low gurgling sound best heard with the diaphragm of the stethoscope. 2. loud, whooshing, blowing sound best heard with the bell of the stethoscope. 3. soft, whooshing, pulsatile sound best heard with the bell of the stethoscope. 4. high-pitched tinkling sound best heard with the diaphragm of the stethoscope. 3 If the thyroid gland is enlarged, auscultate it for the presence of a bruit, which is a soft, pulsatile, whooshing, blowing sound heard best with the bell of the stethoscope. The nurse notices that an infant has a large, soft lump on the side of his head and that his mother is very concerned. She tells the nurse that she noticed the lump about 8 hours after her baby's birth, and that it seems to be getting bigger. One possible explanation for this is: 1. hydrocephalus. 2. craniosynostosis. 3. cephalhematoma. 4. caput succedaneum. 3 A cephalhematoma is a subperiosteal hemorrhage that is the result of birth trauma. It is soft, fluctuant, and well defined over one cranial bone. It appears several hours after birth and gradually increases in size. A mother brings in her newborn infant for an assessment and tells the nurse that she has noticed that whenever her newborn's head is turned to the right side, she straightens out the arm and leg on the same side and flexes the opposite arm and leg. After finding this on examination, the nurse would tell her that this is: 1. abnormal and is called the atonic neck reflex. 2. normal and should disappear by the first year of life. 3. normal and should disappear between 3 and 4 months of age. 4. abnormal. The baby should be flexing the arm and leg on the right side of his body when the head is turned to the right. 3 By 2 weeks the infant shows the tonic neck reflex when supine and the head is turned to one side (extension of same arm and leg, flexion of opposite arm and leg). The tonic neck reflex disappears between 3 and 4 months of age. During an examination, the nurse knows that Paget's disease would be indicated by which of the following findings? 1. Positive Macewen's sign 2. Premature closure of the sagittal suture 3. Headache, vertigo, tinnitus, and deafness 4. Elongated head with heavy eyebrow ridge 3 Paget's disease occurs more often in males and is characterized by bowed, long bones, sudden fractures, and enlarging skull bones that press on cranial nerves causing symptoms of headache, vertigo, tinnitus, and progressive deafness. During an admission assessment, the nurse notices that a male patient has an enlarged and rather thick skull. The nurse suspects acromegaly and would further assess for: 1. exophthalmos. 2. bowed long bones. 3. coarse facial features. 4. an acorn-shaped cranium. 3 Acromegaly is excessive secretion of growth hormone that creates an enlarged skull and thickened cranial bones. Patients will have elongated heads, massive faces, prominent noses and lower jaws, heavy eyebrow ridges, and coarse facial features. When examining children affected with Down syndrome (trisomy 21), the nurse looks for the possible presence of: 1. ear dysplasia. 2. a long, thin neck. 3. a protruding thin tongue. 4. a narrow and raised nasal bridge. 1 With the chromosomal aberration trisomy 21, also known as Down syndrome, head and face characteristics may include upslanting eyes with inner epicanthal folds, a flat nasal bridge, a small broad flat nose, a protruding thick tongue, ear dysplasia, a short broad neck with webbing, and small hands with a single palmar crease. A patient visits the clinic because he has recently noticed that the left side of his mouth is paralyzed. He states that he cannot raise his eyebrow or whistle. The nurse suspects that he has: 1. Cushing's syndrome. 2. Parkinson's syndrome. 3. Bell's palsy. 4. had a cerebrovascular accident (stroke). 4 With an upper motor neuron lesion (as with CVA) the patient will have paralysis of lower facial muscles, but the upper half of the face is not affected owing to the intact nerve from the unaffected hemisphere. The person is still able to wrinkle the forehead and close the eyes. A woman comes to the clinic and states, "My eyes have gotten so puffy, and my eyebrows and hair have become coarse and dry." The nurse suspects: 1. cachexia. 2. cretinism. 3. myxedema. 4. scleroderma. 3 Myxedema (hypothyroidism) is a deficiency of thyroid hormone that, when severe, causes a nonpitting edema or myxedema. The patient will have a puffy edematous face especially around eyes (periorbital edema), coarse facial features, dry skin, and dry, coarse hair and eyebrows During an examination of a female patient, the nurse notes lymphadenopathy and suspects an acute infection. Acutely infected lymph nodes would be: 1. clumped. 2. unilateral. 3. firm but freely movable. 4. hard and nontender. 3 Acutely infected lymph nodes are bilateral, enlarged, warm, tender, and firm but freely movable. The physician reports that a patient has a tracheal shift. The nurse is aware that this means that the patient's trachea is: 1. pulled to the affected side with systole. 2. pushed to the unaffected side with a tumor. 3. pulled to the unaffected side with plural adhesions. 4. pushed to the affected side with thyroid enlargement. 2 The trachea is pushed to the unaffected side with an aortic aneurysm, a tumor, unilateral thyroid lobe enlargement, and pneumothorax. During an assessment of an infant, the nurse notes that the fontanels are depressed and sunken. The nurse suspects which condition? 1. Rickets 2. Dehydration 3. Mental retardation 4. Increased intracranial pressure 2 Depressed and sunken fontanels occur with dehydration or malnutrition. The nurse is performing an assessment on a 7-year-old child who has symptoms of chronic watery eyes, sneezing, and clear nasal drainage. The nurse notes the presence of a transverse line across the bridge of the nose, dark blue shadows below the eyes, and a double crease on the lower eyelids. These findings are characteristic of: 1. allergies. 2. a sinus infection. 3. nasal congestion. 4. an upper respiratory infection. 1 Chronic allergies often develop chronic facial characteristics. These include blue shadows below the eyes, a double or single crease on the lower eyelids, open-mouth breathing, and a transverse line on the nose. While performing a well-child assessment on a 5-year-old, the nurse notes the presence of palpable, bilateral, cervical, and inguinal lymph nodes. They are approxi- mately 0.5 cm in size, round, mobile, and nontender. The nurse suspects that this: 1. child has chronic allergies. 2. child may have an infection. 3. is a normal finding for a well child of this age. 4. child should be referred for additional evaluation. 3 Palpable lymph nodes are normal in children until puberty when the lymphoid tissue begins to atrophy. Lymph nodes may be up to 1 cm in size in the cervical and inguinal areas, but are discrete, movable, and nontender. The nurse has just completed a lymph assessment on a 60-year-old healthy female patient. The nurse knows that most lymph nodes in healthy adults are normally: 1. shotty. 2. not palpable. 3. large, firm, and fixed to the tissue. 4. rubbery, discrete, and mobile. 2 Most lymph nodes are not palpable in adults. The palpability of lymph nodes decreases with age. During an examination of a patient in her third trimester of pregnancy, the nurse notices that the patient's thyroid gland is slightly enlarged. No enlargement had been noted previously. The nurse suspects that: 1. she has an iodine deficiency. 2. she is exhibiting early signs of goiter. 3. this is a normal finding during pregnancy. 4. further tests are needed for possible thyroid cancer. 3 The thyroid gland enlarges slightly during pregnancy owing to hyperplasia of the tissue and increased vascularity. During an examination, the nurse knows that the best way to palpate the lymph nodes in the neck is described by which statement? 1. Using gentle pressure, palpate with both hands to compare the two sides. 2. Using strong pressure, palpate with both hands to compare the two sides. 3. Gently pinch each node between one's thumb and forefinger and move down the neck muscle. 4. Using the index and middle fingers, gently palpate by applying pressure in a rotating pattern. 1 Use gentle pressure because strong pressure could push the nodes into the neck muscles. It is usually most efficient to palpate with both hands, comparing the two sides symmetrically. During a well-baby checkup, a mother is concerned because her 2-month- old infant cannot hold her head up when she is pulled to a sitting position. Which response by the nurse is appropriate? 1. "Head control is usually achieved by 4 months of age." 2. "You shouldn't be trying to pull your baby up like that until she is older." 3. "This is a concern because head control should be achieved by this time." 4. "This is a concern because it indicates possible nerve damage to the neck muscles." 1 Head control is achieved by 4 months, when the baby can hold the head erect and steady when pulled to a vertical position. During an examination of a 3-year-old child, the nurse notes a bruit over the left temporal area. What should the nurse do? 1. Continue the examination because this is a normal finding for this age. 2. Check for the bruit again in 1 hour. 3. Notify the parents that a bruit has been detected in their child. 4. Stop the examination and notify the physician. 1 Bruits are common in the skull in children under 4 or 5 years of age or in children with anemia. They are systolic or continuous and are heard over the temporal area. During an examination, the nurse finds that a patient's left temporal artery is more tortuous and feels hardened and tender compared with the right temporal artery. The nurse suspects which condition? 1. Crepitation 2. Mastoiditis 3. Temporal arteritis 4. Bell's palsy 3 The artery looks more tortuous and feels hardened and tender with temporal arteritis. During assessment of infants and children, the nurse measures the head circumference and compares the measure to the chest circumference. For each finding listed below, match to the appropriate age. 1. Newborn infant 2. Toddler, age 2 years 3. Child, age 4 years - 1. Head circumference equal to chest circumference. 2. Head circumference greater than chest circumference. 3. Head circumference less than chest circumference. Moving a limb toward the midline of the body is called adduction; abduction is moving a limb away from the midline of the body. Flexion is bending a limb at a joint; extension is straightening a limb at a joint. patient is being assessed for range of joint movement. The nurse asks him to move his arm in toward the center of his body. This movement is called: A) flexion. B) abduction. C) adduction. D) extension. Flexion, or bending a limb at a joint, would be required to move the hand to the mouth. Extension is straightening a limb at a joint. Moving a limb toward the midline of the body is called adduction; abduction is moving a limb away from the midline of the body. patient tells the nurse that she is having a hard time bringing her hand to her mouth when she eats or tries to brush her teeth. The nurse knows that for her to move her hand to her mouth, she must perform which movement? A) flexion. B) abduction. C) adduction. D) extension. Joints are the functional units of the musculoskeletal system because they permit the mobility needed for the activities of daily living. The skeleton (bones) is the framework of the body. - The functional units of the musculoskeletal system are the: A) joints. B) bones. C) muscles. D) tendons. one marrow. The musculoskeletal system functions to encase and protect inner vital organs, support the body, produce red blood cells in the bone marrow, and store minerals. - When reviewing the musculoskeletal system, the nurse recalls that hematopoiesis takes place in the: A) liver. B) spleen. C) kidneys. D) bone marrow. Fibrous bands running directly from one bone to another that strengthen the joint and help prevent movement in undesirable directions are called ligaments. - Fibrous bands running directly from one bone to another that strengthen the joint and help prevent movement in undesirable directions are called: A) bursa. B) tendons. C) cartilage. D) ligaments. Circumduction is defined as moving the arm in a circle around the shoulder. - The nurse notices that a woman in an exercise class is unable to jump rope. The nurse knows that to jump rope, one's shoulder has to be capable of: A) inversion. B) supination. C) protraction. D) circumduction. The articulation of the mandible and the temporal bone is the temporomandibular joint. The other responses are not correct. - The articulation of the mandible and the temporal bone is known as the: A) intervertebral foramen. B) condyle of the mandible. C) temporomandibular joint. D) zygomatic arch of the temporal bone. The temporomandibular joint can be felt in the depression anterior to the tragus of the ear. The other locations are not correct. - To palpate the temporomandibular joint, the nurse's fingers should be placed in the depression _____ of the ear. A) distal to the helix B) proximal to the helix C) anterior to the tragus D) posterior to the tragus There are 7 cervical, 12 thoracic, 5 lumbar, 5 sacral, and 3 to 4 coccygeal vertebrae. - Of the 33 vertebrae in the spinal column, there are: A) 5 lumbar. B) 5 thoracic. C) 7 sacral. D) 12 cervical. An imaginary line connecting the highest point on each iliac crest crosses the fourth lumbar vertebra. n imaginary line connecting the highest point on each iliac crest would cross the _____ vertebra. A) first sacral B) fourth lumbar C) seventh cervical D) twelfth thoracic Intervertebral disks are elastic fibrocartilaginous plates that cushion the spine like shock absorbers and help it move. The vertebral column is the spinal column itself. The nucleus pulposus is located in the center of each disk. The vertebral foramen is the channel, or opening, for the spinal cord in the vertebrae. - The nurse is explaining to a patient that there are "shock absorbers" in his back to cushion the spine and to help it move. The nurse is referring to his: A) vertebral column. B) nucleus pulposus. C) vertebral foramen. D) intervertebral disks. A rotator cuff injury involves the glenohumeral joint, which is enclosed by a group of four powerful muscles and tendons that support and stabilize it. The nucleus pulposus is located in the center of each intervertebral disk. The medial epicondyle is located at the elbow. - The nurse is providing patient education for a man who has been diagnosed with a rotator cuff injury. The nurse knows that a rotator cuff injury involves the: A) nucleus pulposus. B) articular process. C) medial epicondyle. D) glenohumeral joint. The bump of the scapula's acromion process is felt at the very top of the shoulder. The other options are not correct. uring an interview the patient states, "I can feel this bump on the top of both of my shoulders—it doesn't hurt but I am curious about what it might be." The nurse should tell the patient, "That is: A) your subacromial bursa." B) your acromion process." C) your glenohumeral joint." D) the greater tubercle of your humerus." The knee is a hinge joint, permitting flexion and extension of the lower leg on a single plane. The knee is not capable of the other movements listed. - The nurse is checking the range of motion in a patient's knee and knows that the knee is capable of which movement(s)? A) Flexion and extension B) Supination and pronation C) Circumduction D) Inversion and eversion The joint located just above the ring on the finger is the metacarpophalangeal joint. The interphalangeal joint is located distal to the metacarpophalangeal joint. The tarsometatarsal and tibiotalar joints are found in the foot and ankle. See Figure 22-10 for a diagram of the bones and joints of the hand and fingers. patient is visiting the clinic for an evaluation of a swollen, painful knuckle. The nurse notices that the knuckle above his ring on the left hand is swollen and that he is unable to remove his wedding ring. This joint is called the _____ joint. A) interphalangeal B) tarsometatarsal C) metacarpophalangeal D) tibiotalar The nurse is examining the hip area of a patient and palpates a flat depression on the upper, lateral side of the thigh when the patient is standing. The nurse interprets this finding as the: A) ischial tuberosity. B) greater trochanter. C) iliac crest. D) gluteus maximus muscle. The ankle joint is the articulation of the tibia, the fibula, and the: A) talus. B) cuboid. C) calcaneus. D) cuneiform bones. Lengthening occurs at the epiphyses, or growth plates. The other options are not correct. - The nurse is explaining the mechanism of the growth of long bones to a mother of a toddler. Where does lengthening of the bones occur? A) Bursa B) Calcaneus C) Epiphyses D) Tuberosities Lordosis compensates for the enlarging fetus, which would shift the center of balance forward. This shift in balance in turn creates strain on the low back muscles, felt as low back pain during late pregnancy by some women. Scoliosis is lateral curvature of portions of the spine; ankylosis is extreme flexion of the wrist, as seen with severe rheumatoid arthritis; and kyphosis is an enhanced thoracic curvature of the spine. woman who is 8 months pregnant comments that she has noticed a change in posture and is having lower back pain. The nurse tells her that during pregnancy women have a posture shift to compensate for the enlarging fetus. This shift in posture is known as: A) lordosis. B) scoliosis. C) ankylosis. D) kyphosis. Weight-bearing exercises include walking, low-impact aerobics, dancing, or stationary cycling. Swimming is not considered a weight-bearing exercise. The other responses are correct. - The nurse is teaching a class on osteoporosis prevention to a group of postmenopausal woman. A participant shows that she needs more instruction when she states, "I will: A) start swimming to increase my weight-bearing exercise." B) try to stop smoking as soon as possible." C) check with my doctor about taking calcium supplements." D) get a bone-density test soon." A fracture causes sharp pain that increases with movement. The other pains do not occur with a fracture. teenage girl has arrived complaining of pain in her left wrist. She was playing basketball when she fell and landed on her left hand. The nurse examines her hand and would expect a fracture if the girl complains: A) of a dull ache. B) that the pain in her wrist is deep. C) of sharp pain that increases with movement. D) of dull throbbing pain that increases with rest. Rheumatoid arthritis is worse in the morning when arising. Movement increases most joint pain, except in rheumatoid arthritis, in which movement decreases pain. The other options are not correct. patient is complaining of pain in his joints that is worse in the morning, is better after he has moved around for awhile, and then gets worse again if he sits for long periods of time. The nurse should assess for other signs of what problem? A) Tendinitis B) Osteoarthritis C) Rheumatoid arthritis D) Intermittent claudication Crepitation is an audible and palpable crunching or grating that accompanies movement and occurs when articular surfaces in the joints are roughened, as with rheumatoid arthritis. The other options are not correct. patient states, "I can hear a crunching or grating sound when I kneel." She also states that "it is very difficult to get out of bed in the morning because of stiffness and pain in my joints." The nurse should assess for signs of what problem? A) Crepitation B) A bone spur C) A loose tendon D) Fluid in the knee joint Rotator cuff lesions may cause limited range of motion and pain and muscle spasm during abduction, whereas forward flexion stays fairly normal. The other options are not correct. patient is able to flex his right arm forward without difficulty or pain but is unable to abduct his arm because of pain and muscle spasms; the nurse should suspect: A) crepitation. B) rotator cuff lesions. C) dislocated shoulder. D) rheumatoid arthritis. The epicondyles, the head of radius, and tendons are common sites of inflammation and local tenderness, or "tennis elbow." The other locations are not affected. professional tennis player comes into the clinic complaining of a sore elbow. The nurse will assess for tenderness at the: A) olecranon bursa. B) annular ligament. C) base of the radius. D) medial and lateral epicondyle. For the Phalen's test, the nurse should ask the person to hold both hands back to back while flexing the wrists 90 degrees. Acute flexion of the wrist for 60 seconds produces no symptoms in the normal hand. The Phalen's test reproduces numbness and burning in a person with carpal tunnel syndrome. The other actions are not correct for testing for carpal tunnel syndrome. - The nurse suspects that a patient has carpal tunnel syndrome and wants to perform the Phalen's test. To perform this test, the nurse should instruct the patient to: A) dorsiflex the foot. B) plantarflex the foot. C) hold both hands back to back while flexing the wrists 90 degrees for 60 seconds. D) hyperextend the wrists with the palmar surface of both hands touching and wait for 60 seconds. Limitation of abduction of the hip while supine is the most common motion dysfunction found in hip disease. The other options are not correct. n 80-year-old woman is visiting the clinic for a checkup. She states, "I can't walk as much as I used to." The nurse is observing for motor dysfunction in her hip and should have her: A) internally rotate her hip while she is sitting. B) abduct her hip while she is lying on her back. C) adduct her hip while she is lying on her back. D) externally rotate her hip while she is standing. For swelling in the suprapatellar pouch, the bulge sign confirms the presence of fluid. The other options are not correct. - The nurse has completed the musculoskeletal examination of a patient's knee and has found a positive bulge sign. The nurse interprets this finding to indicate: A) irregular bony margins. B) soft tissue swelling in the joint. C) swelling from fluid in the epicondyle. D) swelling from fluid in the suprapatellar pouch. Lateral tilting and sciatic pain with straight leg raising are findings that occur with a herniated nucleus pulposus. The other options are not correct. uring an examination, the nurse asks a patient to bend forward from the waist and notices that the patient has lateral tilting. When his leg is raised straight up, he complains of a pain going down his buttock into his leg. The nurse suspects: A) scoliosis. B) meniscus tear. C) herniated nucleus pulposus. D) spasm of paravertebral muscles. Normally this maneuver feels smooth and has no sound. With a positive Ortolani's sign, the nurse will feel and hear a "clunk" as the head of the femur pops back into place. A positive Ortolani's sign reflects hip instability. The Allis test also tests for hip dislocation, but is done by comparing leg lengths. - The nurse is examining a 3-month-old infant. While holding the thumbs on the infant's inner mid thighs and the fingers outside on the hips, touching the greater trochanter, the nurse adducts the legs until the nurse's thumbs touch and then abducts the legs until the infant's knees touch the table. The nurse does not notice any "clunking" sounds and is confident to record a: A) positive Allis test. B) negative Allis test. C) positive Ortolani's sign. D) negative Ortolani's sign. Polydactyly is the presence of extra fingers or toes. Syndactyly is webbing between adjacent fingers or toes. The other terms are not correct. uring a neonatal examination, the nurse notices that the newborn infant has six toes. This finding is documented as: A) unidactyly. B) syndactyly. C) polydactyly. D) multidactyly. For a fractured clavicle, the nurse should observe for limited arm range of motion and unilateral response to the Moro's reflex. The other tests are not appropriate for this problem. mother brings her newborn baby boy in for a checkup; she tells the nurse that he doesn't seem to be moving his right arm as much as his left and that he seems to have pain when she lifts him up under the arms. The nurse suspects a fractured clavicle and would observe for: A) a negative Allis test. B) a positive Ortolani's sign. C) limited range of motion during the Moro's reflex. D) limited range of motion during Lasègue's test Acute gout occurs primarily in men over 40 years of age. Clinical findings consist of redness, swelling, heat, and extreme tenderness. Gout is a metabolic disorder of disturbed purine metabolism, associated with elevated serum uric acid. See Table 22-1 for descriptions of the other terms. 40-year-old man has come into the clinic with complaints of "extreme tenderness in my toes." The nurse notices that his toes are slightly swollen, reddened, and warm to the touch. His complaints would suggest: A) osteoporosis. B) acute gout. C) ankylosing spondylitis. D) degenerative joint disease. Dislocated shoulder occurs with trauma involving abduction, extension, and external rotation (e.g., falling on an outstretched arm or diving into a pool). See Table 22-2 for a description of the other conditions. young swimmer comes to the sports clinic complaining of a very sore shoulder. He was running at the pool, slipped on some wet concrete, and tried to catch himself with his outstretched hand. He landed on his outstretched hand and has not been able to move his shoulder since then. The nurse suspects: A) joint effusion. B) tear of rotator cuff. C) adhesive capsulitis. D) dislocated shoulder. Subcutaneous nodules are raised, firm, and nontender and occur with rheumatoid arthritis in the olecranon bursa and along the extensor surface of the ulna. See Table 22-3 for a description of the other conditions. 68-year-old woman has come in for an assessment of her rheumatoid arthritis, and the nurse notices raised, firm, nontender nodules at the olecranon bursa and along the ulna. These nodules are most commonly diagnosed as: A) epicondylitis. B) gouty arthritis. C) olecranon bursitis. D) subcutaneous nodules. Fingers drift to the ulnar side because of stretching of the articular capsule and muscle imbalance caused by chronic rheumatoid arthritis. Radial drift is not seen. See Table 22-4 for descriptions of swan neck deformity and Dupuytren's contracture. woman who has had rheumatoid arthritis for years is starting to notice that her fingers are drifting to the side. The nurse knows that this condition is commonly referred to as: A) radial drift. B) ulnar deviation. C) swan neck deformity. D) Dupuytren's contracture. Changes in the fingers caused by chronic rheumatoid arthritis include swan neck and boutonniere deformities. Heberden's nodes and Bouchard's nodules are associated with osteoarthritis. Dupuytren's contractures occur because of chronic hyperplasia of the palmar fascia and causes contractures of the digits (see Table 22-4). patient who has had rheumatoid arthritis for years comes to the clinic to ask about changes in her fingers. The nurse will assess for signs of what problems? A) Heberden's nodes B) Bouchard's nodules C) Swan neck deformities D) Dupuytren's contractures Functional scoliosis is flexible; it is apparent with standing and disappears with forward bending. Structural scoliosis is fixed; the curvature shows both when standing and when bending forward. See Table 22-7 for description of herniated nucleus pulposus. These findings are not indicative of a dislocated hip. patient's annual physical examination reveals a lateral curvature of the thoracic and lumbar segments of his spine; however, this curvature disappears with forward bending. The nurse knows that this abnormality of the spine is called: A) structural scoliosis. B) functional scoliosis. C) herniated nucleus pulposus. D) dislocated hip. Osgood-Schlatter disease is painful swelling of the tibial tubercle just below the knee. It is most likely due to repeated stress on the patellar tendon. It is usually self-limited, occurring during rapid growth and most often in males. The symptoms resolve with rest. The other responses are not appropriate. 14-year-old boy who has been diagnosed with Osgood-Schlatter disease reports painful swelling just below the knee for the past 5 months. Which response by the nurse is appropriate? A) "If these symptoms persist, you may need arthroscopic surgery." B) "You are experiencing degeneration of your knee, which may not resolve." C) "Your disease is due to repeated stress on the patellar tendon. It is usually self-limited, and your symptoms should resolve with rest." D) "Increasing your activity and performing knee-strengthening exercises will help to decrease the inflammation and maintain mobility in the knee." Complete range of motion against gravity is normal muscle strength and is recorded as Grade 5 muscle strength. - When assessing muscle strength, the nurse observes that a patient has complete range of motion against gravity with full resistance. What Grade should the nurse record using a 0 to 5 point scale? A) 2 B) 3 C) 4 D) 5 Finding one knee significantly lower than the other is a positive Allis sign and suggests hip dislocation. Normally the tops of the knees are at the same elevation. The other statements are not correct. - The nurse is examining a 6-month-old infant and places the infant's feet flat on the table and flexes his knees up. The nurse notes that the right knee is significantly lower than the left. Which of these statements is true of this finding? A) This is a positive Allis sign and suggests hip dislocation. B) The infant probably has a dislocated patella on the right. C) This is a normal finding for the Allis test for an infant of this age. D) The infant should return to the clinic in 2 weeks to see if this has changed. An infant who starts to "slip" between the nurse's hands shows weakness of the shoulder muscles. An infant with normal muscle strength wedges securely between the nurse's hands. The other responses are not correct. - The nurse is assessing a 1-week-old infant and is testing his muscle strength. The nurse lifts the infant with hands under the axillae and notices that the infant starts to "slip" between the hands. The nurse should: A) suspect a fractured clavicle. B) suspect that the infant may have a deformity of the spine. C) suspect that the infant may have weakness of the shoulder muscles. D) consider this a normal finding because the musculature of an infant this age is undeveloped. Unequal gluteal folds may accompany hip dislocation after 2 to 3 months of age, but some asymmetry may occur in healthy children. Further assessment is needed. The other responses are not correct. - The nurse is examining a 2-month-old infant and notices asymmetry of the infant's gluteal folds. The nurse should assess for other signs of what disorder? A) Fractured clavicle B) Down syndrome C) Spina bifida D) Hip dislocation allottement - The nurse should use which test to check for large amounts of fluid around the patella? A) Ballottement B) Tinel sign C) Phalen's test D) McMurray's test Genu valgum is also known as "knock knees" and is present when there is more than 2.5 cm between the medial malleoli when the knees are together. patient tells the nurse that "all my life I've been called 'knock knees.'" The nurse knows that another term for "knock knees" is: A) genu varum. B) genu valgum. C) pes planus. D) metatarsus adductus. A ganglionic cyst is a common benign tumor; it does not become malignant, and it does not need to be drained. It is not caused by chronic repetitive motion injury patient has been diagnosed with a ganglion cyst over the dorsum of his left wrist. He asks the nurse, "What is this thing?" The nurse's best answer would be, "It is: A) a common benign tumor." B) a tumor that will have to be watched because it may turn malignant." C) caused by chronic repetitive motion injury." D) a skin infection that will need to be drained." Tophi are collections of sodium urate crystals resulting from chronic gout in and around the joint that cause extreme swelling and joint deformity. They appear as hard, painless nodules (tophi) over the metatarsophalangeal joint of the first toe and they sometimes burst with a chalky discharge (See Table 22-6). See Table 22-6 for descriptions of the other conditions. man who has had gout for several years comes to the clinic with a problem with his toe. On examination, the nurse notices the presence of hard, painless nodules over the great toe; one has burst open with a chalky discharge. This finding is known as: A) a callus. B) a plantar wart. C) a bunion. D) tophi. The musculoskeletal assessment should be done in an orderly approach, head to toe, proximal to distal, from the midline outward. The other options are not correct. - When performing a musculoskeletal assessment, the nurse knows that the correct approach for the examination should be: A) proximal to distal. B) distal to proximal. C) posterior to anterior. D) anterior to posterior. ANS: Asymmetric joint involvement Pain with motion of affected joints Affected joints are swollen with hard, bony protuberances In osteoarthritis, asymmetric joint involvement commonly affects hands, knees, hips, and lumbar and cervical segments of the spine. Affected joints have stiffness, swelling with hard bony protuberances, pain with motion, and limitation of motion. The other options reflect signs of rheumatoid arthritis. - The nurse is assessing the joints of a woman who has stated, "I have a long family history of arthritis, and my joints hurt." The nurse suspects that she has osteoarthritis. Which of these are symptoms of osteoarthritis? Select all that apply. ANS: flex the hip. The ischial tuberosity lies under the gluteus maximus muscle and is palpable when the hip is flexed. - The nurse is assessing a patient's ischial tuberosity. To palpate the ischial tuberosity, the nurse knows that it is best to have the patient: ANS: of the shortening of the vertebral column. Postural changes are evident with aging; decreased height is most noticeable and is due to shortening of the vertebral column. Long bones do not shorten with age. Intervertebral disks actually get thinner with age. Subcutaneous fat is not lost but is redistributed to the abdomen and hips. n 85-year-old patient comments during his annual physical that he seems to be getting shorter as he ages. The nurse should explain that decreased height occurs with aging because: ANS: loss of bone density. After age 40, loss of bone matrix (resorption) occurs more rapidly than new bone formation. The net effect is a gradual loss of bone density, or osteoporosis. The other options are not correct. patient has been diagnosed with osteoporosis and asks the nurse, "What is osteoporosis?" The nurse explains to the patient that osteoporosis is defined as: The portion of the ear that consists of movable cartilage and skin is called the: 1. auricle. 2. concha. 3. outer meatus. 4. mastoid process. 1 The external ear is called the auricle or pinna and consists of movable cartilage and skin. The nurse is examining a patient's ears and notices cerumen in the external canal. Which of the following statements about cerumen is correct? 1. Sticky honey-colored cerumen is a sign of infection. 2. The presence of cerumen is indicative of poor hygiene. 3. The purpose of cerumen is to protect and lubricate the ear. 4. Cerumen is necessary for transmitting sound through the auditory canal. 3 The ear is lined with glands that secrete cerumen, a yellow waxy material that lubricates and protects the ear. When examining the ear with an otoscope, the nurse remembers that the tympanic membrane should appear: 1. light pink with a slight bulge. 2. pearly gray and slightly concave. 3. pulled in at the base of the cone of light. 4. whitish with a small fleck of light in the superior portion. 2 It is a translucent membrane with a pearly gray color and a prominent cone of light in the anteroinferior quadrant, which is the reflection of the otoscope light. The drum is oval and slightly concave, pulled in at its center by one of the middle ear ossicles, the malleus. Which of the following statements concerning the eustachian tube is true? 1. It is responsible for the production of cerumen. 2. It remains open except when swallowing or yawning. 3. It allows passage of air between the middle and outer ear. 4. It helps equalize air pressure on both sides of the tympanic membrane. 4 The eustachian tube allows equalization of air pressure on each side of the tympanic membrane so that the membrane does not rupture (e.g., during altitude changes in an airplane). The tube is normally closed, but it opens with swallowing or yawning. A patient with a middle ear infection asks the nurse, "What does the middle ear do?" The nurse responds by telling the patient that the middle ear functions to: 1. maintain balance. 2. interpret sounds as they enter the ear. 3. conduct vibrations of sounds to the inner ear. 4. increase amplitude of sound for the inner ear to function. 3 Among its other functions, the middle ear conducts sound vibrations from the outer ear to the central hearing apparatus in the inner ear. Which of the following cranial nerves is responsible for conducting nerve impulses to the brain from the organ of Corti? 1. CN I 2. CN III 3. CN VIII 4. CN XI 3 The nerve impulses are conducted by the auditory portion of CN VIII to the brain. Which of the following statements is true concerning air conduction? 1. It is the most efficient pathway for hearing. 2. It is caused by the vibrations of bones in the skull. 3. The amplitude of sound determines the pitch that is heard. 4. A loss of air conduction is called a conductive hearing loss. 1 The normal pathway of hearing is air conduction, and it is the most efficient. A patient has been shown to have a sensorineural hearing loss. During the assessment, it would be important for the nurse to: 1. speak loudly so he can hear the questions. 2. assess for middle ear infection as a possible cause. 3. ask the patient what medications he is currently taking. 4. look for the source of the obstruction in the external ear. 3 A simple increase in amplitude may not enable the person to understand words. Sensorineural hearing loss may be caused by presbycusis, a gradual nerve degenera- tion that occurs with aging and by ototoxic drugs, which affect the hair cells in the cochlea. During an interview, the patient states he has the sensation that "everything around him is spinning." The nurse recognizes that the portion of the ear responsible for this sensation is: 1. the cochlea. 2. cranial nerve VIII. 3. the organ of Corti. 4. the bony labyrinth. 4 If the labyrinth ever becomes inflamed, it feeds the wrong information to the brain, creating a staggering gait and a strong, spinning, whirling sensation called vertigo. A patient in her first trimester of pregnancy is diagnosed with rubella. The nurse recognizes that the significance of this in relation to the infant's hearing is which of the following? 1. Rubella may affect the mother's hearing but not the infant's. 2. Rubella can damage the infant's organ of Corti, which will impair hearing. 3. Rubella is only dangerous to the infant in the second trimester of pregnancy. 4. Rubella can impair the development of CN VIII and thus affect hearing. 2 If maternal rubella infection occurs during the first trimester, it can damage the organ of Corti and impair hearing. The mother of a 2-year-old is concerned because her son has had three ear infections in the past year. What would be an appropriate response by the nurse? 1. "It is unusual for a small child to have frequent ear infections unless there is something else wrong." 2. "We need to check the immune system of your son to see why he is having so many ear infections." 3. "Ear infections are not uncommon in infants and toddlers because they tend to have more cerumen in the external ear." 4. "Your son's eustachian tube is shorter and wider than yours because of his age, which allows for infections to develop more easily." 4 The infant's eustachian tube is relatively shorter and wider, and its position is more horizontal than the adult's, so it is easier for pathogens from the nasopharynx to migrate through to the middle ear. A 31-year-old patient tells the nurse that he has noticed a progressive loss in his hearing. He says that it does seem to help when people speak louder or if he turns up the volume. The most likely cause of his hearing loss is: 1. otosclerosis. 2. presbycusis. 3. trauma to the bones. 4. frequent ear infections. 1 Otosclerosis is a common cause of conductive hearing loss in young adults between the ages of 20 and 40 years. A 70-year-old patient tells the nurse that he has noticed that he is having trouble hearing, especially in large groups. He says he "can't always tell where the sound is coming from" and the words often sound "mixed up." What might the nurse suspect as the cause for this change? 1. Atrophy of the apocrine glands 2. Cilia becoming coarse and stiff 3. Nerve degeneration in the inner ear 4. Scarring of the tympanic membrane 3 Presbycusis is a type of hearing loss that occurs with aging, even in people living in a quiet environment. It is a gradual sensorineural loss caused by nerve degeneration in the inner ear or auditory nerve. This makes words sound garbled. The ability to localize sound is impaired also. This communication dysfunction is accentuated when background noise is present. During an assessment of a 20-year-old Asian patient, the nurse notices that he has dry, flaky cerumen in his canal. What is the significance of this finding? 1. This is probably the result of lesions from eczema in his ear. 2. This represents poor hygiene. 3. This is a normal finding and no further follow-up is necessary. 4. This could be indicative of change in cilia; the nurse should assess for conductive hearing loss. 3 Asians and American Indians are more likely to have dry cerumen, whereas blacks and whites usually have wet cerumen. The nurse is taking the history of a patient who may have a perforated eardrum. What would be an important question in this situation? 1. "Do you ever notice ringing or crackling in your ears?" 2. "When was the last time you had your hearing checked?" 3. "Have you ever been told you have any type of hearing loss?" 4. "Was there any relationship between the ear pain and the discharge you mentioned?" 4 Typically with perforation, ear pain occurs first, stopping with a popping sensation, and then drainage occurs. A 31-year-old patient tells the nurse that he has noticed pain in his left ear when people speak loudly to him. The nurse knows that this finding: 1. is normal for people of that age. 2. is a characteristic of recruitment. 3. may indicate a middle ear infection. 4. indicates that the patient has a cerumen impaction. 2 Recruitment is a marked loss occurring when sound is at low intensity; sound actually may become painful when repeated at a louder volume. While discussing the history of a 6-month-old infant, the mother tells the nurse that she took a great deal of aspirin while she was pregnant. What question would the nurse want to include in the history? 1. "Does your baby seem to startle with loud noise?" 2. "Has the baby had any surgeries on the ears?" 3. "Have you noticed any drainage from her ears?" 4. "How many ear infections has your baby had since birth?" 1 Children at risk for hearing deficit include those exposed in utero to a variety of conditions, such as maternal rubella, or to maternal ototoxic drugs. The nurse is performing an otoscopic examination on an adult. Which of the following is true? 1. Tilt the person's head forward during the exam. 2. Once the speculum is in the ear, release the traction. 3. Pull the pinna up and back before inserting the speculum. 4. Use the smallest speculum to decrease the amount of discomfort. 3 Pull the pinna up and back on an adult or older child. This helps straighten the S- shape of the canal. The nurse is assessing a 16-year-old patient with head injuries from a recent motor vehicle accident. Which of the following statements indicates the most important reason for assessing for any drainage from the canal? 1. If the drum has ruptured, there will be purulent drainage. 2. Bloody or clear watery drainage can indicate a basal skull fracture. 3. The auditory canal many be occluded from increased cerumen. 4. There may be occlusion of the canal caused by foreign bodies from the accident. 2 Frank blood or clear watery drainage (cerebrospinal leak) after trauma suggests a basal skull fracture and warrants immediate referral. A colleague is assessing an 80-year-old patient who has ear pain and asks him to hold his nose and swallow. The nurse knows that which of the following is true concerning this technique? 1. This should not be used in an 80-year-old patient. 2. This technique is helpful in assessing for otitis media. 3. This is especially useful in assessing a patient with an upper respiratory infection. 4. This will cause the eardrum to bulge slightly and make landmarks more visible. 1 The eardrum is flat, slightly pulled in at the center, and flutters when the person performs the Valsalva maneuver or holds the nose and swallows (insufflation). One may elicit these maneuvers to assess drum mobility. Avoid these with an aging person because they may disrupt equilibrium. In performing a voice test to assess hearing, which of the following would the nurse do? 1. Shield the lips so that the sound is muffled. 2. Whisper two-syllable words and ask the patient to repeat them. 3. Ask the patient to place his finger in his ear to occlude outside noise. 4. Stand about 4 feet away to ensure that the patient can really hear at this distance. 2 With your head 30 to 60 cm (1 to 2 ft) from the person's ear, exhale and whisper slowly some two-syllable words such as Tuesday, armchair, baseball, or fourteen. Normally, the person repeats each word correctly after you say it. In performing an examination of a 3-year-old with a suspected ear infection, the nurse would: 1. omit the otoscopic exam if the child has a fever. 2. pull the ear up and back before inserting the speculum. 3. ask the mother to leave the room while examining the child. 4. perform the otoscopic examination at the end of the assessment. 4 In addition to its place in the complete examination, eardrum assessment is manda- tory for any infant or child requiring care for illness or fever. For the infant or young child, the timing of the otoscopic examination is best toward the end of the complete examination. Which of the following would be true regarding otoscopic examination of a newborn? 1. Immobility of the drum is a normal finding. 2. An injected membrane would indicate infection. 3. The normal membrane may appear thick and opaque. 4. The appearance of the membrane is identical to that of an adult. 3 During the first few days, the tympanic membrane often looks thickened and opaque. It may look "injected" and have a mild redness from increased vascularity. The nurse assesses the hearing of a 7-month-old by clapping hands. What is the expected response? 1. The infant turns the head to localize sound. 2. No obvious response to noise 3. A startle and acoustic blink reflex 4. The infant stops movement and appears to listen. 1 With a loud sudden noise, you should note these responses: 6 to 8 months—infant turns head to localize sound, responds to own name. During the ear examination of an 80-year-old patient, which of the following would be a normal finding? 1. A high-tone frequency loss 2. Increased elasticity of the pinna 3. A thin, translucent membrane 4. A shiny, pink tympanic membrane 1 A high-tone frequency hearing loss is apparent for those affected with presbycusis, the hearing loss that occurs with aging. An assessment of a 23-year-old patient reveals the following: an auricle that is tender and reddish-blue in color with small vesicles. Additional information the nurse would need to know includes which of the following? 1. Any change in the ability to hear 2. Any recent drainage from the ear 3. Recent history of trauma to the ear 4. Any prolonged exposure to extreme cold 4 Frostbite causes reddish-blue discoloration and swelling of the auricle after exposure to extreme cold. Vesicles or bullae may develop, and the person feels pain and tenderness. While performing the otoscopic exam of a 3-year-old boy who has been pulling on his left ear, the nurse finds that his left tympanic membrane is bright red and the light reflex is not visible. The most likely cause is: 1. fungal infection. 2. acute otitis media. 3. rupture of the drum. 4. blood behind the drum. 2 Absent or distorted light reflex and a bright red color of the eardrum are indicative of acute otitis media. The mother of a 2-year-old is concerned about the upcoming placement of tympanostomy tubes in her son's ears. The nurse would include which of the following in the teaching plan? 1. The tubes are placed in the inner ear. 2. The tubes are used in children with sensorineural loss. 3. The tubes are permanently inserted during a surgical procedure. 4. The purpose of the tubes is to decrease the pressure and allow for drainage. 4 Polyethylene tubes are inserted surgically into the eardrum to relieve middle ear pressure and promote drainage of chronic or recurrent middle ear infections. Tubes extrude spontaneously in 6 months to 1 year. 29. During a hearing assessment the nurse finds that sound lateralizes to the patient's left ear with the Weber test. What can the nurse conclude from this? 1. The patient has a conductive hearing loss in the right ear. 2. Lateralization is a normal finding with the Weber test. 3. The patient could have either a sensorineural or a conductive loss. 4. A mistake has occurred; the test must be repeated. 3 It is necessary to perform the Weber and Rinne tests to determine the type of loss. With conductive loss, sound lateralizes to the "poorer" ear owing to background room noise. With sensorineural loss, sound lateralizes to the "better" ear or unaffected ear. A patient states that she is unable to hear well with her left ear. The Weber test shows lateralization to the right ear. Rinne has AC>BC with ratio of 2:1 in both ears, left-AC 10 sec and BC 5 sec, right-AC 30 sec and BC 15 sec. What would be the interpretation of these results? 1. The patient may have sensorineural loss. 2. The test results are reflective of normal hearing. 3. Conduction of sound through bones is impaired. 4. These results make no sense, so further tests should be done. 1 With sensorineural loss, sound lateralizes to "better" ear or unaffected ear. Normal ratio of AC>BC is intact but is reduced overall. That is, the person hears poorly both ways. In an individual with otitis externa, which of the following signs would the nurse expect to find on assessment? 1. Rhinorrhea 2. Periorbital edema 3. Pain over the maxillary sinuses 4. Enlarged superficial cervical nodes 4 The lymphatic drainage of the external ear flows to the parotid, mastoid, and superficial cervical nodes. The signs are severe swelling of the canal, inflammation, and tenderness. When performing an otoscopic examination of a 5-year-old child with a history of chronic ear infections, the nurse sees that his right tympanic membrane is amber- yellow in color and there are air bubbles behind the tympanic membrane. The child reports occasional hearing loss and a popping sound with swallowing. The preliminary analysis based on this information is that: 1. this is most likely a serous otitis media. 2. the child has an acute purulent otitis media. 3. there is evidence of a resolving cholesteatoma. 4. the child is experiencing the early stages of perforation. 1 An amber-yellow color to the tympanic membrane suggests serum in the middle ear. Often an air/fluid level or bubbles behind the tympanic membrane are visible. The patient may have feelings of fullness, transient hearing loss, and a popping sound with swallowing. The nurse is performing an assessment on a 65-year-old male. He reports a crusty nodule behind the pinna. It bleeds intermittently and has not healed over the past 6 months. On physical assessment, the nurse finds an ulcerated crusted nodule with an indurated base. The preliminary analysis in this situation is that this: 1. is most likely a benign sebaceous cyst. 2. is most likely a Darwin's tubercle and is not significant. 3. could be a potential carcinoma and should be referred. 4. is a tophus, which is common in the elderly and is a sign of gout. 3 An ulcerated crusted nodule with an indurated base that fails to heal is characteristic of a carcinoma. These lesions fail to heal and bleed intermittently. Individuals with such symptoms should be referred for a biopsy. The nurse suspects that a patient has otitis media. Early signs of otitis media include which of the following findings of the tympanic membrane? 1. Red and bulging 2. Hypomobility 3. Retraction with landmarks clearly visible 4. Flat, slightly pulled in at the center, and moves with insufflation 2 An early sign of otitis media is hypomobility of the tympanic membrane. The nurse is performing a middle ear assessment on a 15-year-old patient who has a history of chronic ear infections. When examining the right tympanic membrane, the nurse sees the presence of dense white patches. The tympanic membrane is otherwise unremarkable. It is pearly, with the light reflex at 5 o'clock and landmarks visible. The nurse should: 1. refer the patient for the possibility of a fungal infection. 2. know that these are scars caused from frequent ear infections. 3. consider that these findings may represent the presence of blood in the middle ear. 4. be concerned about the ability to hear because of this abnormality on the tympanic membrane. 2 Dense white patches on the tympanic membrane are sequelae of repeated ear infections. They do not necessarily affect hearing. The nurse is preparing to do an otoscopic examination on a 2-year-old child. Which of the following reflects correct procedure? 1. Pull the pinna down. 2. Pull the pinna up and back. 3. Tilt the child's head slightly toward the examiner. 4. Have the child touch his chin to his chest. 1 Pull the pinna down on an infant and a child under 3 years of age. Which of the following is a risk factor for ear infections in young children? 1. Family history 2. Air conditioning 3. Excessive cerumen 4. Secondhand cigarette smoke 4 Passive or second hand smoke is a risk factor for ear infections. During an otoscopic examination, the nurse notes an area of black and white dots on the tympanic membrane and ear canal wall. What does this finding suggest? 1. Malignancy 2. Viral infection 3. Blood in the middle ear 4. Yeast or fungal infection 4 A colony of black or white dots on the drum or canal wall suggests a yeast or fungal infection (otomycosis). A 17-year-old student is a swimmer on her high school's swim team. She has had three bouts of otitis externa so far this season and wants to know what to do to prevent it. The nurse instructs her to: 1. use a cotton-tipped swab to dry the ear canals thoroughly after each swim. 2. use rubbing alcohol or 2% acetic acid eardrops after every swim. 3. irrigate the ears with warm water and a bulb syringe after each swim. 4. rinse the ears with a warmed solution of mineral oil and hydrogen peroxide. 2 With otitis externa (swimmer's ear), swimming causes the external canal to become waterlogged and swell; skinfolds are set up for infection. Prevent by using rubbing alcohol or 2% acetic acid ear drops after every swim. During an examination, the patient states he is hearing a buzzing sound and says that it is "driving me crazy!" The nurse recognizes that this symptom is: 1. vertigo. 2. pruritus. 3. tinnitus. 4. cholesteatoma. 3 Tinnitus is a sound that comes from within a person; it can be a ringing, crackling, or buzzing sound. It accompanies some hearing or ear disorders. The nurse is testing the hearing of a 78-year-old man and keeps in mind the changes in hearing that occur with aging, such as: (Select all that apply.) 1. Hearing loss related to aging begins in the mid 40s. 2. The progression is slow. 3. The aging person has low-frequency tone loss. 4. The aging person may find it harder to hear consonants than vowels. 5. Sounds may be garbled and difficult to localize. 6. Hearing loss reflects nerve degeneration of the middle ear. 2, 4, 5 Presbycusis is a type of hearing loss that occurs with aging; it is a gradual sorineural loss caused by nerve degeneration in the inner ear or auditory nerve. Its onset usually occurs in the fifth decade, and then it slowly progresses. The person first notices a high-frequency tone loss; it is harder to hear consonants (high-pitched components of speech) than vowels. This makes words sound garbled. The ability to localize sound is impaired also. Which of the following statements is true regarding the internal structures of the breast? The breast is: a. Primarily muscle with very little fibrous tissue. b. Fibrous, glandular, and adipose tissues. c. Primarily milk ducts, known as lactiferous ducts. d. Glandular tissue, which supports the breast by attaching to the chest wall. omposed of fibrous, glandular, and adipose tissue. The breast is composed of glandular tissue, fibrous tissue (including the suspensory ligaments), and adipose tissue. In performing a breast examination, the nurse knows that it is especially important to examine the upper outer quadrant of the breast. The reason for this is that the upper outer quadrant is: a. The largest quadrant of the breast. b. The location of most breast tumors. c. Where most of the suspensory ligaments attach. d. More prone to injury and calcifications than other locations in the breast. the location of most breast tumors. The upper outer quadrant is the site of most breast tumors. In the upper outer quadrant, the nurse should notice the axillary tail of Spence, the cone-shaped breast tissue that projects up into the axilla, close to the pectoral group of axillary lymph nodes. In performing an assessment of a woman's axillary lymph system, the nurse should assess which of these nodes? a. Central, axillary, lateral, and sternal b. Pectoral, lateral, anterior, and sternal c. Central, lateral, pectoral, and subscapular d. Lateral, pectoral, axillary, and suprascapular entral, lateral, pectoral, and subscapular nodes The breast has extensive lymphatic drainage. Four groups of axillary nodes are present: (1) central, (2) pectoral (anterior), (3) subscapular (posterior), and (4) lateral. If a patient reports a recent breast infection, then the nurse should expect to find _____ node enlargement. a. Nonspecific b. Ipsilateral axillary c. Contralateral axillary d. Inguinal and cervical ipsilateral axillary The breast has extensive lymphatic drainage. Most of the lymph, more than 75%, drains into the ipsilateral, or same side, axillary nodes. A 9-year-old girl is in the clinic for a sports physical. After some initial shyness she finally asks, "Am I normal? I don't seem to need a bra yet, but I have some friends who do. What if I never get breasts?" The nurse's best response would be: a. "Don't worry, you still have plenty of time to develop." b. "I know just how you feel, I was a late bloomer myself. Just be patient, and they will grow." c. "You will probably get your periods before you notice any significant growth in your breasts." d. "I understand that it is hard to feel different from your friends. Breasts usually develop between 8 and 10 years of age." "I understand that it is hard to feel different from your friends. Breasts usually develop between 8 and 10 years of age." Adolescent breast development usually begins between 8 and 10 years of age. The nurse should not belittle the girl's feelings by using statements like "don't worry" or by sharing personal experiences. The beginning of breast development precedes menarche by about 2 years. A patient contacts the office and tells the nurse that she is worried about her 10-year-old daughter having breast cancer. She describes a unilateral enlargement of the right breast with associated tenderness. She is worried because the left breast is not enlarged. What would be the nurse's best response? a. Breast development is usually fairly symmetric and that the daughter should be examined right away. b. She should bring in her daughter right away because breast cancer is fairly common in preadolescent girls. c. Although an examination of her daughter would rule out a problem, her breast development is most likely normal. d. It is unusual for breasts that are first developing to feel tender because they haven't developed much fibrous tissue. Tell the mother that, although an examination of her daughter would rule out a problem, it is most likely normal breast development. Occasionally one breast may grow faster than the other, producing a temporary asymmetry. This may cause some distress; reassurance is necessary. Tenderness is common also. A 14-year-old girl is anxious about not having reached menarche. When taking the history, the nurse should ascertain which of the following? The age: a. The girl began to develop breasts. b. Her mother developed breasts. c. She began to develop pubic hair. d. She began to develop axillary hair. she began to develop breasts Full development from stage 2 to stage 5 takes an average of 3 years, although the range is 1.5 to 6 years. Pubic hair develops during this time, and axillary hair appears 2 years after the onset of pubic hair. The beginning of breast development precedes menarche by about 2 years. Menarche occurs in breast development stage 3 or 4, usually just after the peak of the adolescent growth spurt, which occurs around age 12 years. See Figure 17-6. A woman is in the family planning clinic seeking birth control information. She states that her breasts "change all month long" and that she is worried that this is unusual. What is the nurse's best response? a. Continual changes in her breasts are unusual. The breasts of nonpregnant women usually stay pretty much the same all month long. b. Breast changes in response to stress are very common and that she should assess her life for stressful events. c. Because of the changing hormones during the monthly menstrual cycle, cyclic breast changes are common. d. Breast changes normally occur only during pregnancy and that a pregnancy test is needed at this time. Tell her that, because of the changing hormones during the monthly menstrual cycle, cyclic breast changes are common. Breasts of the nonpregnant woman change with the ebb and flow of hormones during the monthly menstrual cycle. During the 3 to 4 days before menstruation, the breasts feel full, tight, heavy, and occasionally sore. The breast volume is smallest on days 4 to 7 of the menstrual cycle. A woman has just learned that she is pregnant. What are some things the nurse should teach her about her breasts? a. She can expect her areolae to become larger and darker in color. b. Breasts may begin secreting milk after the fourth month of pregnancy. c. She should inspect her breasts for visible veins and immediately report these. d. During pregnancy, breast changes are fairly uncommon; most of the changes occur after the birth. She can expect her areolae to become larger and darker in color. The areolae become larger and grow a darker brown as pregnancy progresses, and the tubercles become more prominent. (The brown color fades after lactation, but the areolae never return to the original color). A venous pattern is prominent over the skin surface and does not need to be reported as it is an expected finding. After the fourth month, colostrum, a thick, yellow fluid (precursor to milk) may be expressed from the breasts. The nurse is teaching a pregnant woman about breast milk. Which statement by the nurse is correct? a. "Your breast milk is immediately present after the delivery of your baby." b. "Breast milk is rich in protein and sugars (lactose) but has very little fat." c. "The colostrum, which is present right after birth, does not contain the same nutrients as breast milk." d. "You may notice a thick, yellow fluid expressed from your breasts as early as the fourth month of pregnancy." "You may notice a thick, yellow fluid expressed from your breasts as early as the fourth month of pregnancy." After the fourth month, colostrum may be expressed. This thick yellow fluid is the precursor of milk, and it contains the same amount of protein and lactose but practically no fat. The breasts produce colostrum for the first few days after delivery. It is rich with antibodies that protect the newborn against infection, so breastfeeding is important. A 65-year-old patient remarks that she just can't believe that her breasts sag so much. She states it must be from lack of exercise. What explanation should the nurse offer her? a. Only women with large breasts experience sagging. b. Sagging is usually due to decreased muscle mass within the breast. c. A diet that is high in protein will help maintain muscle mass, which keeps the breasts from sagging. d. The glandular and fat tissue atrophies, causing breast size and elasticity to diminish, resulting in breasts that sag. fter menopause, the glandular and fat tissue atrophies, causing breast size and elasticity to diminish, resulting in breasts that sag. After menopause, the glandular tissue atrophies and is replaced with connective tissue. The fat envelope atrophies also, beginning in the middle years and becoming marked in the eighth and ninth decades. These changes decrease breast size and elasticity, so the breasts droop and sag, looking flattened and flabby. In examining a 70-year-old male patient, the nurse notices that he has bilateral gynecomastia. Which of the following describes the nurse's best course of action? a. Recommend that he make an appointment with his physician for a mammogram. b. Ignore it. Benign breast enlargement in men is not unusual. c. Explain that this condition may be the result of hormonal changes, and recommend that he see his physician. d. Explain that gynecomastia in men is usually associated with prostate enlargement and recommend that he be thoroughly screened. Explain that this condition may be the result of hormonal changes and recommend that he see his physician. Gynecomastia may reappear in the aging male and may be due to testosterone deficiency. During an examination of a 7-year-old girl, the nurse notices that the girl is showing breast budding. What should the nurse do next? a. Ask the young girl if her periods have started. b. Assess the girl's weight and body mass index (BMI). c. Ask the girl's mother at what age she started to develop breasts. d. Nothing; breast budding is a normal finding. ssess the girl's weight and body mass index (BMI). Research has shown that girls with overweight or obese BMI levels have a higher occurrence of early onset of breast budding (before age 8 years for African-American girls and age 10 years for white girls) and early menarche. The nurse is reviewing statistics regarding breast cancer. Which woman, aged 40 years in the United States, has the highest risk for development of breast cancer? a. Black b. White c. Asian d. American Indian frican-American The incidence of breast cancer varies with different cultural groups. White women have a higher incidence of breast cancer than African-American women starting at age 45 years; but African-American women have a higher incidence before age 45 years. Asian, Hispanic, and American Indian women have a lower risk for development of breast cancer (American Cancer Society, 2009-2010). The nurse is preparing for a class in early detection of breast cancer. Which statement is true with regard to breast cancer in African-American women in the United States? a. Breast cancer is not a threat to black women. b. Black women have a lower incidence of regional or distant breast cancer than white women. c. Black women are more likely to die of breast cancer at any age. d. Breast cancer incidence in black women is higher than that of white women after age 45. frican-American women are more likely to die of breast cancer at any age. African-American women have a higher incidence of breast cancer before age 45 years than white women, and are more likely to die of their disease. In addition, African-American women are significantly more likely to be diagnosed with regional or distant breast cancer than are white women. This racial difference in mortality rates may be related to insufficient use of screening measures and lack of access to health care. During a breast health interview, a patient states that she has noticed pain in her left breast. The nurse's most appropriate response to this would be: a. "Don't worry about the pain; breast cancer is not painful." b. "I would like some more information about the pain in your left breast." c. "Oh, I had pain like that after my son was born; it turned out to be a blocked milk duct." d. "Breast pain is almost always the result of benign breast disease." "I would like some more information about the pain in your left breast." Breast pain occurs with trauma, inflammation, infection, or benign breast disease. The nurse will need to gather more information about the patient's pain rather than make statements that ignore the patient's concerns. During a history interview, a female patient states that she has noticed a few drops of clear discharge from her right nipple. What should the nurse do next? a. Immediately contact the physician to report the discharge. b. Ask her if she is possibly pregnant. c. Ask the patient some additional questions about the medications she is taking. d. Immediately obtain a sample for culture and sensitivity testing. sk her some additional questions about the medications she is taking. The use of some medications, such as oral contraceptives, phenothiazines, diuretics, digitalis, steroids, methyldopa, and calcium channel blockers, may cause clear nipple discharge. Bloody or blood-tinged discharge from the nipple, not clear, is significant, especially if a lump is also present. In the pregnant female, colostrum would be a thick, yellowish liquid, and it would be expressed after the fourth month of pregnancy. During a physical examination, a 45-year-old woman states that she has had a crusty, itchy rash on her breast for about 2 weeks. In trying to find the cause of the rash, which of these would be important for the nurse to determine? a. "Is the rash raised and red?" b. "Does it appear to be cyclic?" c. "Where did the rash first appear—on the nipple, the areola, or the surrounding skin?" d. "What was she doing when she first noticed the rash, and do her actions make it worse?" Where did it first appear—on the nipple, the areola, or the surrounding skin? It is important for the nurse to determine where the rash first appeared. Paget's disease starts with a small crust on the nipple apex and then spreads to the areola. Eczema or other dermatitis rarely starts at nipple unless it results from breastfeeding. It usually starts on the areola or surrounding skin and then spreads to the nipple. See Table 17-6. A patient is newly diagnosed with benign breast disease. The nurse recognizes that which statement about benign breast disease is true? The presence of benign breast disease: a. Makes it hard to examine the breasts. b. Frequently turns into cancer in a woman's later years. c. Is easily reduced with hormone replacement therapy. d. Is usually diagnosed before a woman reaches childbearing age. makes it harder to examine the breasts. The presence of benign breast disease (formerly fibrocystic breast disease) makes it harder to examine the breasts; the general lumpiness of the breast conceals a new lump. The other statements are not true. During an annual physical exam, a 43-year-old patient states that she doesn't perform monthly breast self-examinations (BSE). She tells the nurse that she believes that mammograms "do a much better job than I ever could to find a lump." The nurse should explain to her that: a. BSEs may detect lumps that appear between mammograms. b. BSEs are unnecessary until the age of 50 years. c. She is correct—mammography is a good replacement for BSE. d. She does not need to perform BSEs as long as a physician checks her breasts annually. - SEs may detect lumps that appear between mammograms. The monthly practice of breast self-examination, along with clinical breast examination and mammograms are complementary screening measures. Mammography can reveal cancers too small to be detected by the woman or by the most experienced examiner. However, interval lumps may become palpable between mammograms. During an interview, a patient reveals that she is pregnant. She states that she is not sure whether she will breastfeed her baby and asks for some information about this. Which of these statements by the nurse is accurate with regard to breastfeeding? a. "Breastfed babies tend to be more colicky." b. "Breastfeeding provides the perfect food and antibodies for your baby." c. "Breastfed babies eat more often than infants on formula." d. "Breastfeeding is second nature, and every woman can do it." "Breastfeeding provides the perfect food and antibodies for your baby." Exclusively breastfeeding for 6 months provides the perfect food and antibodies for the baby, decreases the risk of ear infections, promotes bonding, and provides relaxation. The nurse is reviewing risk factors for breast cancer. Which of these women have risk factors that place them at a higher risk for breast cancer? a. 37 year old who is slightly overweight b. 42 year old who has had ovarian cancer c. 45 year old who has never been pregnant d. 65 year old whose mother had breast cancer 65 year old whose mother had breast cancer Risk factors for breast cancer include having a first-degree relative with breast cancer (mother, sister, or daughter) and being older than 50 years. Refer to Table 17- 2 for other risk factors. During an examination of a woman, the nurse notices that her left breast is slightly larger than her right breast. Which of these statements is true about this finding? a. Breasts should always be symmetric. b. Asymmetry of breast size and shape is probably due to breastfeeding and is nothing to worry about. c. Asymmetry is not unusual, but the nurse should verify that this change is not new. d. Asymmetry of breast size and shape is very unusual and means she may have an inflammation or growth. This finding is not unusual, but the nurse should verify that this change is not new. The nurse should notice symmetry of size and shape. It is common to have a slight asymmetry in size; often the left breast is slightly larger than the right. A sudden increase in the size of one breast signifies inflammation or new growth. The nurse is assisting with a self-breast examination clinic. Which of these women reflect abnormal findings during the inspection phase of breast examination? a. Woman whose nipples are in different planes (deviated). b. Woman whose left breast is slightly larger than her right. c. Nonpregnant woman whose skin is marked with linear striae. d. Pregnant woman whose breasts have a fine blue network of veins visible under the skin. Woman whose nipples are in different planes (deviated) The nipples should be symmetrically placed on the same plane on the two breasts. With deviation in pointing, an underlying cancer causes fibrosis in the mammary ducts, which pulls the nipple angle toward it. The other examples are normal findings. See Table 17-3. During the physical examination, the nurse notices that a female patient has an inverted left nipple. Which statement regarding this is most accurate? a. Normal nipple inversion is usually bilateral. b. Unilateral inversion of a nipple is always a serious sign. c. Whether the inversion is a recent change should be determined. d. Nipple inversion is not significant unless accompanied by an underlying palpable mass. It should be determined whether the inversion is a recent change. The nurse should distinguish a recently retracted nipple from one that has been inverted for many years or since puberty. Normal nipple inversion may be unilateral or bilateral and usually can be pulled out (i.e., it is not fixed). Recent nipple retraction signifies acquired disease. See Table 17-3. The nurse is performing a breast examination. Which of these statements best describes the correct procedure to use when screening for nipple and skin retraction during a breast examination? Have the woman: a. Bend over and touch her toes. b. Lie down on her left side and notice any retraction. c. Shift from a supine position to a standing position, and note any lag or retraction. d. Slowly lift her arms above her head, and note any retraction or lag in movement. slowly lift her arms above her head and note any retraction or lag in movement Direct the woman to change position while checking the breasts for skin retraction signs. First ask her to lift her arms slowly over her head. Both breasts should move up symmetrically. Retraction signs are due to fibrosis in the breast tissue, usually caused by growing neoplasms. The nurse should notice if there is a lag in movement of one breast. The nurse is palpating a female patient's breasts during an examination. Which of these positions is most likely to make significant lumps more distinct during breast palpation? a. Supine with the arms raised over her head b. Sitting with the arms relaxed at her sides c. Supine with the arms relaxed at her sides d. Sitting with the arms flexed and fingertips touching her shoulders Supine with arms raised over her head The nurse should help the woman to a supine position, tuck a small pad under the side to be palpated, and help the woman raise her arm over her head. These maneuvers will flatten the breast tissue and displace it medially. Any significant lumps will then feel more distinct. Which of these clinical situations would the nurse consider to be outside normal limits? a. A patient has had one pregnancy and states that she believes she may be entering menopause. Her breast examination reveals breasts that are soft and slightly sagging. b. A patient has never been pregnant. Her breast examination reveals large pendulous breasts that have a firm, transverse ridge along the lower quadrant in both breasts. c. A patient has never been pregnant and reports that she should begin her period tomorrow. Her breast examination reveals breast tissue that is nodular and somewhat engorged. She states that the examination was slightly painful. d. A patient has had two pregnancies, and she breastfed both of her children. Her youngest child is now 10 years old. Her breast examination reveals breast tissue that is somewhat soft, and she has a small amount of thick yellow discharge from both nipples. patient has had two pregnancies and she breastfed both of her children. Her youngest child is now 10 years old. Her breast examination reveals breast tissue that is somewhat soft and she has a small amount of thick yellow discharge from both nipples. In nulliparous women, normal breast tissue feels firm, smooth, and elastic; after pregnancy, the tissue feels softer and looser. If any discharge appears, the nurse should note its color and consistency. Except in pregnancy and lactation, discharge is abnormal. Premenstrual engorgement is normal, and consists of a slight enlargement, tenderness to palpation, and a generalized nodularity. A firm, transverse ridge of compressed tissue in the lower quadrants, known as the inframammary ridge, is especially noticeable in large breasts. A patient states during the interview that she noticed a new lump in the shower a few days ago. It was on her left breast near her axilla. The nurse should plan to: a. Palpate the lump first. b. Palpate the unaffected breast first. c. Avoid palpating the lump because it could be a cyst, which might rupture. d. Palpate the breast with the lump first but plan to palpate the axilla last. palpate the unaffected breast first. If the woman mentions a breast lump she has discovered herself, the nurse should examine the unaffected breast first to learn a baseline of normal consistency for this individual. The nurse has palpated a lump in a female patient's right breast. The nurse documents this as a small, round, firm, distinct, lump located at 2 o'clock, 2 cm from the nipple. It is nontender and fixed. There is no associated retraction of skin or nipple, no erythema, and no axillary lymphadenopathy. Which of these statements reveals the information that is missing from the documentation? It is missing information about: a. Shape of the lump b. Consistency of the lump c. Size of the lump d. Whether the lump is solitary or multiple the size of the lump. If the nurse feels a lump or mass, he or she should note these characteristics: (1) location, (2) size—judge in centimeters in three dimensions: width length thickness, (3) shape, (4) consistency, (5) motility, (6) distinctness, (7) nipple, (8) the skin over the lump, (9) tenderness, and (10) lymphadenopathy. The nurse is conducting a class about breast self-examination (BSE). Which of these statements indicates proper BSE technique? a. The best time to perform BSE is in the middle of the menstrual cycle. b. The woman needs to perform BSE only bimonthly unless she has fibrocystic breast tissue. c. The best time to perform a BSE is 4 to 7 days after the first day of the menstrual period. d. If she suspects that she is pregnant, then the woman should not perform a BSE until her baby is born. The best time to perform BSE is 4 to 7 days after the first day of the menstrual period. The nurse should help each woman establish a regular schedule of self-care. The best time to conduct breast self-examination is right after the menstrual period, or the fourth through seventh day of the menstrual cycle, when the breasts are the smallest and least congested. Advise the pregnant or menopausal woman who is not having menstrual periods to select a familiar date to examine her breasts each month, for example, her birth date or the day the rent is due. The nurse is preparing to teach a woman about breast self-examination (BSE). Which statement by the nurse is correct? a. "BSE is more important than ever for you because you have never had any children." b. "BSE is so important because one out of nine women will develop breast cancer in her lifetime." c. "BSE on a monthly basis will help you become familiar with your own breasts and feel their normal variations." d. "BSE will save your life because you are likely to find a cancerous lump between mammograms." "BSE on a monthly basis will help you feel familiar with your own breasts and their normal variations." The nurse should stress that a regular monthly self-examination will familiarize her with her own breasts and their normal variations. This is a positive step that will reassure her of her healthy state. While teaching, the nurse should focus on the positive aspects of breast self-examination and should avoid citing frightening mortality statistics about breast cancer. This may generate excessive fear and denial that actually obstructs a woman's self-care action. A 55-year-old postmenopausal woman is being seen in the clinic for a yearly examination. She is concerned about changes in her breasts that she has noticed over the past 5 years. She states that her breasts have decreased in size and that the elasticity has changed so that her breasts seem "flat and flabby." The nurse's best reply would be: a. "This change occurs most often because of long-term use of bras that do not provide enough support to the breast tissues." b. "This is a normal change that occurs as women get older and is due to the increased levels of progesterone during the aging process." c. "Decreases in hormones after menopause causes atrophy of the glandular tissue in the breast and is a normal process of aging." d. "Postural changes in the spine make it appear that your breasts have changed in shape. Exercises to strengthen the muscles of the upper back and chest wall will help prevent the changes in elasticity and size." "Decreases in hormones after menopause causes atrophy of the glandular tissue in the breast. This is a normal process of aging." The hormonal changes of menopause cause the breast glandular tissue to atrophy, making the breasts more pendulous, flattened, and sagging. A 43-year-old woman is at the clinic for a routine examination. She reports that she has had a breast lump in her right breast for years. Recently, it has begun to change in consistency and is becoming harder. She reports that 5 years ago her physician evaluated the lump and determined that it "was nothing to worry about." The examination validates the presence of a mass in the right upper outer quadrant at 1 o'clock, approximately 5 cm from the nipple. It is firm, mobile, nontender, with borders that are not well defined. The nurse's recommendation to her is: a. "Because of the change in consistency of the lump, it should be further evaluated by a physician." b. "The changes could be related to your menstrual cycles. Keep track of the changes in the mass each month." c. "The lump is probably nothing to worry about because it has been present for years and was determined to be noncancerous 5 years ago." d. "Because you are experiencing no pain and the size has not changed, you should continue to monitor the lump and return to the clinic in 3 months." "Because of the change in consistency of the lump, it should be further evaluated by a physician." A lump that has been present for years and is not exhibiting changes may not be serious but still should be explored. Any recent change or new lump should be evaluated. The other responses are not correct. During a discussion about breast self-examination with a 30-year-old woman, which of these statements by the nurse is most appropriate? a. "The best time to examine your breasts is during ovulation." b. "Examine your breasts every month on the same day of the month." c. "Examine your breasts shortly after your menstrual period each month." d. "The best time to examine your breasts is immediately before menstruation." "Examine your breasts shortly after your menstrual period each month." The best time to conduct breast self-examination is shortly after the menstrual period when the breasts are the smallest and least congested. The nurse is discussing breast self-examination with a postmenopausal woman. The best time for postmenopausal women to perform breast self-examination is: a. On the same day every month. b. Daily, during the shower or bath. c. One week after her menstrual period. d. Every year with her annual gynecologic examination. the same day every month. Postmenopausal women are no longer experiencing regular menstrual cycles but need to continue to perform breast self-examination on a monthly basis. Choosing the same day of the month is a helpful reminder to perform breast self-examination. While inspecting a patient's breasts, the nurse finds that the left breast is slightly larger than the right with the presence of Montgomery's glands bilaterally. The nurse should: a. Palpate over the Montgomery glands, checking for drainage. b. Consider these findings as normal, and proceed with the examination. c. Ask extensive health history questions regarding the woman's breast asymmetry. d. Continue with the examination, and then refer the patient for further evaluation of the Montgomery glands. onsider these normal findings and proceed with the examination. Normal findings of the breast include one breast (most often the left) slightly larger than the other and the presence of Montgomery's glands across the areola. During an examination, the nurse notes a supernumerary nipple just under the patient's left breast. The patient tells the nurse that she always thought it was a mole. Which statement about this finding is correct? a. This variation is normal and not a significant finding. b. This finding is significant and needs further investigation. c. A supernumerary nipple also contains glandular tissue and may leak milk during pregnancy and lactation. d. The patient is correct—a supernumerary nipple is actually a mole that happens to be located under the breast. It is a normal variation and not a significant finding. A supernumerary nipple looks like a mole, but close examination reveals a tiny nipple and areola. It is not a significant finding. While examining a 75-year-old woman, the nurse notices that the skin over her right breast is thickened and the hair follicles are exaggerated. This condition is known as: a. Dimpling. b. Retraction. c. Peau d'orange. d. Benign breast disease. peau d'orange. This condition is known as peau d'orange. Lymphatic obstruction produces edema, which thickens the skin and exaggerates the hair follicles. The skin has a pig-skin or orange-peel look, and this condition suggests cancer. When a breastfeeding mother is diagnosed with a breast abscess, which of these instructions from the nurse is correct? The mother needs to: a. Continue to nurse on both sides to encourage milk flow. b. Immediately discontinue nursing to allow for healing. c. Temporarily discontinue nursing on the affected breast, and manually express milk and discard it. d. Temporarily discontinue nursing on affected breast, but manually express milk and give it to the baby. temporarily discontinue nursing on affected breast and manually express milk and discard it. With a breast abscess, the patient must temporarily discontinue nursing on the affected breast, manually express the milk, and discard it. Nursing can continue on the unaffected side. A new mother calls the clinic to report that part of her left breast is red, swollen, tender, very hot, and hard. She has a fever of 101 F. She has also had symptoms of the flu, such as chills, sweating, and feeling tired. The nurse notices that she has been breastfeeding for 1 month. From her description, what condition does the nurse suspect? a. Mastitis b. Paget disease c. Plugged milk duct d. Mammary duct ectasia Mastitis The symptoms describe mastitis, which stems from infection or stasis caused by a plugged duct. A plugged duct does not have infection present. (See Table 17-7.) Refer to Table 17-6 for descriptions of Paget's disease and mammary duct ectasia. During a breast examination on a female patient, the nurse notices that the nipple is flat, broad, and fixed. The patient states it "started doing that a few months ago." This finding suggests: a. Dimpling. b. Retracted nipple. c. Nipple inversion. d. Deviation in nipple pointing. retracted nipple. The retracted nipple looks flatter and broader, like an underlying crater. A recent retraction suggests cancer, which causes fibrosis of the whole duct system and pulls in the nipple. It also may occur with benign lesions such as ectasia of the ducts. The nurse should not confuse retraction with the normal long-standing type of nipple inversion, which has no broadening and is not fixed. A 54-year-old man comes to the clinic with a "horrible problem." He tells the nurse that he has just discovered a lump on his breast and is fearful of cancer. The nurse knows that which statement about breast cancer in males is true? a. Breast masses in men are difficult to detect because of minimal breast tissue. b. Breast cancer in men rarely spreads to the lymph nodes. c. One percent of all breast cancers occurs in men. d. Most breast masses in men are diagnosed as gynecomastia. One percent of all breast cancer occurs in men. One percent of all breast cancer occurs in men. Early spread to axillary lymph nodes occurs due to minimal breast tissue. The nurse is assessing the breasts of a 68-year-old woman and discovers a mass in the upper outer quadrant of the left breast. When assessing this mass, the nurse keeps in mind that characteristics of a cancerous mass include which of the following? Select all that apply. a. Nontender mass b. Dull, heavy pain on palpation c. Rubbery texture and mobile d. Hard, dense, and immobile e. Regular border f. Irregular, poorly delineated border Nontender mass Hard, dense, and immobile Irregular,poorly delineated border Cancerous breast masses are solitary, unilateral, nontender, masses. They are solid, hard, dense, and fixed to underlying tissues or skin as cancer becomes invasive. Their borders are irregular and poorly delineated. They are often painless, although the person may have pain. They are most common in upper outer quadrant. A dull, heavy pain on palpation and a mass with a rubbery texture and a regular border are characteristics of benign breast disease. The nurse is examining a 62-year-old man and notes that he has gynecomastia bilaterally. The nurse should explore his history for which related conditions? Select all that apply. a. Malnutrition b. Hyperthyroidism c. Type 2 diabetes mellitus d. Liver disease e. History of alcohol abuse Hyperthyroidism Liver disease History of alcohol abuse Gynecomastia occurs with obesity, Cushing's syndrome, liver cirrhosis, adrenal disease, hyperthyroidism, and numerous drugs: alcohol and marijuana use, estrogen treatment for prostate cancer, antibiotics (metronidazole, isoniazid), digoxin, ACE inhibitors, diazepam, and tricyclic antidepressants. 1. Which of these statements is true regarding the vertebra prominens? The vertebra prominens is: a. The spinous process of C7. b. Usually nonpalpable in most individuals. c. Opposite the interior border of the scapula. d. Located next to the manubrium of the sternum. The spinous process of C7 is the vertebra prominens and is the most prominent bony spur protruding at the base of the neck. Counting ribs and intercostal spaces on the posterior thorax is difficult because of the muscles and soft tissue. The vertebra prominens is easier to identify and is used as a starting point in counting thoracic processes and identifying landmarks on the posterior chest. 2. When performing a respiratory assessment on a patient, the nurse notices a costal angle of approximately 90 degrees. This characteristic is: a. Observed in patients with kyphosis. b. Indicative of pectus excavatum. c. A normal finding in a healthy adult. d. An expected finding in a patient with a barrel chest. The right and left costal margins form an angle where they meet at the xiphoid process. Usually, this angle is 90 degrees or less. The angle increases when the rib cage is chronically overinflated, as in emphysema. 3. When assessing a patients lungs, the nurse recalls that the left lung: a. Consists of two lobes. b. Is divided by the horizontal fissure. c. Primarily consists of an upper lobe on the posterior chest. d. Is shorter than the right lung because of the underlying stomach. The left lung has two lobes, and the right lung has three lobes. The right lung is shorter than the left lung because of the underlying liver. The left lung is narrower than the right lung because the heart bulges to the left. The posterior chest is almost all lower lobes. 4. Which statement about the apices of the lungs is true? The apices of the lungs: a. Are at the level of the second rib anteriorly. b. Extend 3 to 4 cm above the inner third of the clavicles. c. Are located at the sixth rib anteriorly and the eighth rib laterally. d. Rest on the diaphragm at the fifth intercostal space in the midclavicular line (MCL). - The apex of the lung on the anterior chest is 3 to 4 cm above the inner third of the clavicles. On the posterior chest, the apices are at the level of C7. 5. During an examination of the anterior thorax, the nurse is aware that the trachea bifurcates anteriorly at the: a. Costal angle. b. Sternal angle. c. Xiphoid process. d. Suprasternal notch. - The sternal angle marks the site of tracheal bifurcation into the right and left main bronchi; it corresponds with the upper borders of the atria of the heart, and it lies above the fourth thoracic vertebra on the back. 6. During an assessment, the nurse knows that expected assessment findings in the normal adult lung include the presence of: a. Adventitious sounds and limited chest expansion. b. Increased tactile fremitus and dull percussion tones. c. Muffled voice sounds and symmetric tactile fremitus. d. Absent voice sounds and hyperresonant percussion tones. Normal lung findings include symmetric chest expansion, resonant percussion tones, vesicular breath sounds over the peripheral lung fields, muffled voice sounds, and no adventitious sounds. 7. The primary muscles of respiration include the: a. Diaphragm and intercostals. b. Sternomastoids and scaleni. c. Trapezii and rectus abdominis. d. External obliques and pectoralis major. The major muscle of respiration is the diaphragm. The intercostal muscles lift the sternum and elevate the ribs during inspiration, increasing the anteroposterior diameter. Expiration is primarily passive. Forced inspiration involves the use of other muscles, such as the accessory neck musclessternomastoid, scaleni, and trapezii muscles. Forced expiration involves the abdominal muscles. 8. A 65-year-old patient with a history of heart failure comes to the clinic with complaints of being awakened from sleep with shortness of breath. Which action by the nurse is most appropriate? a. Obtaining a detailed health history of the patients allergies and a history of asthma b. Telling the patient to sleep on his or her right side to facilitate ease of respirations c. Assessing for other signs and symptoms of paroxysmal nocturnal dyspnea d. Assuring the patient that paroxysmal nocturnal dyspnea is normal and will probably resolve within the next week The patient is experiencing paroxysmal nocturnal dyspneabeing awakened from sleep with shortness of breath and the need to be upright to achieve comfort. 9. When assessing tactile fremitus, the nurse recalls that it is normal to feel tactile fremitus most intensely over which location? a. Between the scapulae b. Third intercostal space, MCL c. Fifth intercostal space, midaxillary line (MAL) d. Over the lower lobes, posterior side Normally, fremitus is most prominent between the scapulae and around the sternum. These sites are where the major bronchi are closest to the chest wall. Fremitus normally decreases as one progresses down the chest because more tissue impedes sound transmission. 10. The nurse is reviewing the technique of palpating for tactile fremitus with a new graduate. Which statement by the graduate nurse reflects a correct understanding of tactile fremitus? Tactile fremitus: a. Is caused by moisture in the alveoli. b. Indicates that air is present in the subcutaneous tissues. c. Is caused by sounds generated from the larynx. d. Reflects the blood flow through the pulmonary arteries. Fremitus is a palpable vibration. Sounds generated from the larynx are transmitted through patent bronchi and the lung parenchyma to the chest wall where they are felt as vibrations. Crepitus is the term for air in the subcutaneous tissues. 11. During percussion, the nurse knows that a dull percussion note elicited over a lung lobe most likely results from: a. Shallow breathing. b. Normal lung tissue. c. Decreased adipose tissue. d. Increased density of lung tissue. A dull percussion note indicates an abnormal density in the lungs, as with pneumonia, pleural effusion, atelectasis, or a tumor. Resonance is the expected finding in normal lung tissue. 12. The nurse is observing the auscultation technique of another nurse. The correct method to use when progressing from one auscultatory site on the thorax to another is _______ comparison. a. Side-to-side b. Top-to-bottom c. Posterior-to-anterior d. Interspace-by-interspace Side-to-side comparison is most important when auscultating the chest. The nurse should listen to at least one full respiration in each location. The other techniques are not correct. 13. When auscultating the lungs of an adult patient, the nurse notes that low-pitched, soft breath sounds are heard over the posterior lower lobes, with inspiration being longer than expiration. The nurse interprets that these sounds are: a. Normally auscultated over the trachea. b. Bronchial breath sounds and normal in that location. c. Vesicular breath sounds and normal in that location. d. Bronchovesicular breath sounds and normal in that location. Vesicular breath sounds are low-pitched, soft sounds with inspiration being longer than expiration. These breath sounds are expected over the peripheral lung fields where air flows through smaller bronchioles and alveoli. 14. The nurse is auscultating the chest in an adult. Which technique is correct? a. Instructing the patient to take deep, rapid breaths b. Instructing the patient to breathe in and out through his or her nose c. Firmly holding the diaphragm of the stethoscope against the chest d. Lightly holding the bell of the stethoscope against the chest to avoid friction Firmly holding the diaphragm of the stethoscope against the chest is the correct way to auscultate breath sounds. The patient should be instructed to breathe through his or her mouth, a little deeper than usual, but not to hyperventilate. 15. The nurse is percussing over the lungs of a patient with pneumonia. The nurse knows that percussion over an area of atelectasis in the lungs will reveal: a. Dullness. b. Tympany. c. Resonance. d. Hyperresonance. A dull percussion note signals an abnormal density in the lungs, as with pneumonia, pleural effusion, atelectasis, or a tumor. 16. During auscultation of the lungs, the nurse expects decreased breath sounds to be heard in which situation? a. When the bronchial tree is obstructed b. When adventitious sounds are present c. In conjunction with whispered pectoriloquy d. In conditions of consolidation, such as pneumonia Decreased or absent breath sounds occur when the bronchial tree is obstructed, as in emphysema, and when sound transmission is obstructed, as in pleurisy, pneumothorax, or pleural effusion. 17. The nurse knows that a normal finding when assessing the respiratory system of an older adult is: a. Increased thoracic expansion. b. Decreased mobility of the thorax. c. Decreased anteroposterior diameter. d. Bronchovesicular breath sounds throughout the lungs. - The costal cartilages become calcified with aging, resulting in a less mobile thorax. Chest expansion may be somewhat decreased, and the chest cage commonly shows an increased anteroposterior diameter. 18. A mother brings her 3-month-old infant to the clinic for evaluation of a cold. She tells the nurse that he has had a runny nose for a week. When performing the physical assessment, the nurse notes that the child has nasal flaring and sternal and intercostal retractions. The nurses next action should be to: a. Assure the mother that these signs are normal symptoms of a cold. b. Recognize that these are serious signs, and contact the physician. c. Ask the mother if the infant has had trouble with feedings. d. Perform a complete cardiac assessment because these signs are probably indicative of early heart failure. - The infant is an obligatory nose breather until the age of 3 months. Normally, no flaring of the nostrils and no sternal or intercostal retraction occurs. Significant retractions of the sternum and intercostal muscles and nasal flaring indicate increased inspiratory effort, as in pneumonia, acute airway obstruction, asthma, and atelectasis; therefore, immediate referral to the physician is warranted. These signs do not indicate heart failure, and an assessment of the infants feeding is not a priority at this time. 19. When assessing the respiratory system of a 4-year-old child, which of these findings would the nurse expect? a. Crepitus palpated at the costochondral junctions b. No diaphragmatic excursion as a result of a childs decreased inspiratory volume c. Presence of bronchovesicular breath sounds in the peripheral lung fields d. Irregular respiratory pattern and a respiratory rate of 40 breaths per minute at rest Bronchovesicular breath sounds in the peripheral lung fields of the infant and young child up to age 5 or 6 years are normal findings. Their thin chest walls with underdeveloped musculature do not dampen the sound, as do the thicker chest walls of adults; therefore, breath sounds are loud and harsh. 20. When inspecting the anterior chest of an adult, the nurse should include which assessment? a. Diaphragmatic excursion b. Symmetric chest expansion c. Presence of breath sounds d. Shape and configuration of the chest wall Inspection of the anterior chest includes shape and configuration of the chest wall; assessment of the patients level of consciousness and the patients skin color and condition; quality of respirations; presence or absence of retraction and bulging of the intercostal spaces; and use of accessory muscles. Symmetric chest expansion is assessed by palpation. Diaphragmatic excursion is assessed by percussion of the posterior chest. Breath sounds are assessed by auscultation. 21. The nurse knows that auscultation of fine crackles would most likely be noticed in: a. A healthy 5-year-old child. b. A pregnant woman. c. The immediate newborn period. d. Association with a pneumothorax. Fine crackles are commonly heard in the immediate newborn period as a result of the opening of the airways and a clearing of fluid. Persistent fine crackles would be noticed with pneumonia, bronchiolitis, or atelectasis. 22. During an assessment of an adult, the nurse has noted unequal chest expansion and recognizes that this occurs in which situation? a. In an obese patient b. When part of the lung is obstructed or collapsed c. When bulging of the intercostal spaces is present d. When accessory muscles are used to augment respiratory effort - Unequal chest expansion occurs when part of the lung is obstructed or collapsed, as with pneumonia, or when guarding to avoid postoperative incisional pain. 23. During auscultation of the lungs of an adult patient, the nurse notices the presence of bronchophony. The nurse should assess for signs of which condition? a. Airway obstruction b. Emphysema c. Pulmonary consolidation d. Asthma Pathologic conditions that increase lung density, such as pulmonary consolidation, will enhance the transmission of voice sounds, such as bronchophony (see Table 18-7). 24. The nurse is reviewing the characteristics of breath sounds. Which statement about bronchovesicular breath sounds is true? Bronchovesicular breath sounds are: a. Musical in quality. b. Usually caused by a pathologic disease. c. Expected near the major airways. d. Similar to bronchial sounds except shorter in duration. Bronchovesicular breath sounds are heard over major bronchi where fewer alveoli are located posteriorlybetween the scapulae, especially on the right; and anteriorly, around the upper sternum in the first and second intercostal spaces. The other responses are not correct. 25. The nurse is listening to the breath sounds of a patient with severe asthma. Air passing through narrowed bronchioles would produce which of these adventitious sounds? a. Wheezes b. Bronchial sounds c. Bronchophony d. Whispered pectoriloquy Wheezes are caused by air squeezed or compressed through passageways narrowed almost to closure by collapsing, swelling, secretions, or tumors, such as with acute asthma or chronic emphysema. 26. A patient has a long history of chronic obstructive pulmonary disease (COPD). During the assessment, the nurse will most likely observe which of these? a. Unequal chest expansion b. Increased tactile fremitus c. Atrophied neck and trapezius muscles d. Anteroposterior-to-transverse diameter ratio of 1:1 An anteroposterior-to-transverse diameter ratio of 1:1 or barrel chest is observed in individuals with COPD because of hyperinflation of the lungs. The ribs are more horizontal, and the chest appears as if held in continuous inspiration. Neck muscles are hypertrophied from aiding in forced respiration. Chest expansion may be decreased but symmetric. Decreased tactile fremitus occurs from decreased transmission of vibrations. 27. A teenage patient comes to the emergency department with complaints of an inability to breathe and a sharp pain in the left side of his chest. The assessment findings include cyanosis, tachypnea, tracheal deviation to the right, decreased tactile fremitus on the left, hyperresonance on the left, and decreased breath sounds on the left. The nurse interprets that these assessment findings are consistent with: a. Bronchitis. b. Pneumothorax. c. Acute pneumonia. d. Asthmatic attack. - With a pneumothorax, free air in the pleural space causes partial or complete lung collapse. If the pneumothorax is large, then tachypnea and cyanosis are evident. Unequal chest expansion, decreased or absent tactile fremitus, tracheal deviation to the unaffected side, decreased chest expansion, hyperresonant percussion tones, and decreased or absent breath sounds are found with the presence of pneumothorax. (See Table 18-8 for descriptions of the other conditions.) 28. An adult patient with a history of allergies comes to the clinic complaining of wheezing and difficulty in breathing when working in his yard. The assessment findings include tachypnea, the use of accessory neck muscles, prolonged expiration, intercostal retractions, decreased breath sounds, and expiratory wheezes. The nurse interprets that these assessment findings are consistent with: a. Asthma. b. Atelectasis. c. Lobar pneumonia. d. Heart failure. Asthma is allergic hypersensitivity to certain inhaled particles that produces inflammation and a reaction of bronchospasm, which increases airway resistance, especially during expiration. An increased respiratory rate, the use of accessory muscles, a retraction of the intercostal muscles, prolonged expiration, decreased breath sounds, and expiratory wheezing are all characteristics of asthma. (See Table 18-8 for descriptions of the other conditions.) 29. The nurse is assessing the lungs of an older adult. Which of these changes are normal in the respiratory system of the older adult? a. Severe dyspnea is experienced on exertion, resulting from changes in the lungs. b. Respiratory muscle strength increases to compensate for a decreased vital capacity. c. Decrease in small airway closure occurs, leading to problems with atelectasis. d. Lungs are less elastic and distensible, which decreases their ability to collapse and recoil. In the aging adult, the lungs are less elastic and distensible, which decreases their ability to collapse and recoil. Vital capacity is decreased, and a loss of intra-alveolar septa occurs, causing less surface area for gas exchange. The lung bases become less ventilated, and the older person is at risk for dyspnea with exertion beyond his or her usual workload. 30. A woman in her 26th week of pregnancy states that she is not really short of breath but feels that she is aware of her breathing and the need to breathe. What is the nurses best reply? a. The diaphragm becomes fixed during pregnancy, making it difficult to take in a deep breath. b. The increase in estrogen levels during pregnancy often causes a decrease in the diameter of the rib cage and makes it difficult to breathe. c. What you are experiencing is normal. Some women may interpret this as shortness of breath, but it is a normal finding and nothing is wrong. d. This increased awareness of the need to breathe is normal as the fetus grows because of the increased oxygen demand on the mothers body, which results in an increased respiratory rate. During pregnancy, the woman may develop an increased awareness of the need to breathe. Some women may interpret this as dyspnea, although structurally nothing is wrong. Increases in estrogen relax the chest cage ligaments, causing an increase in the transverse diameter. Although the growing fetus increases the oxygen demand on the mothers body, this increased demand is easily met by the increasing tidal volume (deeper breathing). Little change occurs in the respiratory rate. 31. A 35-year-old recent immigrant is being seen in the clinic for complaints of a cough that is associated with rust-colored sputum, low-grade afternoon fevers, and night sweats for the past 2 months. The nurses preliminary analysis, based on this history, is that this patient may be suffering from: a. Bronchitis. b. Pneumonia. c. Tuberculosis. d. Pulmonary edema. Sputum is not diagnostic alone, but some conditions have characteristic sputum production. Tuberculosis often produces rust-colored sputum in addition to other symptoms of night sweats and low-grade afternoon fevers (see Table 18-8). 32. A 70-year-old patient is being seen in the clinic for severe exacerbation of his heart failure. Which of these findings is the nurse most likely to observe in this patient? a. Shortness of breath, orthopnea, paroxysmal nocturnal dyspnea, and ankle edema b. Rasping cough, thick mucoid sputum, wheezing, and bronchitis c. Productive cough, dyspnea, weight loss, anorexia, and tuberculosis d. Fever, dry nonproductive cough, and diminished breath sounds A person with heart failure often exhibits increased respiratory rate, shortness of breath on exertion, orthopnea, paroxysmal nocturnal dyspnea, nocturia, ankle edema, and pallor in light-skinned individuals. A patient with rasping cough, thick mucoid sputum, and wheezing may have bronchitis. Productive cough, dyspnea, weight loss, and dyspnea indicate tuberculosis; fever, dry nonproductive cough, and diminished breath sounds may indicate Pneumocystis jiroveci (P. carinii) pneumonia (see Table 18-8). 33. A patient comes to the clinic complaining of a cough that is worse at night but not as bad during the day. The nurse recognizes that this cough may indicate: a. Pneumonia. b. Postnasal drip or sinusitis. c. Exposure to irritants at work. d. Chronic bronchial irritation from smoking. - A cough that primarily occurs at night may indicate postnasal drip or sinusitis. Exposure to irritants at work causes an afternoon or evening cough. Smokers experience early morning coughing. Coughing associated with acute illnesses such as pneumonia is continuous throughout the day. 34. During a morning assessment, the nurse notices that the patients sputum is frothy and pink. Which condition could this finding indicate? a. Croup b. Tuberculosis c. Viral infection d. Pulmonary edema Sputum, alone, is not diagnostic, but some conditions have characteristic sputum production. Pink, frothy sputum indicates pulmonary edema or it may be a side effect of sympathomimetic medications. Croup is associated with a barking cough, not sputum production. Tuberculosis may produce rust-colored sputum. Viral infections may produce white or clear mucoid sputum. 35. During auscultation of breath sounds, the nurse should correctly use the stethoscope in which of the following ways? a. Listening to at least one full respiration in each location b. Listening as the patient inhales and then going to the next site during exhalation c. Instructing the patient to breathe in and out rapidly while listening to the breath sounds d. If the patient is modest, listening to sounds over his or her clothing or hospital gown During auscultation of breath sounds with a stethoscope, listening to one full respiration in each location is important. During the examination, the nurse should monitor the breathing and offer times for the person to breathe normally to prevent possible dizziness. 36. A patient has been admitted to the emergency department with a possible medical diagnosis of pulmonary embolism. The nurse expects to see which assessment findings related to this condition? a. Absent or decreased breath sounds b. Productive cough with thin, frothy sputum c. Chest pain that is worse on deep inspiration and dyspnea d. Diffuse infiltrates with areas of dullness upon percussion Findings for pulmonary embolism include chest pain that is worse on deep inspiration, dyspnea, apprehension, anxiety, restlessness, partial arterial pressure of oxygen (PaO2) less than 80 mm Hg, diaphoresis, hypotension, crackles, and wheezes. 37. During palpation of the anterior chest wall, the nurse notices a coarse, crackling sensation over the skin surface. On the basis of these findings, the nurse suspects: a. Tactile fremitus. b. Crepitus. c. Friction rub. d. Adventitious sounds. - Crepitus is a coarse, crackling sensation palpable over the skin surface. It occurs in subcutaneous emphysema when air escapes from the lung and enters the subcutaneous tissue, such as after open thoracic injury or surgery. 38. The nurse is auscultating the lungs of a patient who had been sleeping and notices short, popping, crackling sounds that stop after a few breaths. The nurse recognizes that these breath sounds are: a. Atelectatic crackles that do not have a pathologic cause. b. Fine crackles and may be a sign of pneumonia. c. Vesicular breath sounds. d. Fine wheezes. One type of adventitious sound, atelectatic crackles, does not have a pathologic cause. They are short, popping, crackling sounds that sound similar to fine crackles but do not last beyond a few breaths. When sections of alveoli are not fully aerated (as in people who are asleep or in older adults), they deflate slightly and accumulate secretions. Crackles are heard when these sections are expanded by a few deep breaths. Atelectatic crackles are heard only in the periphery, usually in dependent portions of the lungs, and disappear after the first few breaths or after a cough. 39. A patient has been admitted to the emergency department for a suspected drug overdose. His respirations are shallow, with an irregular pattern, with a rate of 12 respirations per minute. The nurse interprets this respiration pattern as which of the following? a. Bradypnea b. Cheyne-Stokes respirations c. Hypoventilation d. Chronic obstructive breathing Hypoventilation is characterized by an irregular, shallow pattern, and can be caused by an overdose of narcotics or anesthetics. Bradypnea is slow breathing, with a rate less than 10 respirations per minute. (See Table 18-4 for descriptions of Cheyne-Stokes respirations and chronic obstructive breathing.) 40. A patient with pleuritis has been admitted to the hospital and complains of pain with breathing. What other key assessment finding would the nurse expect to find upon auscultation? a. Stridor b. Friction rub c. Crackles d. Wheezing - A patient with pleuritis will exhibit a pleural friction rub upon auscultation. This sound is made when the pleurae become inflamed and rub together during respiration. The sound is superficial, coarse, and low-pitched, as if two pieces of leather are being rubbed together. Stridor is associated with croup, acute epiglottitis in children, and foreign body inhalation. Crackles are associated with pneumonia, heart failure, chronic bronchitis, and other diseases (see Table 18-6). Wheezes are associated with diffuse airway obstruction caused by acute asthma or chronic emphysema. 1. The nurse is assessing voice sounds during a respiratory assessment. Which of these findings indicates a normal assessment? Select all that apply. a. Voice sounds are faint, muffled, and almost inaudible when the patient whispers one, two, three in a very soft voice. b. As the patient repeatedly says ninety-nine, the examiner clearly hears the words ninety-nine. c. When the patient speaks in a normal voice, the examiner can hear a sound but cannot exactly distinguish what is being said. d. As the patient says a long ee-ee-ee sound, the examiner also hears a long ee-ee-ee sound. e. As the patient says a long ee-ee-ee sound, the examiner hears a long aaaaaa sound. , C, D As a patient repeatedly says ninety-nine, normally the examiner hears voice sounds but cannot distinguish what is being said. If a clear ninety-nine is auscultated, then it could indicate increased lung density, which enhances the transmission of voice sounds, which is a measure of bronchophony. When a patient says a long ee-ee-ee sound, normally the examiner also hears a long ee-ee-ee sound through auscultation, which is a measure of egophony. If the examiner hears a long aaaaaa sound instead, this sound could indicate areas of consolidation or compression. With whispered pectoriloquy, as when a patient whispers a phrase such as one-two-three, the normal response when auscultating voice sounds is to hear sounds that are faint, muffled, and almost inaudible. If the examiner clearly hears the whispered voice, as if the patient is speaking through the stethoscope, then consolidation of the lung fields may exist. 1. The two parts of the nervous system are the: a. Motor and sensory. b. Central and peripheral. c. Peripheral and autonomic. d. Hypothalamus and cerebral. - The nervous system can be divided into two partscentral and peripheral. The central nervous system includes the brain and spinal cord. The peripheral nervous system includes the 12 pairs of cranial nerves (CNs), the 31 pairs of spinal nerves, and all of their branches. 2. The wife of a 65-year-old man tells the nurse that she is concerned because she has noticed a change in her husbands personality and ability to understand. He also cries very easily and becomes angry. The nurse recalls that the cerebral lobe responsible for these behaviors is the __________ lobe. a. Frontal b. Parietal c. Occipital d. Temporal The frontal lobe has areas responsible for personality, behavior, emotions, and intellectual function. The parietal lobe has areas responsible for sensation; the occipital lobe is responsible for visual reception; and the temporal lobe is responsible for hearing, taste, and smell. 3. Which statement concerning the areas of the brain is true? a. The cerebellum is the center for speech and emotions. b. The hypothalamus controls body temperature and regulates sleep. c. The basal ganglia are responsible for controlling voluntary movements. d. Motor pathways of the spinal cord and brainstem synapse in the thalamus. - The hypothalamus is a vital area with many important functions: body temperature controller, sleep center, anterior and posterior pituitary gland regulator, and coordinator of autonomic nervous system activity and emotional status. The cerebellum controls motor coordination, equilibrium, and balance. The basal ganglia control autonomic movements of the body. The motor pathways of the spinal cord synapse in various areas of the spinal cord, not in the thalamus. 4. The area of the nervous system that is responsible for mediating reflexes is the: a. Medulla. b. Cerebellum. c. Spinal cord. d. Cerebral cortex. The spinal cord is the main highway for ascending and descending fiber tracts that connect the brain to the spinal nerves; it is responsible for mediating reflexes. 5. While gathering equipment after an injection, a nurse accidentally received a prick from an improperly capped needle. To interpret this sensation, which of these areas must be intact? a. Corticospinal tract, medulla, and basal ganglia b. Pyramidal tract, hypothalamus, and sensory cortex c. Lateral spinothalamic tract, thalamus, and sensory cortex d. Anterior spinothalamic tract, basal ganglia, and sensory cortex The spinothalamic tract contains sensory fibers that transmit the sensations of pain, temperature, and crude or light touch. Fibers carrying pain and temperature sensations ascend the lateral spinothalamic tract, whereas the sensations of crude touch form the anterior spinothalamic tract. At the thalamus, the fibers synapse with another sensory neuron, which carries the message to the sensory cortex for full interpretation. The other options are not correct. 6. A patient with a lack of oxygen to his heart will have pain in his chest and possibly in the shoulder, arms, or jaw. The nurse knows that the best explanation why this occurs is which one of these statements? a. A problem exists with the sensory cortex and its ability to discriminate the location. b. The lack of oxygen in his heart has resulted in decreased amount of oxygen to the areas experiencing the pain. c. The sensory cortex does not have the ability to localize pain in the heart; consequently, the pain is felt elsewhere. d. A lesion has developed in the dorsal root, which is preventing the sensation from being transmitted normally. The sensory cortex is arranged in a specific pattern, forming a corresponding map of the body. Pain in the right hand is perceived at a specific spot on the map. Some organs, such as the heart, liver, and spleen, are absent from the brain map. Pain originating in these organs is referred because no felt image exists in which to have pain. Pain is felt by proxy, that is, by another body part that does have a felt image. The other responses are not correct explanations. 7. The ability that humans have to perform very skilled movements such as writing is controlled by the: a. Basal ganglia. b. Corticospinal tract. c. Spinothalamic tract. d. Extrapyramidal tract. - Corticospinal fibers mediate voluntary movement, particularly very skilled, discrete, and purposeful movements, such as writing. The corticospinal tract, also known as the pyramidal tract, is a newer, higher motor system that humans have that permits very skilled and purposeful movements. The other responses are not related to skilled movements. 8. A 30-year-old woman tells the nurse that she has been very unsteady and has had difficulty in maintaining her balance. Which area of the brain that is related to these findings would concern the nurse? a. Thalamus b. Brainstem c. Cerebellum d. Extrapyramidal tract The cerebellar system coordinates movement, maintains equilibrium, and helps maintain posture. The thalamus is the primary relay station where sensory pathways of the spinal cord, cerebellum, and brainstem form synapses on their way to the cerebral cortex. The brainstem consists of the midbrain, pons, and medulla and has various functions, especially concerning autonomic centers. The extrapyramidal tract maintains muscle tone for gross automatic movements, such as walking. 9. Which of these statements about the peripheral nervous system is correct? a. The CNs enter the brain through the spinal cord. b. Efferent fibers carry sensory input to the central nervous system through the spinal cord. c. The peripheral nerves are inside the central nervous system and carry impulses through their motor fibers. d. The peripheral nerves carry input to the central nervous system by afferent fibers and away from the central nervous system by efferent fibers. A nerve is a bundle of fibers outside of the central nervous system. The peripheral nerves carry input to the central nervous system by their sensory afferent fibers and deliver output from the central nervous system by their efferent fibers. The other responses are not related to the peripheral nervous system. 10. A patient has a severed spinal nerve as a result of trauma. Which statement is true in this situation? a. Because there are 31 pairs of spinal nerves, no effect results if only one nerve is severed. b. The dermatome served by this nerve will no longer experience any sensation. c. The adjacent spinal nerves will continue to carry sensations for the dermatome served by the severed nerve. d. A severed spinal nerve will only affect motor function of the patient because spinal nerves have no sensory component. A dermatome is a circumscribed skin area that is primarily supplied from one spinal cord segment through a particular spinal nerve. The dermatomes overlap, which is a form of biologic insurance; that is, if one nerve is severed, then most of the sensations can be transmitted by the spinal nerve above and the spinal nerve below the severed nerve. 11. A 21-year-old patient has a head injury resulting from trauma and is unconscious. There are no other injuries. During the assessment what would the nurse expect to find when testing the patients deep tendon reflexes? a. Reflexes will be normal. b. Reflexes cannot be elicited. c. All reflexes will be diminished but present. d. Some reflexes will be present, depending on the area of injury. A reflex is a defense mechanism of the nervous system. It operates below the level of conscious control and permits a quick reaction to potentially painful or damaging situations. 12. A mother of a 1-month-old infant asks the nurse why it takes so long for infants to learn to roll over. The nurse knows that the reason for this is: a. A demyelinating process must be occurring with her infant. b. Myelin is needed to conduct the impulses, and the neurons of a newborn are not yet myelinated. c. The cerebral cortex is not fully developed; therefore, control over motor function gradually occurs. d. The spinal cord is controlling the movement because the cerebellum is not yet fully developed. - The infants sensory and motor development proceeds along with the gradual acquisition of myelin, which is needed to conduct most impulses. Very little cortical control exists, and the neurons are not yet myelinated. The other responses are not correct. 13. During an assessment of an 80-year-old patient, the nurse notices the following: an inability to identify vibrations at her ankle and to identify the position of her big toe, a slower and more deliberate gait, and a slightly impaired tactile sensation. All other neurologic findings are normal. The nurse should interpret that these findings indicate: a. CN dysfunction. b. Lesion in the cerebral cortex. c. Normal changes attributable to aging. d. Demyelination of nerves attributable to a lesion. Some aging adults show a slower response to requests, especially for those calling for coordination of movements. The findings listed are normal in the absence of other significant abnormal findings. The other responses are incorrect. 14. A 70-year-old woman tells the nurse that every time she gets up in the morning or after shes been sitting, she gets really dizzy and feels like she is going to fall over. The nurses best response would be: a. Have you been extremely tired lately? b. You probably just need to drink more liquids. c. Ill refer you for a complete neurologic examination. d. You need to get up slowly when youve been lying down or sitting. Aging is accompanied by a progressive decrease in cerebral blood flow. In some people, this decrease causes dizziness and a loss of balance with a position change. These individuals need to be taught to get up slowly. The other responses are incorrect. 15. During the taking of the health history, a patient tells the nurse that it feels like the room is spinning around me. The nurse would document this finding as: a. Vertigo. b. Syncope. c. Dizziness. d. Seizure activity. True vertigo is rotational spinning caused by a neurologic dysfunction or a problem in the vestibular apparatus or the vestibular nuclei in the brainstem. Syncope is a sudden loss of strength or a temporary loss of consciousness. Dizziness is a lightheaded, swimming sensation. Seizure activity is characterized by altered or loss of consciousness, involuntary muscle movements, and sensory disturbances. 16. When taking the health history on a patient with a seizure disorder, the nurse assesses whether the patient has an aura. Which of these would be the best question for obtaining this information? a. Does your muscle tone seem tense or limp? b. After the seizure, do you spend a lot of time sleeping? c. Do you have any warning sign before your seizure starts? d. Do you experience any color change or incontinence during the seizure? Aura is a subjective sensation that precedes a seizure; it could be auditory, visual, or motor. The other questions do not solicit information about an aura. 17. While obtaining a health history of a 3-month-old infant from the mother, the nurse asks about the infants ability to suck and grasp the mothers finger. What is the nurse assessing? a. Reflexes b. Intelligence c. CNs d. Cerebral cortex function Questions regarding reflexes include such questions as, What have you noticed about the infants behavior, Are the infants sucking and swallowing seem coordinated, and Does the infant grasp your finger? The other responses are incorrect. 18. In obtaining a health history on a 74-year-old patient, the nurse notes that he drinks alcohol daily and that he has noticed a tremor in his hands that affects his ability to hold things. With this information, what response should the nurse make? a. Does your family know you are drinking every day? b. Does the tremor change when you drink alcohol? c. Well do some tests to see what is causing the tremor. d. You really shouldnt drink so much alcohol; it may be causing your tremor. - Senile tremor is relieved by alcohol, although not a recommended treatment. The nurse should assess whether the person is abusing alcohol in an effort to relieve the tremor. 19. A 50-year-old woman is in the clinic for weakness in her left arm and leg that she has noticed for the past week. The nurse should perform which type of neurologic examination? a. Glasgow Coma Scale b. Neurologic recheck examination c. Screening neurologic examination d. Complete neurologic examination The nurse should perform a complete neurologic examination on an individual who has neurologic concerns (e.g., headache, weakness, loss of coordination) or who is showing signs of neurologic dysfunction. The Glasgow Coma Scale is used to define a persons level of consciousness. The neurologic recheck examination is appropriate for those who are demonstrating neurologic deficits. The screening neurologic examination is performed on seemingly well individuals who have no significant subjective findings from the health history. 20. During an assessment of the CNs, the nurse finds the following: asymmetry when the patient smiles or frowns, uneven lifting of the eyebrows, sagging of the lower eyelids, and escape of air when the nurse presses against the right puffed cheek. This would indicate dysfunction of which of these CNs? a. Motor component of CN IV b. Motor component of CN VII c. Motor and sensory components of CN XI d. Motor component of CN X and sensory component of CN VII - The findings listed reflect a dysfunction of the motor component of the facial nerve (CN VII). 21. The nurse is testing the function of CN XI. Which statement best describes the response the nurse should expect if this nerve is intact? The patient: a. Demonstrates the ability to hear normal conversation. b. Sticks out the tongue midline without tremors or deviation. c. Follows an object with his or her eyes without nystagmus or strabismus. d. Moves the head and shoulders against resistance with equal strength. The following normal findings are expected when testing the spinal accessory nerve (CN XI): The patients sternomastoid and trapezius muscles are equal in size; the person can forcibly rotate the head both ways against resistance applied to the side of the chin with equal strength; and the patient can shrug the shoulders against resistance with equal strength on both sides. Checking the patients ability to hear normal conversation checks the function of CN VIII. Having the patient stick out the tongue checks the function of CN XII. Testing the eyes for nystagmus or strabismus is performed to check CNs III, IV, and VI. 22. During the neurologic assessment of a healthy 35-year-old patient, the nurse asks him to relax his muscles completely. The nurse then moves each extremity through full range of motion. Which of these results would the nurse expect to find? a. Firm, rigid resistance to movement b. Mild, even resistance to movement c. Hypotonic muscles as a result of total relaxation d. Slight pain with some directions of movement - Tone is the normal degree of tension (contraction) in voluntarily relaxed muscles. It shows a mild resistance to passive stretching. Normally, the nurse will notice a mild, even resistance to movement. The other responses are not correct. 23. When the nurse asks a 68-year-old patient to stand with his feet together and arms at his side with his eyes closed, he starts to sway and moves his feet farther apart. The nurse would document this finding as: a. Ataxia. b. Lack of coordination. c. Negative Homans sign. d. Positive Romberg sign. Abnormal findings for the Romberg test include swaying, falling, and a widening base of the feet to avoid falling. A positive Romberg sign is a loss of balance that is increased by the closing of the eyes. Ataxia is an uncoordinated or unsteady gait. Homans sign is used to test the legs for deep-vein thrombosis. 24. The nurse is performing an assessment on a 29-year-old woman who visits the clinic complaining of always dropping things and falling down. While testing rapid alternating movements, the nurse notices that the woman is unable to pat both of her knees. Her response is extremely slow and she frequently misses. What should the nurse suspect? a. Vestibular disease b. Lesion of CN IX c. Dysfunction of the cerebellum d. Inability to understand directions When a person tries to perform rapid, alternating movements, responses that are slow, clumsy, and sloppy are indicative of cerebellar disease. The other responses are incorrect. 25. During the taking of the health history of a 78-year-old man, his wife states that he occasionally has problems with short-term memory loss and confusion: He cant even remember how to button his shirt. When assessing his sensory system, which action by the nurse is most appropriate? a. The nurse would not test the sensory system as part of the examination because the results would not be valid. b. The nurse would perform the tests, knowing that mental status does not affect sensory ability. c. The nurse would proceed with an explanation of each test, making certain that the wife understands. d. Before testing, the nurse would assess the patients mental status and ability to follow directions. The nurse should ensure the validity of the sensory system testing by making certain that the patient is alert, cooperative, comfortable, and has an adequate attention span. Otherwise, the nurse may obtain misleading and invalid results. 26. The assessment of a 60-year-old patient has taken longer than anticipated. In testing his pain perception, the nurse decides to complete the test as quickly as possible. When the nurse applies the sharp point of the pin on his arm several times, he is only able to identify these as one very sharp prick. What would be the most accurate explanation for this? a. The patient has hyperesthesia as a result of the aging process. b. This response is most likely the result of the summation effect. c. The nurse was probably not poking hard enough with the pin in the other areas. d. The patient most likely has analgesia in some areas of arm and hyperalgesia in others. - At least 2 seconds should be allowed to elapse between each stimulus to avoid summation. With summation, frequent consecutive stimuli are perceived as one strong stimulus. The other responses are incorrect. 27. The nurse is performing a neurologic assessment on a 41-year-old woman with a history of diabetes. When testing her ability to feel the vibrations of a tuning fork, the nurse notices that the patient is unable to feel vibrations on the great toe or ankle bilaterally, but she is able to feel vibrations on both patellae. Given this information, what would the nurse suspect? a. Hyperalgesia b. Hyperesthesia c. Peripheral neuropathy d. Lesion of sensory cortex Loss of vibration sense occurs with peripheral neuropathy (e.g., diabetes and alcoholism). Peripheral neuropathy is worse at the feet and gradually improves as the examiner moves up the leg, as opposed to a specific nerve lesion, which has a clear zone of deficit for its dermatome. The other responses are incorrect 28. The nurse places a key in the hand of a patient and he identifies it as a penny. What term would the nurse use to describe this finding? a. Extinction b. Astereognosis c. Graphesthesia d. Tactile discrimination - Stereognosis is the persons ability to recognize objects by feeling their forms, sizes, and weights. Astereognosis is an inability to identify objects correctly, and it occurs in sensory cortex lesions. Tactile discrimination tests fine touch. Extinction tests the persons ability to feel sensations on both sides of the body at the same point. 29. The nurse is testing the deep tendon reflexes of a 30-year-old woman who is in the clinic for an annual physical examination. When striking the Achilles heel and quadriceps muscle, the nurse is unable to elicit a reflex. The nurses next response should be to: a. Ask the patient to lock her fingers and pull. b. Complete the examination, and then test these reflexes again. c. Refer the patient to a specialist for further testing. d. Document these reflexes as 0 on a scale of 0 to 4+. Sometimes the reflex response fails to appear. Documenting the reflexes as absent is inappropriate this soon in the examination. The nurse should try to further encourage relaxation, varying the persons position or increasing the strength of the blow. Reinforcement is another technique to relax the muscles and enhance the response. The person should be asked to perform an isometric exercise in a muscle group somewhat away from the one being tested. For example, to enhance a patellar reflex, the person should be asked to lock the fingers together and pull. 30. In assessing a 70-year-old patient who has had a recent cerebrovascular accident, the nurse notices right-sided weakness. What might the nurse expect to find when testing his reflexes on the right side? a. Lack of reflexes b. Normal reflexes c. Diminished reflexes d. Hyperactive reflexes Hyperreflexia is the exaggerated reflex observed when the monosynaptic reflex arc is released from the influence of higher cortical levels. This response occurs with upper motor neuron lesions (e.g., a cerebrovascular accident). The other responses are incorrect. 31. When the nurse is testing the triceps reflex, what is the expected response? a. Flexion of the hand b. Pronation of the hand c. Extension of the forearm d. Flexion of the forearm The normal response of the triceps reflex is extension of the forearm. The normal response of the biceps reflex causes flexion of the forearm. The other responses are incorrect. 32. The nurse is testing superficial reflexes on an adult patient. When stroking up the lateral side of the sole and across the ball of the foot, the nurse notices the plantar flexion of the toes. How should the nurse document this finding? a. Positive Babinski sign b. Plantar reflex abnormal c. Plantar reflex present d. Plantar reflex 2+ on a scale from 0 to 4+ With the same instrument, the nurse should draw a light stroke up the lateral side of the sole of the foot and across the ball of the foot, similar to an upside-down J. The normal response is plantar flexion of the toes and sometimes of the entire foot. A positive Babinski sign is abnormal and occurs with the response of dorsiflexion of the big toe and fanning of all toes. The plantar reflex is not graded on a 0 to 4+ scale. 33. In the assessment of a 1-month-old infant, the nurse notices a lack of response to noise or stimulation. The mother reports that in the last week he has been sleeping all of the time, and when he is awake all he does is cry. The nurse hears that the infants cries are very high pitched and shrill. What should be the nurses appropriate response to these findings? a. Refer the infant for further testing. b. Talk with the mother about eating habits. c. Do nothing; these are expected findings for an infant this age. d. Tell the mother to bring the baby back in 1 week for a recheck. A high-pitched, shrill cry or cat-sounding screech occurs with central nervous system damage. Lethargy, hyporeactivity, and hyperirritability, as well as the parents report of significant changes in behavior all warrant referral. The other options are not correct responses. 34. Which of these tests would the nurse use to check the motor coordination of an 11-month-old infant? a. Denver II b. Stereognosis c. Deep tendon reflexes d. Rapid alternating movements To screen gross and fine motor coordination, the nurse should use the Denver II with its age-specific developmental milestones. Stereognosis tests a persons ability to recognize objects by feeling them and is not appropriate for an 11-month-old infant. Testing the deep tendon reflexes is not appropriate for checking motor coordination. Testing rapid alternating movements is appropriate for testing coordination in adults. 35. To assess the head control of a 4-month-old infant, the nurse lifts up the infant in a prone position while supporting his chest. The nurse looks for what normal response? The infant: a. Raises the head, and arches the back. b. Extends the arms, and drops down the head. c. Flexes the knees and elbows with the back straight. d. Holds the head at 45 degrees, and keeps the back straight. At 3 months of age, the infant raises the head and arches the back as if in a swan dive. This response is the Landau reflex, which persists until 1 years of age (see Figure 23-43). The other responses are incorrect. 36. While assessing a 7-month-old infant, the nurse makes a loud noise and notices the following response: abduction and flexion of the arms and legs; fanning of the fingers, and curling of the index finger and thumb in a C position, followed by the infant bringing in the arms and legs to the body. What does the nurse know about this response? a. This response could indicate brachial nerve palsy. b. This reaction is an expected startle response at this age. c. This reflex should have disappeared between 1 and 4 months of age. d. This response is normal as long as the movements are bilaterally symmetric. The Moro reflex is present at birth and usually disappears at 1 to 4 months. Absence of the Moro reflex in the newborn or its persistence after 5 months of age indicates severe central nervous system injury. The other responses are incorrect. 37. To test for gross motor skill and coordination of a 6-year-old child, which of these techniques would be appropriate? Ask the child to: a. Hop on one foot. b. Stand on his head. c. Touch his finger to his nose. d. Make funny faces at the nurse. Normally, a child can hop on one foot and can balance on one foot for approximately 5 seconds by 4 years of age and can balance on one foot for 8 to 10 seconds at 5 years of age. Children enjoy performing these tests. Failure to hop after 5 years of age indicates incoordination of gross motor skills. Asking the child to touch his or her finger to the nose checks fine motor coordination; and asking the child to make funny faces tests CN VII. Asking a child to stand on his or her head is not appropriate. 38. During the assessment of an 80-year-old patient, the nurse notices that his hands show tremors when he reaches for something and his head is always nodding. No associated rigidity is observed with movement. Which of these statements is most accurate? a. These findings are normal, resulting from aging. b. These findings could be related to hyperthyroidism. c. These findings are the result of Parkinson disease. d. This patient should be evaluated for a cerebellar lesion. Senile tremors occasionally occur. These benign tremors include an intention tremor of the hands, head nodding (as if saying yes or no), and tongue protrusion. Tremors associated with Parkinson disease include rigidity, slowness, and a weakness of voluntary movement. The other responses are incorrect. 39. While the nurse is taking the history of a 68-year-old patient who sustained a head injury 3 days earlier, he tells the nurse that he is on a cruise ship and is 30 years old. The nurse knows that this finding is indicative of a(n): a. Great sense of humor. b. Uncooperative behavior. c. Inability to understand questions. d. Decreased level of consciousness. A change in consciousness may be subtle. The nurse should notice any decreasing level of consciousness, disorientation, memory loss, uncooperative behavior, or even complacency in a previously combative person. The other responses are incorrect. 40. The nurse is caring for a patient who has just had neurosurgery. To assess for increased intracranial pressure, what would the nurse include in the assessment? a. CNs, motor function, and sensory function b. Deep tendon reflexes, vital signs, and coordinated movements c. Level of consciousness, motor function, pupillary response, and vital signs d. Mental status, deep tendon reflexes, sensory function, and pupillary response Some hospitalized persons have head trauma or a neurologic deficit from a systemic disease process. These people must be closely monitored for any improvement or deterioration in neurologic status and for any indication of increasing intracranial pressure. The nurse should use an abbreviation of the neurologic examination in the following sequence: level of consciousness, motor function, pupillary response, and vital signs. 41. During an assessment of a 22-year-old woman who sustained a head injury from an automobile accident 4 hours earlier, the nurse notices the following changes: pupils were equal, but now the right pupil is fully dilated and nonreactive, and the left pupil is 4 mm and reacts to light. What do these findings suggest? a. Injury to the right eye b. Increased intracranial pressure c. Test inaccurately performed d. Normal response after a head injury - In a person with a brain injury, a sudden, unilateral, dilated, and nonreactive pupil is ominous. CN III runs parallel to the brainstem. When increasing intracranial pressure pushes down the brainstem (uncal herniation), it puts pressure on CN III, causing pupil dilation. The other responses are incorrect. 42. A 32-year-old woman tells the nurse that she has noticed very sudden, jerky movements mainly in her hands and arms. She says, They seem to come and go, primarily when I am trying to do something. I havent noticed them when Im sleeping. This description suggests: a. Tics. b. Athetosis. c. Myoclonus. d. Chorea. Chorea is characterized by sudden, rapid, jerky, purposeless movements that involve the limbs, trunk, or face. Chorea occurs at irregular intervals, and the movements are all accentuated by voluntary actions. (See Table 23-5 for the descriptions of athetosis, myoclonus, and tics.) 43. During an assessment of a 62-year-old man, the nurse notices the patient has a stooped posture, shuffling walk with short steps, flat facial expression, and pill-rolling finger movements. These findings would be consistent with: a. Parkinsonism. b. Cerebral palsy. c. Cerebellar ataxia. d. Muscular dystrophy. The stooped posture, shuffling walk, short steps, flat facial expression, and pill-rolling finger movements are all found in parkinsonism. (See Table 23-8 for more information and for the descriptions of the other options.) 44. During an assessment of a 32-year-old patient with a recent head injury, the nurse notices that the patient responds to pain by extending, adducting, and internally rotating his arms. His palms pronate, and his lower extremities extend with plantar flexion. Which statement concerning these findings is most accurate? This patients response: a. Indicates a lesion of the cerebral cortex. b. Indicates a completely nonfunctional brainstem. c. Is normal and will go away in 24 to 48 hours. d. Is a very ominous sign and may indicate brainstem injury. These findings are all indicative of decerebrate rigidity, which is a very ominous condition and may indicate a brainstem injury. 45. A 78-year-old man has a history of a cerebrovascular accident. The nurse notes that when he walks, his left arm is immobile against the body with flexion of the shoulder, elbow, wrist, and fingers and adduction of the shoulder. His left leg is stiff and extended and circumducts with each step. What type of gait disturbance is this individual experiencing? a. Scissors gait b. Cerebellar ataxia c. Parkinsonian gait d. Spastic hemiparesis With spastic hemiparesis, the arm is immobile against the body. Flexion of the shoulder, elbow, wrist, and fingers occurs, and adduction of the shoulder, which does not swing freely, is observed. The leg is stiff and extended and circumducts with each step. Causes of this type of gait include cerebrovascular accident. (See Table 23-6 for more information and for the descriptions of the other abnormal gaits.) 46. In a person with an upper motor neuron lesion such as a cerebrovascular accident, which of these physical assessment findings should the nurse expect? a. Hyperreflexia b. Fasciculations c. Loss of muscle tone and flaccidity d. Atrophy and wasting of the muscles Hyperreflexia, diminished or absent superficial reflexes, and increased muscle tone or spasticity can be expected with upper motor neuron lesions. The other options reflect a lesion of lower motor neurons (see Table 23-7). 47. A 59-year-old patient has a herniated intervertebral disk. Which of the following findings should the nurse expect to see on physical assessment of this individual? a. Hyporeflexia b. Increased muscle tone c. Positive Babinski sign d. Presence of pathologic reflexes With a herniated intervertebral disk or lower motor neuron lesion, loss of tone, flaccidity, atrophy, fasciculations, and hyporeflexia or areflexia are demonstrated. No Babinski sign or pathologic reflexes would be observed (see Table 23-7). The other options reflect a lesion of upper motor neurons. 48. A patient is unable to perform rapid alternating movements such as rapidly patting her knees. The nurse should document this inability as: a. Ataxia. b. Astereognosis. c. Presence of dysdiadochokinesia. d. Loss of kinesthesia. Slow clumsy movements and the inability to perform rapid alternating movements occur with cerebellar disease. The condition is termed dysdiadochokinesia. Ataxia is an uncoordinated or unsteady gait. Astereognosis is the inability to identify an object by feeling it. Kinesthesia is the persons ability to perceive passive movement of the extremities or the loss of position sense. 49. The nurse knows that determining whether a person is oriented to his or her surroundings will test the functioning of which structure(s)? a. Cerebrum b. Cerebellum c. CNs d. Medulla oblongata The cerebral cortex is responsible for thought, memory, reasoning, sensation, and voluntary movement. The other structures are not responsible for a persons level of consciousness. 50. During an examination, the nurse notices severe nystagmus in both eyes of a patient. Which conclusion by the nurse is correct? Severe nystagmus in both eyes: a. Is a normal occurrence. b. May indicate disease of the cerebellum or brainstem. c. Is a sign that the patient is nervous about the examination. d. Indicates a visual problem, and a referral to an ophthalmologist is indicated. - End-point nystagmus at an extreme lateral gaze normally occurs; however, the nurse should carefully assess any other nystagmuses. Severe nystagmus occurs with disease of the vestibular system, cerebellum, or brainstem. 51. The nurse knows that testing kinesthesia is a test of a persons: a. Fine touch. b. Position sense. c. Motor coordination. d. Perception of vibration. - Kinesthesia, or position sense, is the persons ability to perceive passive movements of the extremities. The other options are incorrect. 52. The nurse is reviewing a patients medical record and notes that he is in a coma. Using the Glasgow Coma Scale, which number indicates that the patient is in a coma? a. 6 b. 12 c. 15 d. 24 A fully alert, normal person has a score of 15, whereas a score of 7 or less reflects coma on the Glasgow Coma Scale (see Figure 23-59). 53. A man who was found wandering in a park at 2 AM has been brought to the emergency department for an examination; he said he fell and hit his head. During the examination, the nurse asks him to use his index finger to touch the nurses finger, then his own nose, then the nurses finger again (which has been moved to a different location). The patient is clumsy, unable to follow the instructions, and overshoots the mark, missing the finger. The nurse should suspect which of the following? a. Cerebral injury b. Cerebrovascular accident c. Acute alcohol intoxication d. Peripheral neuropathy During the finger-to-finger test, if the person has clumsy movement with overshooting the mark, either a cerebellar disorder or acute alcohol intoxication should be suspected. The persons movements should be smooth and accurate. The other options are not correct. 54. The nurse is assessing the neurologic status of a patient who has a late-stage brain tumor. With the reflex hammer, the nurse draws a light stroke up the lateral side of the sole of the foot and inward, across the ball of the foot. In response, the patients toes fan out, and the big toe shows dorsiflexion. The nurse interprets this result as: a. Negative Babinski sign, which is normal for adults. b. Positive Babinski sign, which is abnormal for adults. c. Clonus, which is a hyperactive response. d. Achilles reflex, which is an expected response. - Dorsiflexion of the big toe and fanning of all toes is a positive Babinski sign, also called up-going toes. This response occurs with upper motor neuron disease of the corticospinal (or pyramidal) tract and is an abnormal finding for adults. 1. A 69-year-old patient has been admitted to an adult psychiatric unit because his wife thinks he is getting more and more confused. He laughs when he is found to be forgetful, saying Im just getting old! After the nurse completes a thorough neurologic assessment, which findings would be indicative of Alzheimer disease? Select all that apply. a. Occasionally forgetting names or appointments b. Difficulty performing familiar tasks, such as placing a telephone call c. Misplacing items, such as putting dish soap in the refrigerator d. Sometimes having trouble finding the right word e. Rapid mood swings, from calm to tears, for no apparent reason f. Getting lost in ones own neighborhood - , C, E, F Difficulty performing familiar tasks, misplacing items, rapid mood swings, and getting lost in ones own neighborhood can be warning signs of Alzheimer disease. Occasionally forgetting names or appointments, and sometimes having trouble finding the right word are part of normal aging. (For other examples of Alzheimer disease, see Table 23-2.) 1. During an examination, the nurse can assess mental status by which activity? a. Examining the patients electroencephalogram b. Observing the patient as he or she performs an intelligence quotient (IQ) test c. Observing the patient and inferring health or dysfunction d. Examining the patients response to a specific set of questions Mental status cannot be directly scrutinized like the characteristics of skin or heart sounds. Its functioning is inferred through an assessment of an individuals behaviors, such as consciousness, language, mood and affect, and other aspects. 2. The nurse is assessing the mental status of a child. Which statement about children and mental status is true? a. All aspects of mental status in children are interdependent. b. Children are highly labile and unstable until the age of 2 years. c. Childrens mental status is largely a function of their parents level of functioning until the age of 7 years. d. A childs mental status is impossible to assess until the child develops the ability to concentrate. Separating and tracing the development of only one aspect of mental status is difficult. All aspects are interdependent. For example, consciousness is rudimentary at birth because the cerebral cortex is not yet developed. The infant cannot distinguish the self from the mothers body. The other statements are not true. 3. The nurse is assessing a 75-year-old man. As the nurse begins the mental status portion of the assessment, the nurse expects that this patient: a. Will have no decrease in any of his abilities, including response time. b. Will have difficulty on tests of remote memory because this ability typically decreases with age. c. May take a little longer to respond, but his general knowledge and abilities should not have declined. d. Will exhibit had a decrease in his response time because of the loss of language and a decrease in general knowledge. The aging process leaves the parameters of mental status mostly intact. General knowledge does not decrease, and little or no loss in vocabulary occurs. Response time is slower than in a youth. It takes a little longer for the brain to process information and to react to it. Recent memory, which requires some processing, is somewhat decreased with aging, but remote memory is not affected. 4. When assessing aging adults, the nurse knows that one of the first things that should be assessed before making judgments about their mental status is: a. Presence of phobias b. General intelligence c. Presence of irrational thinking patterns d. Sensory-perceptive abilities Age-related changes in sensory perception can affect mental status. For example, vision loss (as detailed in Chapter 14) may result in apathy, social isolation, and depression. Hearing changes are common in older adults, which produces frustration, suspicion, and social isolation and makes the person appear confused. 5. The nurse is preparing to conduct a mental status examination. Which statement is true regarding the mental status examination? a. A patients family is the best resource for information about the patients coping skills. b. Gathering mental status information during the health history interview is usually sufficient. c. Integrating the mental status examination into the health history interview takes an enormous amount of extra time. d. To get a good idea of the patients level of functioning, performing a complete mental status examination is usually necessary. - The full mental status examination is a systematic check of emotional and cognitive functioning. The steps described, however, rarely need to be taken in their entirety. Usually, one can assess mental status through the context of the health history interview. 6. A woman brings her husband to the clinic for an examination. She is particularly worried because after a recent fall, he seems to have lost a great deal of his memory of recent events. Which statement reflects the nurses best course of action? a. Perform a complete mental status examination. b. Refer him to a psychometrician. c. Plan to integrate the mental status examination into the history and physical examination. d. Reassure his wife that memory loss after a physical shock is normal and will soon subside. Performing a complete mental status examination is necessary when any abnormality in affect or behavior is discovered or when family members are concerned about a persons behavioral changes (e.g., memory loss, inappropriate social interaction) or after trauma, such as a head injury. 7. The nurse is conducting a patient interview. Which statement made by the patient should the nurse more fully explore during the interview? a. I sleep like a baby. b. I have no health problems. c. I never did too good in school. d. I am not currently taking any medications. In every mental status examination, the following factors from the health history that could affect the findings should be noted: any known illnesses or health problems, such as alcoholism or chronic renal disease; current medications, the side effects of which may cause confusion or depression; the usual educational and behavioral level, noting this level as the patients normal baseline and not expecting a level of performance on the mental status examination to exceed it; and responses to personal history questions, indicating current stress, social interaction patterns, and sleep habits. 8. A patient is admitted to the unit after an automobile accident. The nurse begins the mental status examination and finds that the patient has dysarthric speech and is lethargic. The nurses best approach regarding this examination is to: a. Plan to defer the rest of the mental status examination. b. Skip the language portion of the examination, and proceed onto assessing mood and affect. c. Conduct an in-depth speech evaluation, and defer the mental status examination to another time. d. Proceed with the examination, and assess the patient for suicidal thoughts because dysarthria is often accompanied by severe depression. In the mental status examination, the sequence of steps forms a hierarchy in which the most basic functions (consciousness, language) are assessed first. The first steps must be accurately assessed to ensure validity of the steps that follow. For example, if consciousness is clouded, then the person cannot be expected to have full attention and to cooperate with new learning. If language is impaired, then a subsequent assessment of new learning or abstract reasoning (anything that requires language functioning) can give erroneous conclusions. 9. A 19-year-old woman comes to the clinic at the insistence of her brother. She is wearing black combat boots and a black lace nightgown over the top of her other clothes. Her hair is dyed pink with black streaks throughout. She has several pierced holes in her nares and ears and is wearing an earring through her eyebrow and heavy black makeup. The nurse concludes that: a. She probably does not have any problems. b. She is only trying to shock people and that her dress should be ignored. c. She has a manic syndrome because of her abnormal dress and grooming. d. More information should be gathered to decide whether her dress is appropriate. Grooming and hygiene should be notedthe person is clean and well groomed, hair is neat and clean, women have moderate or no makeup, and men are shaved or their beards or moustaches are well groomed. Care should be taken when interpreting clothing that is disheveled, bizarre, or in poor repair because these sometimes reflect the persons economic status or a deliberate fashion trend. 10. A patient has been in the intensive care unit for 10 days. He has just been moved to the medical-surgical unit, and the admitting nurse is planning to perform a mental status examination. During the tests of cognitive function, the nurse would expect that he: a. May display some disruption in thought content. b. Will state, I am so relieved to be out of intensive care. c. Will be oriented to place and person, but the patient may not be certain of the date. d. May show evidence of some clouding of his level of consciousness. The nurse can discern the orientation of cognitive function through the course of the interview or can directly and tactfully ask, Some people have trouble keeping up with the dates while in the hospital. Do you know todays date? Many hospitalized people have trouble with the exact date but are fully oriented on the remaining items. 11. During a mental status examination, the nurse wants to assess a patients affect. The nurse should ask the patient which question? a. How do you feel today? b. Would you please repeat the following words? c. Have these medications had any effect on your pain? d. Has this pain affected your ability to get dressed by yourself? Judge mood and affect by body language and facial expression and by directly asking, How do you feel today? or How do you usually feel? The mood should be appropriate to the persons place and condition and should appropriately change with the topics. 12. The nurse is planning to assess new memory with a patient. The best way for the nurse to do this would be to: a. Administer the FACT test. b. Ask him to describe his first job. c. Give him the Four Unrelated Words Test. d. Ask him to describe what television show he was watching before coming to the clinic. Ask questions that can be corroborated, which screens for the occasional person who confabulates or makes up answers to fill in the gaps of memory loss. The Four Unrelated Words Test tests the persons ability to lay down new memories and is a highly sensitive and valid memory test. 13. A 45-year-old woman is at the clinic for a mental status assessment. In giving her the Four Unrelated Words Test, the nurse would be concerned if she could not ____ four unrelated words ____. a. Invent; within 5 minutes b. Invent; within 30 seconds c. Recall; after a 30-minute delay d. Recall; after a 60-minute delay The Four Unrelated Words Test tests the persons ability to lay down new memories. It is a highly sensitive and valid memory test. It requires more effort than the recall of personal or historic events. To the person say, I am going to say four words. I want you to remember them. In a few minutes I will ask you to recall them. After 5 minutes, ask for the four words. The normal response for persons under 60 years is an accurate three- or four-word recall after a 5-, 10-, and 30-minute delay. 14. During a mental status assessment, which question by the nurse would best assess a persons judgment? a. Do you feel that you are being watched, followed, or controlled? b. Tell me what you plan to do once you are discharged from the hospital. c. What does the statement, People in glass houses shouldnt throw stones, mean to you? d. What would you do if you found a stamped, addressed envelope lying on the sidewalk? - A person exercises judgment when he or she can compare and evaluate the alternatives in a situation and reach an appropriate course of action. Rather than testing the persons response to a hypothetical situation (as illustrated in the option with the envelope), the nurse should be more interested in the persons judgment about daily or long-term goals, the likelihood of acting in response to delusions or hallucinations, and the capacity for violent or suicidal behavior. 15. Which of these individuals would the nurse consider at highest risk for a suicide attempt? a. Man who jokes about death b. Woman who, during a past episode of major depression, attempted suicide c. Adolescent who just broke up with her boyfriend and states that she would like to kill herself d. Older adult man who tells the nurse that he is going to join his wife in heaven tomorrow and plans to use a gun When the person expresses feelings of sadness, hopelessness, despair, or grief, assessing any possible risk of physical harm to him or herself is important. The interview should begin with more general questions. If the nurse hears affirmative answers, then he or she should continue with more specific questions. A precise suicide plan to take place in the next 24 to 48 hours with use of a lethal method constitutes high risk. 16. The nurse is performing a mental status assessment on a 5-year-old girl. Her parents are undergoing a bitter divorce and are worried about the effect it is having on their daughter. Which action or statement might lead the nurse to be concerned about the girls mental status? a. She clings to her mother whenever the nurse is in the room. b. She appears angry and will not make eye contact with the nurse. c. Her mother states that she has begun to ride a tricycle around their yard. d. Her mother states that her daughter prefers to play with toddlers instead of kids her own age while in daycare. The mental status assessment of infants and children covers behavioral, cognitive, and psychosocial development and examines how the child is coping with his or her environment. Essentially, the nurse should follow the same Association for Behavioral and Cognitive Therapies (ABCT) guidelines as those for the adult, with special consideration for developmental milestones. The best examination technique arises from a thorough knowledge of the developmental milestones (described in Chapter 2). Abnormalities are often problems of omission (e.g., the child does not achieve a milestone as expected). 17. The nurse is assessing orientation in a 79-year-old patient. Which of these responses would lead the nurse to conclude that this patient is oriented? a. I know my name is John. I couldnt tell you where I am. I think it is 2010, though. b. I know my name is John, but to tell you the truth, I get kind of confused about the date. c. I know my name is John; I guess Im at the hospital in Spokane. No, I dont know the date. d. I know my name is John. I am at the hospital in Spokane. I couldnt tell you what date it is, but I know that it is February of a new year2010. Many aging persons experience social isolation, loss of structure without a job, a change in residence, or some short-term memory loss. These factors affect orientation, and the person may not provide the precise date or complete name of the agency. You may consider aging persons oriented if they generally know where they are and the present period. They should be considered oriented to time if the year and month are correctly stated. Orientation to place is accepted with the correct identification of the type of setting (e.g., hospital) and the name of the town. 18. The nurse is performing the Denver II screening test on a 12-month-old infant during a routine well-child visit. The nurse should tell the infants parents that the Denver II: a. Tests three areas of development: cognitive, physical, and psychological b. Will indicate whether the child has a speech disorder so that treatment can begin. c. Is a screening instrument designed to detect children who are slow in development. d. Is a test to determine intellectual ability and may indicate whether problems will develop later in school. The Denver II is a screening instrument designed to detect developmental delays in infants and preschoolers. It tests four functions: gross motor, language, fine motor-adaptive, and personal-social. The Denver II is not an intelligence test; it does not predict current or future intellectual ability. It is not diagnostic; it does not suggest treatment regimens. 19. A patient drifts off to sleep when she is not being stimulated. The nurse can easily arouse her by calling her name, but the patient remains drowsy during the conversation. The best description of this patients level of consciousness would be: a. Lethargic b. Obtunded c. Stuporous d. Semialert Lethargic (or somnolent) is when the person is not fully alert, drifts off to sleep when not stimulated, and can be aroused when called by name in a normal voice but looks drowsy. He or she appropriately responds to questions or commands, but thinking seems slow and fuzzy. He or she is inattentive and loses the train of thought. Spontaneous movements are decreased. (See Table 5-3 for the definitions of the other terms.) 20. A patient has had a cerebrovascular accident (stroke). He is trying very hard to communicate. He seems driven to speak and says, I buy obie get spirding and take my train. What is the best description of this patients problem? a. Global aphasia b. Brocas aphasia c. Echolalia d. Wernickes aphasia This type of communication illustrates Wernickes or receptive aphasia. The person can hear sounds and words but cannot relate them to previous experiences. Speech is fluent, effortless, and well articulated, but it has many paraphasias (word substitutions that are malformed or wrong) and neologisms (made-up words) and often lacks substantive words. Speech can be totally incomprehensible. Often, a great urge to speak is present. Repetition, reading, and writing also are impaired. Echolalia is an imitation or the repetition of another persons words or phrases. (See Table 5-4 for the definitions of the other disorders.) 21. A patient repeatedly seems to have difficulty coming up with a word. He says, I was on my way to work, and when I got there, the thing that you step into that goes up in the air was so full that I decided to take the stairs. The nurse will note on his chart that he is using or experiencing: a. Blocking b. Neologism c. Circumlocution d. Circumstantiality Circumlocution is a roundabout expression, substituting a phrase when one cannot think of the name of the object. 22. During an examination, the nurse notes that a patient is exhibiting flight of ideas. Which statement by the patient is an example of flight of ideas? a. My stomach hurts. Hurts, spurts, burts. b. Kiss, wood, reading, ducks, onto, maybe. c. Take this pill? The pill is red. I see red. Red velvet is soft, soft as a babys bottom. d. I wash my hands, wash them, wash them. I usually go to the sink and wash my hands. Flight of ideas is demonstrated by an abrupt change, rapid skipping from topic to topic, and practically continuous flow of accelerated speech. Topics usually have recognizable associations or are plays on words. 23. A patient describes feeling an unreasonable, irrational fear of snakes. His fear is so persistent that he can no longer comfortably look at even pictures of snakes and has made an effort to identify all the places he might encounter a snake and avoids them. The nurse recognizes that he: a. Has a snake phobia. b. Is a hypochondriac; snakes are usually harmless. c. Has an obsession with snakes. d. Has a delusion that snakes are harmful, which must stem from an early traumatic incident involving snakes. A phobia is a strong, persistent, irrational fear of an object or situation; the person feels driven to avoid it. (See Table 5-7 for the definitions of the other terms.) 24. A patient has been diagnosed with schizophrenia. During a recent interview, he shows the nurse a picture of a man holding a decapitated head. He describes this picture as horrifying but then laughs loudly at the content. This behavior is a display of: a. Confusion b. Ambivalence c. Depersonalization d. Inappropriate affect An inappropriate affect is an affect clearly discordant with the content of the persons speech. (See Table 5-5 for the definitions of the other terms.) 25. During reporting, the nurse hears that a patient is experiencing hallucinations. Which is an example of a hallucination? a. Man believes that his dead wife is talking to him. b. Woman hears the doorbell ring and goes to answer it, but no one is there. c. Child sees a man standing in his closet. When the lights are turned on, it is only a dry cleaning bag. d. Man believes that the dog has curled up on the bed, but when he gets closer he sees that it is a blanket. Hallucinations are sensory perceptions for which no external stimuli exist. They may strike any sense: visual, auditory, tactile, olfactory, or gustatory. 26. A 20-year-old construction worker has been brought into the emergency department with heat stroke. He has delirium as a result of a fluid and electrolyte imbalance. For the mental status examination, the nurse should first assess the patients: a. Affect and mood b. Memory and affect c. Language abilities d. Level of consciousness and cognitive abilities Delirium is a disturbance of consciousness (i.e., reduced clarity of awareness of the environment) with reduced ability to focus, sustain, or shift attention. Delirium is not an alteration in mood, affect, or language abilities. 27. A patient states, I feel so sad all of the time. I cant feel happy even doing things I used to like to do. He also states that he is tired, sleeps poorly, and has no energy. To differentiate between a dysthymic disorder and a major depressive disorder, the nurse should ask which question? a. Have you had any weight changes? b. Are you having any thoughts of suicide? c. How long have you been feeling this way? d. Are you having feelings of worthlessness? Major depressive disorder is characterized by one or more major depressive episodes, that is, at least 2 weeks of depressed mood or loss of interest accompanied by at least four additional symptoms of depression. Dysthymic disorder is characterized by at least 2 years of depressed mood for more days than not, accompanied by additional depressive symptoms. 28. A 26-year-old woman was robbed and beaten a month ago. She is returning to the clinic today for a follow-up assessment. The nurse will want to ask her which one of these questions? a. How are things going with the trial? b. How are things going with your job? c. Tell me about your recent engagement! d. Are you having any disturbing dreams? In posttraumatic stress disorder, the person has been exposed to a traumatic event. The traumatic event is persistently reexperienced by recurrent and intrusive, distressing recollections of the event, including images, thoughts, or perceptions; recurrent distressing dreams of the event; and acting or feeling as if the traumatic event were recurring. 29. The nurse is performing a mental status examination. Which statement is true regarding the assessment of mental status? a. Mental status assessment diagnoses specific psychiatric disorders. b. Mental disorders occur in response to everyday life stressors. c. Mental status functioning is inferred through the assessment of an individuals behaviors. d. Mental status can be directly assessed, similar to other systems of the body (e.g., heart sounds, breath sounds). Mental status functioning is inferred through the assessment of an individuals behaviors. It cannot be directly assessed like the characteristics of the skin or heart sounds. 30. A 23-year-old patient in the clinic appears anxious. Her speech is rapid, and she is fidgety and in constant motion. Which of these questions or statements would be most appropriate for the nurse to use in this situation to assess attention span? a. How do you usually feel? Is this normal behavior for you? b. I am going to say four words. In a few minutes, I will ask you to recall them. c. Describe the meaning of the phrase, Looking through rose-colored glasses. d. Pick up the pencil in your left hand, move it to your right hand, and place it on the table. Attention span is evaluated by assessing the individuals ability to concentrate and complete a thought or task without wandering. Giving a series of directions to follow is one method used to assess attention span. 31. The nurse is planning health teaching for a 65-year-old woman who has had a cerebrovascular accident (stroke) and has aphasia. Which of these questions is most important to use when assessing mental status in this patient? a. Please count backward from 100 by seven. b. I will name three items and ask you to repeat them in a few minutes. c. Please point to articles in the room and parts of the body as I name them. d. What would you do if you found a stamped, addressed envelope on the sidewalk? Additional tests for persons with aphasia include word comprehension (asking the individual to point to articles in the room or parts of the body), reading (asking the person to read available print), and writing (asking the person to make up and write a sentence). 32. A 30-year-old female patient is describing feelings of hopelessness and depression. She has attempted self-mutilation and has a history of suicide attempts. She describes difficulty sleeping at night and has lost 10 pounds in the past month. Which of these statements or questions is the nurses best response in this situation? a. Do you have a weapon? b. How do other people treat you? c. Are you feeling so hopeless that you feel like hurting yourself now? d. People often feel hopeless, but the feelings resolve within a few weeks. When the person expresses feelings of hopelessness, despair, or grief, assessing the risk of physical harm to him or herself is important. This process begins with more general questions. If the answers are affirmative, then the assessment continues with more specific questions. 33. The nurse is providing instructions to newly hired graduates for the minimental state examination (MMSE). Which statement best describes this examination? a. Scores below 30 indicate cognitive impairment. b. The MMSE is a good tool to evaluate mood and thought processes. c. This examination is a good tool to detect delirium and dementia and to differentiate these from psychiatric mental illness. d. The MMSE is useful tool for an initial evaluation of mental status. Additional tools are needed to evaluate cognition changes over time. The MMSE is a quick, easy test of 11 questions and is used for initial and serial evaluations and can demonstrate a worsening or an improvement of cognition over time and with treatment. It evaluates cognitive functioning, not mood or thought processes. MMSE is a good screening tool to detect dementia and delirium and to differentiate these from psychiatric mental illness. 34. The nurse discovers speech problems in a patient during an assessment. The patient has spontaneous speech, but it is mostly absent or is reduced to a few stereotypical words or sounds. This finding reflects which type of aphasia? a. Global b. Brocas c. Dysphonic d. Wernickes Global aphasia is the most common and severe form of aphasia. Spontaneous speech is absent or reduced to a few stereotyped words or sounds, and prognosis for language recovery is poor. (Brocas aphasia and Wernickes aphasia are described in Table 5-4.) Dysphonic aphasia is not a valid condition. 35. A patient repeats, I feel hot. Hot, cot, rot, tot, got. Im a spot. The nurse documents this as an illustration of: a. Blocking b. Clanging c. Echolalia d. Neologism - Clanging is word choice based on sound, not meaning, and includes nonsense rhymes and puns. (See Table 5-6 for the definitions of the other terms.) 36. During an interview, the nurse notes that the patient gets up several times to wash her hands even though they are not dirty. This behavior is an example of: a. Social phobia b. Compulsive disorder c. Generalized anxiety disorder d. Posttraumatic stress disorder - Repetitive behaviors, such as handwashing, are behaviors that the person feels driven to perform in response to an obsession. The behaviors are aimed at preventing or reducing distress or preventing some dreaded event or situation. 37. The nurse is administering a Mini-Cog test to an older adult woman. When asked to draw a clock showing the time of 10:45, the patient drew a clock with the numbers out of order and with an incorrect time. This result indicates which finding? a. Cognitive impairment b. Amnesia c. Delirium d. Attention-deficit disorder The Mini-Cog is a newer instrument that screens for cognitive impairment, often found with dementia. The result of an abnormal drawing of a clock and time indicates a cognitive impairment. 38. During morning rounds, the nurse asks a patient, How are you today? The patient responds, You today, you today, you today! and mumbles the words. This speech pattern is an example of: a. Echolalia b. Clanging c. Word salad d. Perseveration Echolalia occurs when a person imitates or repeats anothers words or phrases, often with a mumbling, mocking, or a mechanical tone. 1. The nurse is assessing a patient who is admitted with possible delirium. Which of these are manifestations of delirium? Select all that apply. a. Develops over a short period. b. Person is experiencing apraxia. c. Person is exhibiting memory impairment or deficits. d. Occurs as a result of a medical condition, such as systemic infection. e. Person is experiencing agnosia. , C, D Delirium is a disturbance of consciousness that develops over a short period and may be attributable to a medical condition. Memory deficits may also occur. Apraxia and agnosia occur with dementia. 1. When evaluating a patients pain, the nurse knows that an example of acute pain would be: a. Arthritic pain. b. Fibromyalgia. c. Kidney stones. d. Low back pain. Acute pain is short-term and dissipates after an injury heals, such as with kidney stones. The other conditions are examples of chronic pain during which the pain continues for 6 months or longer and does not stop when the injury heals. 2. Which statement indicates that the nurse understands the pain experienced by an older adult? a. Older adults must learn to tolerate pain. b. Pain is a normal process of aging and is to be expected. c. Pain indicates a pathologic condition or an injury and is not a normal process of aging. d. Older individuals perceive pain to a lesser degree than do younger individuals. Pain indicates a pathologic condition or an injury and should never be considered something that an older adult should expect or tolerate. Pain is not a normal process of aging, and no evidence suggests that pain perception is reduced with aging. 3. A 4-year-old boy is brought to the emergency department by his mother. She says he points to his stomach and says, It hurts so bad. Which pain assessment tool would be the best choice when assessing this childs pain? a. Descriptor Scale b. Numeric rating scale c. Brief Pain Inventory d. Faces Pain ScaleRevised (FPS-R) Rating scales can be introduced at the age of 4 or 5 years. The FPS-R is designed for use by children and asks the child to choose a face that shows how much hurt (or pain) you have now. Young children should not be asked to rate pain by using numbers. 4. A patient states that the pain medication is not working and rates his postoperative pain at a 10 on a 1-to-10 scale. Which of these assessment findings indicates an acute pain response to poorly controlled pain? a. Confusion b. Hyperventilation c. Increased blood pressure and pulse d. Depression Responses to poorly controlled acute pain include tachycardia, elevated blood pressure, and hypoventilation. Confusion and depression are associated with poorly controlled chronic pain (see Table 10-1). 5. A 60-year-old woman has developed reflexive sympathetic dystrophy after arthroscopic repair of her shoulder. A key feature of this condition is that the: a. Affected extremity will eventually regain its function. b. Pain is felt at one site but originates from another location. c. Patients pain will be associated with nausea, pallor, and diaphoresis. d. Slightest touch, such as a sleeve brushing against her arm, causes severe and intense pain. A key feature of reflexive sympathetic dystrophy is that a typically innocuous stimulus can create a severe, intensely painful response. The affected extremity becomes less functional over time. 6. The nurse is assessing a patients pain. The nurse knows that the most reliable indicator of pain would be the: a. Patients vital signs. b. Physical examination. c. Results of a computerized axial tomographic scan. d. Subjective report. The subjective report is the most reliable indicator of pain. Physical examination findings can lend support, but the clinician cannot exclusively base the diagnosis of pain on physical assessment findings. 7. A patient has had arthritic pain in her hips for several years since a hip fracture. She is able to move around in her room and has not offered any complaints so far this morning. However, when asked, she states that her pain is bad this morning and rates it at an 8 on a 1-to-10 scale. What does the nurse suspect? The patient: a. Is addicted to her pain medications and cannot obtain pain relief. b. Does not want to trouble the nursing staff with her complaints. c. Is not in pain but rates it high to receive pain medication. d. Has experienced chronic pain for years and has adapted to it. Persons with chronic pain typically try to give little indication that they are in pain and, over time, adapt to the pain. As a result, they are at risk for underdetection. 8. The nurse is reviewing the principles of pain. Which type of pain is due to an abnormal processing of the pain impulse through the peripheral or central nervous system? a. Visceral b. Referred c. Cutaneous d. Neuropathic Neuropathic pain implies an abnormal processing of the pain message. The other types of pain are named according to their sources 9. When assessing the quality of a patients pain, the nurse should ask which question? a. When did the pain start? b. Is the pain a stabbing pain? c. Is it a sharp pain or dull pain? d. What does your pain feel like? To assess the quality of a persons pain, the patient is asked to describe the pain in his or her own words. 10. When assessing a patients pain, the nurse knows that an example of visceral pain would be: a. Hip fracture. b. Cholecystitis. c. Second-degree burns. d. Pain after a leg amputation. - Visceral pain originates from the larger interior organs, such as the gallbladder, liver, or kidneys. 11. The nurse is reviewing the principles of nociception. During which phase of nociception does the conscious awareness of a painful sensation occur? a. Perception b. Modulation c. Transduction d. Transmission Perception is the third phase of nociception and indicates the conscious awareness of a painful sensation. During this phase, the sensation is recognized by higher cortical structures and identified as pain. 12. When assessing the intensity of a patients pain, which question by the nurse is appropriate? a. What makes your pain better or worse? b. How much pain do you have now? c. How does pain limit your activities? d. What does your pain feel like? - Asking the patient how much pain do you have? is an assessment of the intensity of a patients pain; various intensity scales can be used. Asking what makes ones pain better or worse assesses alleviating or aggravating factors. Asking whether pain limits ones activities assesses the degree of impairment and quality of life. Asking what does your pain feel like assesses the quality of pain. 13. A patient is complaining of severe knee pain after twisting it during a basketball game and is requesting pain medication. Which action by the nurse is appropriate? a. Completing the physical examination first and then giving the pain medication b. Telling the patient that the pain medication must wait until after the x-ray images are completed c. Evaluating the full range of motion of the knee and then medicating for pain d. Administering pain medication and then proceeding with the assessment According to the American Pain Society (1992), In cases in which the cause of acute pain is uncertain, establishing a diagnosis is a priority, but symptomatic treatment of pain should be given while the investigation is proceeding. With occasional exceptions, (e.g., the initial examination of the patient with an acute condition of the abdomen), it is rarely justified to defer analgesia until a diagnosis is made. In fact, a comfortable patient is better able to cooperate with diagnostic procedures. 14. The nurse knows that which statement is true regarding the pain experienced by infants? a. Pain in infants can only be assessed by physiologic changes, such as an increased heart rate. b. The FPS-R can be used to assess pain in infants. c. A procedure that induces pain in adults will also induce pain in the infant. d. Infants feel pain less than do adults. If a procedure or disease process causes pain in an adult, then it will also cause pain in an infant. Physiologic changes cannot be exclusively used to confirm or deny pain because other factors, such as medications, fluid status, or stress may cause physiologic changes. The FPS-R can be used starting at age 4 years. 15. A patient has been admitted to the hospital with vertebral fractures related to osteoporosis. She is in extreme pain. This type of pain would be classified as: a. Referred. b. Cutaneous. c. Visceral. d. Deep somatic. Deep somatic pain comes from sources such as the blood vessels, joints, tendons, muscles, and bone. Referred pain is felt at one site but originates from another location. Cutaneous pain is derived from the skin surface and subcutaneous tissues. Visceral pain originates from the larger, interior organs. 1. During assessment of a patients pain, the nurse is aware that certain nonverbal behaviors are associated with chronic pain. Which of these behaviors are associated with chronic pain? Select all that apply. a. Sleeping b. Moaning c. Diaphoresis d. Bracing e. Restlessness f. Rubbing , D, F Behaviors that have been associated with chronic pain include bracing, rubbing, diminished activity, sighing, and changes in appetite. In addition, those with chronic pain may sleep in an attempt at distraction. The other behaviors are associated with acute pain. 2. During an admission assessment of a patient with dementia, the nurse assesses for pain because the patient has recently had several falls. Which of these are appropriate for the nurse to assess in a patient with dementia? Select all that apply. a. Ask the patient, Do you have pain? b. Assess the patients breathing independent of vocalization. c. Note whether the patient is calling out, groaning, or crying. d. Have the patient rate pain on a 1-to-10 scale. e. Observe the patients body language for pacing and agitation. The nurse educator is preparing an education module for the nursing staff on the epidermal layer of skin. Which of these statements would be included in the module? The epidermis is: a.Highly vascular. b.Thick and tough. c.Thin and nonstratified. d.Replaced every 4 weeks. The nurse educator is preparing an education module for the nursing staff on the dermis layer of skin. Which of these statements would be included in the module? The dermis: a.Contains mostly fat cells. b.Consists mostly of keratin. c.Is replaced every 4 weeks. d.Contains sensory receptors. The nurse is examining a patient who tells the nurse, "I sure sweat a lot, especially on my face and feet but it doesn't have an odor." The nurse knows that this condition could be related to: a.Eccrine glands. b.Apocrine glands. c.Disorder of the stratum corneum. d.Disorder of the stratum germinativum A newborn infant is in the clinic for a well-baby checkup. The nurse observes the infant for the possibility of fluid loss because of which of these factors? a.Subcutaneous fat deposits are high in the newborn. b.Sebaceous glands are overproductive in the newborn. c.The newborn's skin is more permeable than that of the adult. d.The amount of vernix caseosa dramatically rises in the newborn. The nurse is bathing an 80-year-old man and notices that his skin is wrinkled, thin, lax, and dry. This finding would be related to which factor in the older adult? a.Increased vascularity of the skin b.Increased numbers of sweat and sebaceous glands c.An increase in elastin and a decrease in subcutaneous fat d.An increased loss of elastin and a decrease in subcutaneous fat During the aging process, the hair can look gray or white and begin to feel thin and fine. The nurse knows that this occurs because of a decrease in the number of functioning: a.Metrocytes. b.Fungacytes. c.Phagocytes. d.Melanocytes. During an examination, the nurse finds that a patient has excessive dryness of the skin. The best term to describe this condition is: a.Xerosis. b.Pruritus. c.Alopecia. d.Seborrhea. A 22-year-old woman comes to the clinic because of severe sunburn and states, "I was out in the sun for just a couple of minutes." The nurse begins a medication review with her, paying special attention to which medication class? a.Nonsteroidal antiinflammatory drugs for pain b.Tetracyclines for acne c.Proton pump inhibitors for heartburn d.Thyroid replacement hormone for hypothyroidism A woman is leaving on a trip to Hawaii and has come in for a checkup. During the examination the nurse learns that she has diabetes and takes oral hypoglycemic agents. The patient needs to be concerned about which possible effect of her medications? a.Increased possibility of bruising b.Skin sensitivity as a result of exposure to salt water c.Lack of availability of glucose-monitoring supplies d.Importance of sunscreen and avoiding direct sunlight A 13-year-old girl is interested in obtaining information about the cause of her acne. The nurse should share with her that acne: a.Is contagious. b.Has no known cause. c.Is caused by increased sebum production. d.Has been found to be related to poor hygiene. . A 75-year-old woman who has a history of diabetes and peripheral vascular disease has been trying to remove a corn on the bottom of her foot with a pair of scissors. The nurse will encourage her to stop trying to remove the corn with scissors because: a.The woman could be at increased risk for infection and lesions because of her chronic disease. b.With her diabetes, she has increased circulation to her foot, and it could cause severe bleeding. c.She is 75 years old and is unable to see; consequently, she places herself at greater risk for self-injury with the scissors. d.With her peripheral vascular disease, her range of motion is limited and she may not be able to reach the corn safely. The nurse keeps in mind that a thorough skin assessment is extremely important because the skin holds information about a person's: a.Support systems. b.Circulatory status. c.Socioeconomic status. d.Psychological wellness. A patient comes in for a physical examination and complains of "freezing to death" while waiting for her examination. The nurse notes that her skin is pale and cool and attributes this finding to: a.Venous pooling. b.Peripheral vasodilation. c.Peripheral vasoconstriction. d.Decreased arterial perfusion. A patient comes to the clinic and tells the nurse that he has been confined to his recliner chair for approximately 3 days with his feet down and he asks the nurse to evaluate his feet. During the assessment, the nurse might expect to find: a.Pallor b.Coolness c.Distended veins d.Prolonged capillary filling time - A patient is especially worried about an area of skin on her feet that has turned white. The health care provider has told her that her condition is vitiligo. The nurse explains to her that vitiligo is: a.Caused by an excess of melanin pigment b.Caused by an excess of apocrine glands in her feet c.Caused by the complete absence of melanin pigment d.Related to impetigo and can be treated with an ointment A patient tells the nurse that he has noticed that one of his moles has started to burn and bleed. When assessing his skin, the nurse pays special attention to the danger signs for pigmented lesions and is concerned with which additional finding? a.Color variation b.Border regularity c.Symmetry of lesions d.Diameter of less than 6 mm A patient comes to the clinic and states that he has noticed that his skin is redder than normal. The nurse understands that this condition is due to hyperemia and knows that it can be caused by: a.Decreased amounts of bilirubin in the blood b.Excess blood in the underlying blood vessels c.Decreased perfusion to the surrounding tissues d.Excess blood in the dilated superficial capillaries During a skin assessment, the nurse notices that a Mexican-American patient has skin that is yellowish-brown; however, the skin on the hard and soft palate is pink and the patient's scleras are not yellow. From this finding, the nurse could probably rule out: a.Pallor b.Jaundice c.Cyanosis d.Iron deficiency A black patient is in the intensive care unit because of impending shock after an accident. The nurse expects to find what characteristics in this patient's skin? a.Ruddy blue. b.Generalized pallor. c.Ashen, gray, or dull. d.Patchy areas of pallor. An older adult woman is brought to the emergency department after being found lying on the kitchen floor for 2 days; she is extremely dehydrated. What would the nurse expect to see during the examination? a.Smooth mucous membranes and lips b.Dry mucous membranes and cracked lips c.Pale mucous membranes d.White patches on the mucous membranes A 42-year-old woman complains that she has noticed several small, slightly raised, bright red dots on her chest. On examination, the nurse expects that the spots are probably: a.Anasarca. b.Scleroderma. c.Senile angiomas. d.Latent myeloma. A 65-year-old man with emphysema and bronchitis has come to the clinic for a follow-up appointment. On assessment, the nurse might expect to see which finding? a.Anasarca b.Scleroderma c.Pedal erythema d.Clubbing of the nails A newborn infant has Down syndrome. During the skin assessment, the nurse notices a transient mottling in the trunk and extremities in response to the cool temperature in the examination room. The infant's mother also notices the mottling and asks what it is. The nurse knows that this mottling is called: a.Café au lait. b.Carotenemia. c.Acrocyanosis. d.Cutis marmorata. A 35-year-old pregnant woman comes to the clinic for a monthly appointment. During the assessment, the nurse notices that she has a brown patch of hyperpigmentation on her face. The nurse continues the skin assessment aware that another finding may be: a.Keratoses. b.Xerosis. c.Chloasma. d.Acrochordons. A man has come in to the clinic for a skin assessment because he is worried he might have skin cancer. During the skin assessment the nurse notices several areas of pigmentation that look greasy, dark, and "stuck on" his skin. Which is the best prediction? a.Senile lentigines, which do not become cancerous b.Actinic keratoses, which are precursors to basal cell carcinoma c.Acrochordons, which are precursors to squamous cell carcinoma d.Seborrheic keratoses, which do not become cancerous A 70-year-old woman who loves to garden has small, flat, brown macules over her arms and hands. She asks, "What causes these liver spots?" The nurse tells her, "They are: a."Signs of decreased hematocrit related to anemia." b."Due to the destruction of melanin in your skin from exposure to the sun." c."Clusters of melanocytes that appear after extensive sun exposure." d."Areas of hyperpigmentation related to decreased perfusion and vasoconstriction." The nurse notices that a patient has a solid, elevated, circumscribed lesion that is less than 1 cm in diameter. When documenting this finding, the nurse reports this as a: a.Bulla. b.Wheal. c.Nodule. d.Papule. The nurse just noted from the medical record that the patient has a lesion that is confluent in nature. On examination, the nurse expects to find: a.Lesions that run together. b.Annular lesions that have grown together. c.Lesions arranged in a line along a nerve route. d.Lesions that are grouped or clustered together. A patient has had a "terrible itch" for several months that he has been continuously scratching. On examination, the nurse might expect to find: a.A keloid. b.A fissure. c.Keratosis. d.Lichenification. A physician has diagnosed a patient with purpura. After leaving the room, a nursing student asks the nurse what the physician saw that led to that diagnosis. The nurse should say, "The physician is referring to the: a."Blue dilation of blood vessels in a star-shaped linear pattern on the legs." b."Fiery red, star-shaped marking on the cheek that has a solid circular center." c."Confluent and extensive patch of petechiae and ecchymoses on the feet." d."Tiny areas of hemorrhage that are less than 2 mm, round, discrete, and dark red in color." A mother has noticed that her son, who has been to a new babysitter, has some blisters and scabs on his face and buttocks. On examination, the nurse notices moist, thin-roofed vesicles with a thin erythematous base and suspects: a.Eczema. b.Impetigo. c.Herpes zoster. d.Diaper dermatitis. The nurse notices that a school-aged child has bluish-white, red-based spots in her mouth that are elevated approximately 1 to 3 mm. What other signs would the nurse expect to find in this patient? a.Pink, papular rash on the face and neck b.Pruritic vesicles over her trunk and neck c.Hyperpigmentation on the chest, abdomen, and back of the arms d.Red-purple, maculopapular, blotchy rash behind the ears and on the face The nurse is assessing the skin of a patient who has acquired immunodeficiency syndrome (AIDS) and notices multiple patchlike lesions on the temple and beard area that are faint pink in color. The nurse recognizes these lesions as: a.Measles (rubeola). b.Kaposi's sarcoma. c.Angiomas. d.Herpes zoster. A 45-year-old farmer comes in for a skin evaluation and complains of hair loss on his head. His hair seems to be breaking off in patches, and he notices some scaling on his head. The nurse begins the examination suspecting: a.Tinea capitis. b.Folliculitis. c.Toxic alopecia. d.Seborrheic dermatitis A mother brings her child into the clinic for an examination of the scalp and hair. She states that the child has developed irregularly shaped patches with broken-off, stublike hair in some places; she is worried that this condition could be some form of premature baldness. The nurse tells her that it is: a.Folliculitis that can be treated with an antibiotic. b.Traumatic alopecia that can be treated with antifungal medications. c.Tinea capitis that is highly contagious and needs immediate attention. d.Trichotillomania; her child probably has a habit of absentmindedly twirling her hair. The nurse has discovered decreased skin turgor in a patient and knows that this finding is expected in which condition? a.Severe obesity b.Childhood growth spurts c.Severe dehydration d.Connective tissue disorders such as scleroderma While performing an assessment of a 65-year-old man with a history of hypertension and coronary artery disease, the nurse notices the presence of bilateral pitting edema in the lower legs. The skin is puffy and tight but normal in color. No increased redness or tenderness is observed over his lower legs, and the peripheral pulses are equal and strong. In this situation, the nurse suspects that the likely cause of the edema is which condition? a.Heart failure b.Venous thrombosis c.Local inflammation d.Blockage of lymphatic drainage A 40-year-old woman reports a change in mole size, accompanied by color changes, itching, burning, and bleeding over the past month. She has a dark complexion and has no family history of skin cancer, but she has had many blistering sunburns in the past. The nurse would: a.Tell the patient to watch the lesion and report back in 2 months. b.Refer the patient because of the suggestion of melanoma on the basis of her symptoms. c.Ask additional questions regarding environmental irritants that may have caused this condition. d.Tell the patient that these signs suggest a compound nevus, which is very common in young to middle-aged adults. The ABCD danger signs of melanoma are asymmetry, border irregularity, color variation, and diameter. In addition, individuals may report a change in size, the development of itching, burning, and bleeding, or a new-pigmented lesion. Any one of these signs raises the suggestion of melanoma and warrants immediate referral. The nurse is assessing for clubbing of the fingernails and expects to find: a.Nail bases that are firm and slightly tender. b.Curved nails with a convex profile and ridges across the nails. c.Nail bases that feel spongy with an angle of the nail base of 150 degrees. d.Nail bases with an angle of 180 degrees or greater and nail bases that feel spongy. The nurse is assessing a patient who has liver disease for jaundice. Which of these assessment findings is indicative of true jaundice? a.Yellow patches in the outer sclera b.Yellow color of the sclera that extends up to the iris c.Skin that appears yellow when examined under low light d.Yellow deposits on the palms and soles of the feet where jaundice first appears The nurse is assessing for inflammation in a dark-skinned person. Which technique is the best? a.Assessing the skin for cyanosis and swelling b.Assessing the oral mucosa for generalized erythema c.Palpating the skin for edema and increased warmth d.Palpating for tenderness and local areas of ecchymosis A few days after a summer hiking trip, a 25-year-old man comes to the clinic with a rash. On examination, the nurse notes that the rash is red, macular, with a bull's eye pattern across his midriff and behind his knees. The nurse suspects: a.Rubeola. b.Lyme disease. c.Allergy to mosquito bites. d.Rocky Mountain spotted fever A 52-year-old woman has a papule on her nose that has rounded, pearly borders and a central red ulcer. She said she first noticed it several months ago and that it has slowly grown larger. The nurse suspects which condition? a.Acne b.Basal cell carcinoma c.Melanoma d.Squamous cell carcinoma A father brings in his 2-month-old infant to the clinic because the infant has had diarrhea for the last 24 hours. He says his baby has not been able to keep any formula down and that the diarrhea has been at least every 2 hours. The nurse suspects dehydration. The nurse should test skin mobility and turgor over the infant's: a.Sternum. b.Forehead. c.Forearms. d.Abdomen. A semiconscious woman is brought to the emergency department after she was found on the floor in her kitchen. Her face, nail beds, lips, and oral mucosa are a bright cherry-red color. The nurse suspects that this coloring is due to: a.Polycythemia. b.Carbon monoxide poisoning. c.Carotenemia. d.Uremia. A patient has been admitted for severe psoriasis. The nurse expects to see what finding in the patient's fingernails? a.Splinter hemorrhages b.Paronychia c.Pitting d.Beau lines The nurse is preparing for a certification course in skin care and needs to be familiar with the various lesions that may be identified on assessment of the skin. Which of the following definitions are correct? Select all that apply. a.Petechiae: Tiny punctate hemorrhages, 1 to 3 mm, round and discrete, dark red, purple, or brown in color b.Bulla: Elevated, circumscribed lesion filled with turbid fluid (pus) c.Papule: Hypertrophic scar d.Vesicle: Known as a friction blister e.Nodule: Solid, elevated, and hard or soft growth that is larger than 1 cm A patient has been admitted to a hospital after the staff in the nursing home noticed a pressure ulcer in his sacral area. The nurse examines the pressure ulcer and determines that it is a stage II ulcer. Which of these findings are characteristic of a stage II pressure ulcer? Select all that apply. a.Intact skin appears red but is not broken. b.Partial thickness skin erosion is observed with a loss of epidermis or dermis. c.Ulcer extends into the subcutaneous tissue. d.Localized redness in light skin will blanch with fingertip pressure. e.Open blister areas have a red-pink wound bed. f.Patches of eschar cover parts of the wound When performing a physical assessment, the first technique the nurse will always use is: a.Palpation. b.Inspection. c.Percussion. d.Auscultation. The nurse is preparing to perform a physical assessment. Which statement is true about the physical assessment? The inspection phase: a.Usually yields little information. b.Takes time and reveals a surprising amount of information. c.May be somewhat uncomfortable for the expert practitioner. d.Requires a quick glance at the patient's body systems before proceeding with palpation. The nurse is assessing a patient's skin during an office visit. What part of the hand and technique should be used to best assess the patient's skin temperature? a.Fingertips; they are more sensitive to small changes in temperature. b.Dorsal surface of the hand; the skin is thinner on this surface than on the palms. c.Ulnar portion of the hand; increased blood supply in this area enhances temperature sensitivity. d.Palmar surface of the hand; this surface is the most sensitive to temperature variations because of its increased nerve supply in this area. Which of these techniques uses the sense of touch to assess texture, temperature, moisture, and swelling when the nurse is assessing a patient? a.Palpation b.Inspection c.Percussion d.Auscultation The nurse is preparing to assess a patient's abdomen by palpation. How should the nurse proceed? a.Palpation of reportedly "tender" areas are avoided because palpation in these areas may cause pain. b.Palpating a tender area is quickly performed to avoid any discomfort that the patient may experience. c.The assessment begins with deep palpation, while encouraging the patient to relax and to take deep breaths. d.The assessment begins with light palpation to detect surface characteristics and to accustom the patient to being touched. The nurse would use bimanual palpation technique in which situation? a.Palpating the thorax of an infant b.Palpating the kidneys and uterus c.Assessing pulsations and vibrations d.Assessing the presence of tenderness and pain The nurse is preparing to percuss the abdomen of a patient. The purpose of the percussion is to assess the __________ of the underlying tissue. a.Turgor b.Texture c.Density d.Consistency The nurse is reviewing percussion techniques with a newly graduated nurse. Which technique, if used by the new nurse, indicates that more review is needed? a.Percussing once over each area b.Quickly lifting the striking finger after each stroke c.Striking with the fingertip, not the finger pad d.Using the wrist to make the strikes, not the arm When percussing over the liver of a patient, the nurse notices a dull sound. The nurse should: a.Consider this a normal finding. b.Palpate this area for an underlying mass. c.Reposition the hands, and attempt to percuss in this area again. d.Consider this finding as abnormal, and refer the patient for additional treatment. The nurse is unable to identify any changes in sound when percussing over the abdomen of an obese patient. What should the nurse do next? a.Ask the patient to take deep breaths to relax the abdominal musculature. b.Consider this finding as normal, and proceed with the abdominal assessment. c.Increase the amount of strength used when attempting to percuss over the abdomen. d.Decrease the amount of strength used when attempting to percuss over the abdomen. The nurse hears bilateral loud, long, and low tones when percussing over the lungs of a 4-year-old child. The nurse should: a.Palpate over the area for increased pain and tenderness. b.Ask the child to take shallow breaths, and percuss over the area again. c.Immediately refer the child because of an increased amount of air in the lungs. d.Consider this finding as normal for a child this age, and proceed with the examination A patient has suddenly developed shortness of breath and appears to be in significant respiratory distress. After calling the physician and placing the patient on oxygen, which of these actions is the best for the nurse to take when further assessing the patient? a.Count the patient's respirations. b.Bilaterally percuss the thorax, noting any differences in percussion tones. c.Call for a chest x-ray study, and wait for the results before beginning an assessment. d.Inspect the thorax for any new masses and bleeding associated with respirations. The nurse is teaching a class on basic assessment skills. Which of these statements is true regarding the stethoscope and its use? a.Slope of the earpieces should point posteriorly (toward the occiput). b.Although the stethoscope does not magnify sound, it does block out extraneous room noise. c.Fit and quality of the stethoscope are not as important as its ability to magnify sound. d.Ideal tubing length should be 22 inches to dampen the distortion of sound. The nurse is preparing to use a stethoscope for auscultation. Which statement is true regarding the diaphragm of the stethoscope? The diaphragm: a.Is used to listen for high-pitched sounds. b.Is used to listen for low-pitched sounds. c.Should be lightly held against the person's skin to block out low-pitched sounds. d.Should be lightly held against the person's skin to listen for extra heart sounds and murmurs. Before auscultating the abdomen for the presence of bowel sounds on a patient, the nurse should: a.Warm the endpiece of the stethoscope by placing it in warm water. b.Leave the gown on the patient to ensure that he or she does not get chilled during the examination. c.Ensure that the bell side of the stethoscope is turned to the "on" position. d.Check the temperature of the room, and offer blankets to the patient if he or she feels cold. The nurse will use which technique of assessment to determine the presence of crepitus, swelling, and pulsations? a.Palpation b.Inspection c.Percussion d.Auscultation The nurse is preparing to use an otoscope for an examination. Which statement is true regarding the otoscope? The otoscope: a.Is often used to direct light onto the sinuses. b.Uses a short, broad speculum to help visualize the ear. c.Is used to examine the structures of the internal ear. d.Directs light into the ear canal and onto the tympanic membrane. An examiner is using an ophthalmoscope to examine a patient's eyes. The patient has astigmatism and is nearsighted. The use of which of these techniques would indicate that the examination is being correctly performed? a.Using the large full circle of light when assessing pupils that are not dilated b.Rotating the lens selector dial to the black numbers to compensate for astigmatism c.Using the grid on the lens aperture dial to visualize the external structures of the eye d.Rotating the lens selector dial to bring the object into focus The nurse is unable to palpate the right radial pulse on a patient. The best action would be to: a.Auscultate over the area with a fetoscope. b.Use a goniometer to measure the pulsations. c.Use a Doppler device to check for pulsations over the area. d.Check for the presence of pulsations with a stethoscope. The nurse is preparing to perform a physical assessment. The correct action by the nurse is reflected by which statement? The nurse: a.Performs the examination from the left side of the bed. b.Examines tender or painful areas first to help relieve the patient's anxiety. c.Follows the same examination sequence, regardless of the patient's age or condition. d.Organizes the assessment to ensure that the patient does not change positions too often. A man is at the clinic for a physical examination. He states that he is "very anxious" about the physical examination. What steps can the nurse take to make him more comfortable? a.Appear unhurried and confident when examining him. b.Stay in the room when he undresses in case he needs assistance. c.Ask him to change into an examining gown and to take off his undergarments. d.Defer measuring vital signs until the end of the examination, which allows him time to become comfortable. When performing a physical examination, safety must be considered to protect the examiner and the patient against the spread of infection. Which of these statements describes the most appropriate action the nurse should take when performing a physical examination? a.Washing one's hands after removing gloves is not necessary, as long as the gloves are still intact. b.Hands are washed before and after every physical patient encounter. c.Hands are washed before the examination of each body system to prevent the spread of bacteria from one part of the body to another. d.Gloves are worn throughout the entire examination to demonstrate to the patient concern regarding the spread of infectious diseases. The nurse is examining a patient's lower leg and notices a draining ulceration. Which of these actions is most appropriate in this situation? a.Washing hands, and contacting the physician b.Continuing to examine the ulceration, and then washing hands c.Washing hands, putting on gloves, and continuing with the examination of the ulceration d.Washing hands, proceeding with rest of the physical examination, and then continuing with the examination of the leg ulceration During the examination, offering some brief teaching about the patient's body or the examiner's findings is often appropriate. Which one of these statements by the nurse is most appropriate? a."Your atrial dysrhythmias are under control." b."You have pitting edema and mild varicosities." c."Your pulse is 80 beats per minute, which is within the normal range." d."I'm using my stethoscope to listen for any crackles, wheezes, or rubs." - The nurse keeps in mind that the most important reason to share information and to offer brief teaching while performing the physical examination is to help the: a.Examiner feel more comfortable and to gain control of the situation. b.Examiner to build rapport and to increase the patient's confidence in him or her. c.Patient understand his or her disease process and treatment modalities. d.Patient identify questions about his or her disease and the potential areas of patient education. The nurse is examining an infant and prepares to elicit the Moro reflex at which time during the examination? a.When the infant is sleeping b.At the end of the examination c.Before auscultation of the thorax d.Halfway through the examination When preparing to perform a physical examination on an infant, the nurse should: a.Have the parent remove all clothing except the diaper on a boy. b.Instruct the parent to feed the infant immediately before the examination. c.Encourage the infant to suck on a pacifier during the abdominal examination. d.Ask the parent to leave the room briefly when assessing the infant's vital signs. A 6-month-old infant has been brought to the well-child clinic for a check-up. She is currently sleeping. What should the nurse do first when beginning the examination? a.Auscultate the lungs and heart while the infant is still sleeping. b.Examine the infant's hips, because this procedure is uncomfortable. c.Begin with the assessment of the eye, and continue with the remainder of the examination in a head-to-toe approach. d.Wake the infant before beginning any portion of the examination to obtain the most accurate assessment of body systems. A 2-year-old child has been brought to the clinic for a well-child checkup. The best way for the nurse to begin the assessment is to: a.Ask the parent to place the child on the examining table. b.Have the parent remove all of the child's clothing before the examination. c.Allow the child to keep a security object such as a toy or blanket during the examination. d.Initially focus the interactions on the child, essentially ignoring the parent until the child's trust has been obtained. The nurse is examining a 2-year-old child and asks, "May I listen to your heart now?" Which critique of the nurse's technique is most accurate? a.Asking questions enhances the child's autonomy b.Asking the child for permission helps develop a sense of trust c.This question is an appropriate statement because children at this age like to have choices d.Children at this age like to say, "No." The examiner should not offer a choice when no choice is available With which of these patients would it be most appropriate for the nurse to use games during the assessment, such as having the patient "blow out" the light on the penlight? a.Infant b.Preschool child c.School-age child d.Adolescent The nurse is preparing to examine a 4-year-old child. Which action is appropriate for this age group? a.Explain the procedures in detail to alleviate the child's anxiety. b.Give the child feedback and reassurance during the examination. c.Do not ask the child to remove his or her clothes because children at this age are usually very private. d.Perform an examination of the ear, nose, and throat first, and then examine the thorax and abdomen. When examining a 16-year-old male teenager, the nurse should: a.Discuss health teaching with the parent because the teen is unlikely to be interested in promoting wellness. b.Ask his parent to stay in the room during the history and physical examination to answer any questions and to alleviate his anxiety. c.Talk to him the same manner as one would talk to a younger child because a teen's level of understanding may not match his or her speech. d.Provide feedback that his body is developing normally, and discuss the wide variation among teenagers on the rate of growth and development. When examining an older adult, the nurse should use which technique? a.Avoid touching the patient too much. b.Attempt to perform the entire physical examination during one visit. c.Speak loudly and slowly because most aging adults have hearing deficits. d.Arrange the sequence of the examination to allow as few position changes as possible. The most important step that the nurse can take to prevent the transmission of microorganisms in the hospital setting is to: a.Wear protective eye wear at all times. b.Wear gloves during any and all contact with patients. c.Wash hands before and after contact with each patient. d.Clean the stethoscope with an alcohol swab between patients. Which of these statements is true regarding the use of Standard Precautions in the health care setting? a.Standard Precautions apply to all body fluids, including sweat. b.Use alcohol-based hand rub if hands are visibly dirty. c.Standard Precautions are intended for use with all patients, regardless of their risk or presumed infection status. d.Standard Precautions are to be used only when nonintact skin, excretions containing visible blood, or expected contact with mucous membranes is present. The nurse is preparing to assess a hospitalized patient who is experiencing significant shortness of breath. How should the nurse proceed with the assessment? a.The patient should lie down to obtain an accurate cardiac, respiratory, and abdominal assessment. b.A thorough history and physical assessment information should be obtained from the patient's family member. c.A complete history and physical assessment should be immediately performed to obtain baseline information. d.Body areas appropriate to the problem should be examined and then the assessment completed after the problem has resolved. When examining an infant, the nurse should examine which area first? a.Ear b.Nose c.Throat d.Abdomen While auscultating heart sounds, the nurse hears a murmur. Which of these instruments should be used to assess this murmur? a.Electrocardiogram b.Bell of the stethoscope c.Diaphragm of the stethoscope d.Palpation with the nurse's palm of the hand During an examination of a patient's abdomen, the nurse notes that the abdomen is rounded and firm to the touch. During percussion, the nurse notes a drumlike quality of the sounds across the quadrants. This type of sound indicates: a.Constipation. b.Air-filled areas. c.Presence of a tumor. d.Presence of dense organs. . The nurse is preparing to examine a 6-year-old child. Which action is most appropriate? a.The thorax, abdomen, and genitalia are examined before the head. b.Talking about the equipment being used is avoided because doing so may increase the child's anxiety. c.The nurse should keep in mind that a child at this age will have a sense of modesty. d.The child is asked to undress from the waist up. . During auscultation of a patient's heart sounds, the nurse hears an unfamiliar sound. The nurse should: a.Document the findings in the patient's record. b.Wait 10 minutes, and auscultate the sound again. c.Ask the patient how he or she is feeling. d.Ask another nurse to double check the finding. The nurse is preparing to palpate the thorax and abdomen of a patient. Which of these statements describes the correct technique for this procedure? Select all that apply. a.Warm the hands first before touching the patient. b.For deep palpation, use one long continuous palpation when assessing the liver. c.Start with light palpation to detect surface characteristics. d.Use the fingertips to examine skin texture, swelling, pulsation, and presence of lumps. e.Identify any tender areas, and palpate them last. f.Use the palms of the hands to assess temperature of the skin. The primary purpose of the ciliated mucous membrane in the nose is to: 1. warm the inhaled air. 2. filter out dust and bacteria. 3. filter coarse particles from inhaled air. 4. facilitate movement of air through the nares. The projections in the nasal cavity that increase the surface area are called the: 1. meatus. 2. septum. 3. turbinates. 4. Kiesselbach's plexus. Which of the following is true in relation to a newborn infant? 1. The sphenoid sinuses are full size at birth. 2. The maxillary sinuses reach full size after puberty. 3. The frontal sinuses are fairly well developed at birth. 4. The maxillary and ethmoid sinuses are the only ones present at birth. The tissue that connects the tongue to the floor of the mouth is the: 1. uvula. 2. palate. 3. papillae. 4. frenulum. The salivary gland that is located in the cheek in front of the ear is the: 1. parotid gland. 2. Stenson's gland. 3. sublingual gland. 4. submandibular gland. In assessing the tonsils of a 30-year-old, the nurse notes that they are involuted, granular in appearance, and appear to have deep crypts. What is correct response to these findings? 1. Refer the patient to a throat specialist. 2. Nothing, this is the appearance of normal tonsils. 3. Continue with assessment looking for any other abnormal findings. 4. Obtain a throat culture on the patient for possible strep infection. The nurse is obtaining a history on a 3-month-old infant. During the interview, the mom states, "I think she is getting her first tooth because she has started drooling a lot." The nurse's best response would be 1. "You're right, drooling is usually a sign of the first tooth." 2. "It would be unusual for a 3-month-old to be getting her first tooth." 3. "This could be the sign of a problem with the salivary glands." 4. "She is just starting to salivate and hasn't learned to swallow the saliva." During an assessment of an 80-year-old patient, the nurse would expect to find: 1. hypertrophy of the gums. 2. an increased production of saliva. 3. a decreased ability to identify odors. 4. finer and less prominent nasal hair. The nurse is doing an oral assessment on a 40-year-old black patient and notes the presence of a 1-cm, nontender, grayish-white lesion on the left buccal mucosa. Which of the following is true concerning this lesion? 1. This lesion is leukoedema and is common in blacks. 2. This is the result of hyperpigmentation and is normal. 3. This is torus palatinus and would normally only be found in smokers. 4. This type of lesion is indicative of cancer and should be tested immediately. During the history, a patient tells the nurse that he has frequent nosebleeds and asks the best way to get them to stop. What would be the nurse's best response? 1. "While sitting up, place a cold compress over your nose." 2. "Sit up with your head tilted forward and pinch your nose." 3. "Just let the bleeding stop on its own, but don't blow your nose." 4. "Lie on your back with your head tilted back and pinch your nose." A 92-year-old patient has had a stroke. The right side of his face is drooping. The nurse might also suspect which of the following? 1. Epistaxis 2. Agenesis 3. Dysphagia 4. Xerostomia While obtaining a history from the mother of a 1-year-old, the nurse notices that the baby has had a bottle in his mouth the entire time. The mother states, "it makes a great pacifier." The best response by the nurse would be: 1. "You're right, bottles make very good pacifiers." 2. "Use of a bottle is better for the teeth than thumb sucking." 3. "It's okay to do this as long as the bottle contains milk and not juice." 4. "Prolonged use of a bottle can increase the risk for tooth decay and ear infections." A 72-year-old patient has a history of hypertension and chronic lung disease. An important question for the nurse to include in the history would be: 1. "Do you use a fluoride supplement?" 2. "Have you had tonsillitis in the last year?" 3. "At what age did you get your first tooth?" 4. "Have you noticed any dryness in your mouth?" When using an otoscope to assess the nasal cavity, which of the following would the nurse need to do? 1. Insert the speculum at least 3 cm into the vestibule. 2. Avoid touching the nasal septum with the speculum. 3. Gently displace the nose to the side that is being examined. 4. Keep the speculum tip medial to avoid touching the floor of the nares. The nurse is doing an assessment on a 21-year-old patient and notes that his nasal mucosa appears pale, gray, and swollen. What would be the most appropriate question to ask the patient? 1. "Are you aware of having any allergies?" 2. "Do you have an elevated temperature?" 3. "Have you had any symptoms of a cold?" 4. "Have you been having frequent nosebleeds?" The nurse is palpating the sinus areas. If the findings are normal, the patient would report which sensation? 1. No sensation 2. Firm pressure 3. Pain during palpation 4. Pain sensation behind eyes During an oral assessment of a 30-year-old black patient, the nurse notes bluish lips and a dark line along the gingival margin. What would the nurse do in response to this finding? 1. Check the patient's hemoglobin for anemia. 2. Assess for other signs of insufficient oxygen supply. 3. Proceed with assessment, knowing that this is a normal finding. 4. Ask if he has been exposed to an excessive amount of carbon monoxide. During an assessment of a 20-year-old patient with a 3-day history of nausea and vomiting, the nurse notes the following: dry mucosa and deep fissures in the tongue. This finding is reflective of: 1. dehydration. 2. irritation by gastric juices. 3. a normal oral assessment. 4. side effects from nausea medication. A 32-year-old woman is at the clinic for "little white bumps in my mouth." During the assessment, the nurse notes that she has a 0.5 cm white, nontender papule under her tongue and one on the mucosa of her right cheek. What would the nurse tell the patient? 1. "These spots are seen with infections such as strep throat." 2. "These could be indicative of a serious lesion, so I will refer you to a specialist." 3. "This is called leukoplakia and can be caused by chronic irritation such as smoking." 4. "These bumps are Fordyce's granules, which are sebaceous cysts and not a serious condition." Which of the following best describes the test the nurse should use to assess the function of cranial nerve X? 1. Observe the patient's ability to articulate specific words. 2. Assess movement of the hard palate and uvula with the gag reflex. 3. Have the patient stick out the tongue and observe for tremors or pulling to one side. 4. Ask the patient to say "ahhh" and watch for movement of the soft palate and uvula. A 10-year-old is at the clinic for "a sore throat lasting 6 days." The nurse is aware that which of the following would be consistent with an acute infection? 1. Tonsils 1+/1-4+ and pink 2. Tonsils 2+/1-4+ with small plugs of white debris 3. Tonsils 3+/1-4+ with large white spots 4. Tonsils 3+/1-4+ with yellowish exudate Immediately after birth, the nurse is unable to suction the nares of a newborn. An attempt is made to pass a catheter through both nasal cavities with no success. What would be the nurse's best response? 1. Attempt to suction again with a bulb syringe. 2. Wait a few minutes and try again once the infant stops crying. 3. Recognize this is a situation that requires immediate intervention. 4. Contact the physician and request assistance when he gets a chance. The nurse notices that the mother of a 2-year-old boy brings him into the clinic quite frequently for various injuries and suspects there may be some child abuse involved. In doing the inspection of his mouth, the nurse should inspect for: 1. swollen, red tonsils. 2. ulcerations on the hard palate. 3. bruising on the buccal mucosa or gums. 4. small yellow papules along the hard palate. The nurse is assessing a 3-year-old who is here for "drainage from the nose." On assessment, it is found that there is a purulent drainage from the left nares that has a very foul odor and no drainage from the right nares. The child is afebrile with no other symptoms. What should the nurse do next? 1. Refer to the physician for an antibiotic order. 2. Have the mother bring the child back in 1 week. 3. Perform an otoscopic examination of the left nares. 4. Tell the mother that this is normal for children of this age. During an assessment of a 26-year-old at the clinic for "a spot on my lip I think is cancer" the nurse notes the following findings: a group of clear vesicles with an erythematous base around them located at the lip-skin border. The patient mentions that she just returned from Hawaii. What would be the most appropriate response by the nurse? 1. Tell the patient she will need to see a skin specialist. 2. Discuss the benefits of having a biopsy done of any unusual lesion. 3. Tell the patient this is herpes simplex I and will heal in 4 to 10 days. 4. Tell the patient that this is most likely the result of a riboflavin deficiency and discuss nutrition. While performing an assessment of the mouth, the nurse notices that the patient has a 1-cm ulceration that is crusted with an elevated border. It is located on the outer third of the lower lip. What other information would be most important for the nurse to assess? 1. Nutritional status 2. When the patient first noticed the lesion 3. Whether the patient has had a recent cold 4. Whether the patient has had any recent exposure to sick animals A pregnant woman states that she is concerned about her gums because she has noticed they are swollen and have started bleeding. What would be an appropriate response by the nurse? 1. "This is probably due to a vitamin C deficiency." 2. "I'm not sure what causes it but let me know if it's not better in a few weeks." 3. "You need to make an appointment with your dentist as soon as possible to have this checked." 4. "This can be caused by the change in hormone balance in your system when you're pregnant." A 40-year-old patient who has just finished chemotherapy for breast cancer tells the nurse that she is concerned about her mouth. During the assessment the nurse finds the following: areas of buccal mucosa that are raw and red with some bleeding as well as other areas that have a white, cheesy coating. The nurse recognizes that this abnormality is: 1. carcinoma. 2. candidiasis. 3. leukoplakia. 4. Koplik's spots. The nurse is assessing a patient in the hospital who has received numerous antibiotics and notes that his tongue appears to be black and hairy. In response to his concern, what would the nurse say? 1. "We need to get a biopsy and see what the cause is." 2. "This is an overgrowth of hair and will go away in a few days." 3. "This is a fungal infection caused by all the antibiotics you've received." 4. "This is probably caused by the same bacteria you had in your lungs." The nurse is assessing a patient with a history of intravenous drug abuse. In assessing his mouth, the nurse notices a dark red confluent macule on the hard palate. This could be an early sign of: 1. AIDS. 2. measles. 3. leukemia. 4. carcinoma. A mother brings her 4-month-old to the clinic with concerns regarding a small pad in the middle of the upper lip that has been there since 1 month of age. The infant has no health problems. On physical examination, the nurse notes a 0.5-cm, fleshy, elevated area in the middle of the upper lip. There is no evidence of inflammation or drainage. What would the nurse tell this mother? 1. "This is an area of irritation caused from teething and is nothing to worry about." 2. "This is an abnormal finding and should be evaluated by another health care provider." 3. "This is the result of chronic drooling and should resolve within the next month or two." 4. "This is a sucking tubercle caused from the friction of breast- or bottle-feeding and is normal." A mother is concerned that her 18-month-old has 12 teeth. She is wondering if this is normal for a child of this age. The nurse's best response would be: 1. "How many teeth did you have at this age?" 2. "All 20 deciduous teeth are expected to erupt by age 4 years." 3. "This is a normal number of teeth for an 18-month-old." 4. "Normally, by age 2 1/2, 16 deciduous teeth are expected." When examining an elderly patient, the nurse recognizes that which finding is due to the aging process? 1. Teeth that appear shorter 2. A tongue that looks smoother in appearance 3. Buccal mucosa that is beefy red in appearance 4. A small, painless lump on the dorsum of the tongue When examining the nares of a 45-year-old patient who has complaints of rhinorrhea, itching of the nose and eyes, and sneezing, the nurse notes the following: pale turbinates, swelling of the turbinates, and clear rhinorrhea. Which of the following is most likely the cause? 1. Nasal polyps 2. Acute sinusitis 3. Allergic rhinitis 4. Nasal carcinoma When assessing the tongue of an adult, the nurse knows that an abnormal finding would be: 1. a smooth glossy dorsal surface. 2. a thin white coating over the tongue. 3. raised papillae on the dorsal surface. 4. visible venous patterns on the ventral surface. When assessing the tongue of an adult, the nurse knows that an abnormal finding would be: 1. A painful vesicle inside the cheek for 2 days 2. The presence of moist, nontender Stenson's ducts 3. Stippled gingival margins that adhere snugly to the teeth 4. An ulceration on the side of the tongue with rolled edges. A patient has been diagnosed with strep throat. The nurse is aware that without treatment which complication may occur? 1. Rubella 2. Leukoplakia 3. Rheumatic fever 4. Scarlet fever During a check-up, a 22-year-old woman tells the nurse that she uses an over-the-counter nasal spray because of her allergies. She also notes that it does not work as well as it used to when she started using it. The best response by the nurse would be: 1. "You should never use over-the-counter nasal sprays because of the risk of addiction." 2. "You should try switching to another brand of medication to prevent this problem." 3. "It is important to keep using this spray to keep your allergies under control." 4. "Using these nasal medications irritates the lining of the nose and may cause rebound swelling." During an oral examination of a 4-year-old American Indian child, the nurse notices that her uvula is partially split. Which of the following statements is accurate? 1. This is a cleft palate and is common in American Indians. 2. This is a bifid uvula, which occurs in some American Indian groups. 3. This is due to an injury and should be reported to the authorities. 4. This is torus palatinus, which occurs frequently in American Indians. A patient comes into the clinic complaining of facial pain, fever, and malaise. On examination, the nurse notes swollen turbinates and purulent discharge from the nose. The patient also complains of a dull, throbbing pain in his cheeks and teeth on the right side and pain when the nurse palpates the areas. The nurse recognizes that this patient has: 1. posterior epistaxis. 2. frontal sinusitis. 3. maxillary sinusitis. 4. nasal polyps. A woman who is in the second trimester of pregnancy mentions that she has had "more nosebleeds than ever" since she became pregnant. The nurse recognizes that this is due to: 1. a problem with the patient's coagulation system. 2. increased vascularity in the upper respiratory tract as a result of the pregnancy. 3. increased susceptibility to colds and nasal irritation. 4. inappropriate use of nasal sprays. 1. The nurse is teaching a health class to high-school-age boys. When discussing the topic of the use of smokeless tobacco (SLT), which of the following statements are accurate? (Select all that apply. 1. One pinch of SLT in the mouth for 30 minutes delivers the equivalent of one cigarette. 2. The use of SLT has been associated with a greater risk of oral cancer than smoking has. 3. Pain is an early sign of oral cancer. 4. Pain is rarely an early sign of oral cancer. 5. Tooth decay is another risk of SLT because of the use of sugar as a sweetener. 6. SLT is considered a healthy alternative to smoking. The sac that surrounds and protects the heart is called the: a. Pericardium. b. Myocardium. c. Endocardium. d. Pleural space. The direction of blood flow through the heart is best described by which of these? a. Vena cava right atrium right ventricle lungs pulmonary artery left atrium left ventricle b. Right atrium right ventricle pulmonary artery lungs pulmonary vein left atrium left ventricle c. Aorta right atrium right ventricle lungs pulmonary vein left atrium left ventricle vena cava d. Right atrium right ventricle pulmonary vein lungs pulmonary artery left atrium left ventricle - The nurse is reviewing anatomy and physiology of the heart. Which statement best describes what is meant by atrial kick? a. The atria contract during systole and attempt to push against closed valves. b. Contraction of the atria at the beginning of diastole can be felt as a palpitation. c. Atrial kick is the pressure exerted against the atria as the ventricles contract during systole. d. The atria contract toward the end of diastole and push the remaining blood into the ventricles. When listening to heart sounds, the nurse knows that the valve closures that can be heard best at the base of the heart are: a. Mitral and tricuspid. b. Tricuspid and aortic. c. Aortic and pulmonic. d. Mitral and pulmonic. Which of these statements describes the closure of the valves in a normal cardiac cycle? a. The aortic valve closes slightly before the tricuspid valve. b. The pulmonic valve closes slightly before the aortic valve. c. The tricuspid valve closes slightly later than the mitral valve. d. Both the tricuspid and pulmonic valves close at the same time. The component of the conduction system referred to as the pacemaker of the heart is the: a. Atrioventricular (AV) node. b. Sinoatrial (SA) node. c. Bundle of His. d. Bundle branches. The electrical stimulus of the cardiac cycle follows which sequence? a. AV node SA node bundle of His b. Bundle of His AV node SA node c. SA node AV node bundle of His bundle branches d. AV node SA node bundle of His bundle branches The findings from an assessment of a 70-year-old patient with swelling in his ankles include jugular venous pulsations 5 cm above the sternal angle when the head of his bed is elevated 45 degrees. The nurse knows that this finding indicates: a. Decreased fluid volume. b. Increased cardiac output. c. Narrowing of jugular veins. d. Elevated pressure related to heart failure. When assessing a newborn infant who is 5 minutes old, the nurse knows that which of these statements would be true? a. The left ventricle is larger and weighs more than the right ventricle. b. The circulation of a newborn is identical to that of an adult. c. Blood can flow into the left side of the heart through an opening in the atrial septum. d. The foramen ovale closes just minutes before birth, and the ductus arteriosus closes immediately after. A 25-year-old woman in her fifth month of pregnancy has a blood pressure of 100/70 mm Hg. In reviewing her previous exam, the nurse notes that her blood pressure in her second month was 124/80 mm Hg. In evaluating this change, what does the nurse know to be true? a. This decline in blood pressure is the result of peripheral vasodilatation and is an expected change. b. Because of increased cardiac output, the blood pressure should be higher at this time. c. This change in blood pressure is not an expected finding because it means a decrease in cardiac output. d. This decline in blood pressure means a decrease in circulating blood volume, which is dangerous for the fetus. In assessing a 70-year-old man, the nurse finds the following: blood pressure 140/100 mm Hg; heart rate 104 and slightly irregular; split S2. Which of these findings can be explained by expected hemodynamic changes related to age? a. Increase in resting heart rate b. Increase in systolic blood pressure c. Decrease in diastolic blood pressure d. Increase in diastolic blood pressure A 45-year-old man is in the clinic for a routine physical. During the history the patient states he's been having difficulty sleeping. "I'll be sleeping great and then I wake up and feel like I can't get my breath." The nurse's best response to this would be: a. "When was your last electrocardiogram?" b. "It's probably because it's been so hot at night." c. "Do you have any history of problems with your heart?" d. "Have you had a recent sinus infection or upper respiratory infection?" Paroxysmal nocturnal dyspnea occurs with heart failure. Lying down increases volume of intrathoracic blood, and the weakened heart cannot accommodate the increased load. Classically, the person awakens after 2 hours of sleep, arises, and flings open a window with the perception of needing fresh air. In assessing a patient's major risk factors for heart disease, which would the nurse want to include when taking a history? a. Family history, hypertension, stress, and age b. Personality type, high cholesterol, diabetes, and smoking c. Smoking, hypertension, obesity, diabetes, and high cholesterol d. Alcohol consumption, obesity, diabetes, stress, and high cholesterol The mother of a 3-month-old infant states that her baby has not been gaining weight. With further questioning, the nurse finds that the infant falls asleep after nursing and wakes up after a short amount of time, hungry again. What other information would the nurse want to have? a. Infant's sleeping position b. Sibling history of eating disorders c. Amount of background noise when eating d. Presence of dyspnea or diaphoresis when sucking To screen for heart disease in an infant, focus on feeding. Note fatigue during feeding. An infant with heart failure takes fewer ounces each feeding, becomes dyspneic with sucking, may be diaphoretic and then falls into exhausted sleep and awakens after a short time hungry again. In assessing the carotid arteries of an older patient with cardiovascular disease, the nurse would: a. Palpate the artery in the upper one third of the neck. b. Listen with the bell of the stethoscope to assess for bruits. c. Simultaneously palpate both arteries to compare amplitude. d. Instruct the patient to take slow deep breaths during auscultation. During an assessment of a 68-year-old man with a recent onset of right-sided weakness, the nurse hears a blowing, swishing sound with the bell of the stethoscope over the left carotid artery. This finding would indicate: a. Valvular disorder. b. Blood flow turbulence. c. Fluid volume overload. d. Ventricular hypertrophy. During inspection of the precordium of an adult patient, the nurse notices the chest moving in a forceful manner along the sternal border. This finding most likely suggests: a. Normal heart. b. Systolic murmur. c. Enlargement of the left ventricle. d. Enlargement of the right ventricle. During an assessment of a healthy adult, where would the nurse expect to palpate the apical impulse? a. Third left intercostal space at the midclavicular line b. Fourth left intercostal space at the sternal border c. Fourth left intercostal space at the anterior axillary line d. Fifth left intercostal space at the midclavicular line The nurse is examining a patient who has possible cardiac enlargement. Which statement about percussion of the heart is true? a. Percussion is a useful tool for outlining the heart's borders. b. Percussion is easier in patients who are obese. c. Studies show that percussed cardiac borders do not correlate well with the true cardiac border. d. Only expert health care providers should attempt percussion of the heart. - The nurse is preparing to auscultate for heart sounds. Which technique is correct? a. Listening to the sounds at the aortic, tricuspid, pulmonic, and mitral areas b. Listening by inching the stethoscope in a rough Z pattern, from the base of the heart across and down, then over to the apex c. Listening to the sounds only at the site where the apical pulse is felt to be the strongest d. Listening for all possible sounds at a time at each specified area While counting the apical pulse on a 16-year-old patient, the nurse notices an irregular rhythm. His rate speeds up on inspiration and slows on expiration. What would be the nurse's response? a. Talk with the patient about his intake of caffeine. b. Perform an electrocardiogram after the examination. c. No further response is needed because sinus arrhythmia can occur normally. d. Refer the patient to a cardiologist for further testing. The rhythm should be regular, although sinus arrhythmia occurs normally in young adults and children. With sinus arrhythmia, the rhythm varies with the person's breathing, increasing at the peak of inspiration, and slowing with expiration. When listening to heart sounds, the nurse knows that S1: a. Is louder than the S2 at the base of the heart. b. Indicates the beginning of diastole. c. Coincides with the carotid artery pulse. d. Is caused by the closure of the semilunar valves. During the cardiac auscultation the nurse hears a sound occurring immediately after S2 at the second left intercostal space. To further assess this sound, what should the nurse do? a. Have the patient turn to the left side while the nurse listens with the bell of the stethoscope. b. Ask the patient to hold his or her breath while the nurse listens again. c. No further assessment is needed because the nurse knows this sound is an S3. d. Watch the patient's respirations while listening for the effect on the sound. Which of these findings would the nurse expect to notice during a cardiac assessment on a 4-year-old child? a. S3 when sitting up b. Persistent tachycardia above 150 beats per minute c. Murmur at the second left intercostal space when supine d. Palpable apical impulse in the fifth left intercostal space lateral to midclavicular line While auscultating heart sounds on a 7-year-old child for a routine physical, the nurse hears an S3, a soft murmur at left midsternal border, and a venous hum when the child is standing. Which of these would be a correct interpretation of these findings? a. S3 is indicative of heart disease in children. b. These findings can all be normal in a child. c. These findings are indicative of congenital problems. d. The venous hum most likely indicates an aneurysm. During the precordial assessment on an patient who is 8 months pregnant, the nurse palpates the apical impulse at the fourth left intercostal space lateral to the midclavicular line. This finding would indicate: a. Right ventricular hypertrophy. b. Increased volume and size of the heart as a result of pregnancy. c. Displacement of the heart from elevation of the diaphragm. d. Increased blood flow through the internal mammary artery. In assessing for an S4 heart sound with a stethoscope, the nurse would listen with the: a. Bell of the stethoscope at the base with the patient leaning forward. b. Bell of the stethoscope at the apex with the patient in the left lateral position. c. Diaphragm of the stethoscope in the aortic area with the patient sitting. d. Diaphragm of the stethoscope in the pulmonic area with the patient supine. A 70-year-old patient with a history of hypertension has a blood pressure of 180/100 mm Hg and a heart rate of 90 beats per minute. The nurse hears an extra heart sound at the apex immediately before S1. The sound is heard only with the bell while the patient is in the left lateral position. With these findings and the patient's history, the nurse knows that this extra heart sound is most likely a(n): a. Split S1. b. Atrial gallop. c. Diastolic murmur. d. Summation sound. The nurse is performing a cardiac assessment on a 65-year-old patient 3 days after her myocardial infarction. Heart sounds are normal when she is supine, but when she is sitting and leaning forward, the nurse hears a high-pitched, scratchy sound with the diaphragm of the stethoscope at the apex. It disappears on inspiration. The nurse suspects: a. Increased cardiac output. b. Another MI. c. Inflammation of the precordium. d. Ventricular hypertrophy resulting from muscle damage. The mother of a 10-month-old infant tells the nurse that she has noticed that her son becomes blue when he is crying and that the frequency of this is increasing. He is also not crawling yet. During the examination the nurse palpates a thrill at the left lower sternal border and auscultates a loud systolic murmur in the same area. What would be the most likely cause of these findings? a. Tetralogy of Fallot b. Atrial septal defect c. Patent ductus arteriosus d. Ventricular septal defect A 30-year-old woman with a history of mitral valve problems states that she has been "very tired." She has started waking up at night and feels like her "heart is pounding." During the assessment, the nurse palpates a thrill and lift at the fifth left intercostal space midclavicular line. In the same area the nurse also auscultates a blowing, swishing sound right after S1. These findings would be most consistent with: a. Heart failure. b. Aortic stenosis. c. Pulmonary edema. d. Mitral regurgitation. During a cardiac assessment on a 38 year-old patient in the hospital for "chest pain," the nurse finds the following: jugular vein pulsations 4 cm above sternal angle when he is elevated at 45 degrees, blood pressure 98/60 mm Hg, heart rate 130 beats per minute, ankle edema, difficulty in breathing when supine, and an S3 on auscultation. Which of these conditions best explains the cause of these findings? a. Fluid overload b. Atrial septal defect c. MI d. Heart failure The nurse knows that normal splitting of the second heart sound is associated with: a. Expiration. b. Inspiration. c. Exercise state. d. Low resting heart rate. During a cardiovascular assessment, the nurse knows that a "thrill" is: a. Vibration that is palpable. b. Palpated in the right epigastric area. c. Associated with ventricular hypertrophy. d. Murmur auscultated at the third intercostal space. During a cardiovascular assessment, the nurse knows that an S4 heart sound is: a. Heard at the onset of atrial diastole. b. Usually a normal finding in the older adult. c. Heard at the end of ventricular diastole. d. Heard best over the second left intercostal space with the individual sitting upright. During an assessment, the nurse notes that the patient's apical impulse is displaced laterally, and it is palpable over a wide area. This indicates: a. Systemic hypertension. b. Pulmonic hypertension. c. Pressure overload, as in aortic stenosis. d. Volume overload, as in heart failure. When the nurse is auscultating the carotid artery for bruits, which of these statements reflects correct technique? a. While listening with the bell of the stethoscope, the patient is asked to take a deep breath and hold it. b. While auscultating one side with the bell of the stethoscope, the carotid artery is palpated on the other side to check pulsations. c. While lightly applying the bell of the stethoscope over the carotid artery and listening, the patient is asked to take a breath, exhale, and briefly hold it. d. While firmly placing the bell of the stethoscope over the carotid artery and listening, the patient is asked to take a breath, exhale, and briefly hold it. The nurse is preparing for a class on risk factors for hypertension, and reviews recent statistics. Which racial group has the highest prevalence of hypertension in the world? a. Blacks b. Whites c. American Indians d. Hispanics - The nurse is assessing a patient with possible cardiomyopathy and assesses the hepatojugular reflux. If heart failure is present, then the nurse should see which finding while pushing on the right upper quadrant of the patient's abdomen, just below the rib cage? a. The jugular veins will rise for a few seconds and then recede back to the previous level if the heart is properly working. b. The jugular veins will remain elevated as long as pressure on the abdomen is maintained. c. An impulse will be visible at the fourth or fifth intercostal space at or inside the midclavicular line. d. The jugular veins will not be detected during this maneuver. The nurse is assessing the apical pulse of a 3-month-old infant and finds that the heart rate is 135 beats per minute. The nurse interprets this result as: a. Normal for this age. b. Lower than expected. c. Higher than expected, probably as a result of crying. d. Higher than expected, reflecting persistent tachycardia. The nurse is presenting a class on risk factors for cardiovascular disease. Which of these are considered modifiable risk factors for myocardial infarction (MI)? Select all that apply. a. Ethnicity b. Abnormal lipids c. Smoking d. Gender e. Hypertension f. Diabetes g. Family history Nine modifiable risk factors for MI, as identified by a recent study, include abnormal lipids, smoking, hypertension, diabetes, abdominal obesity, psychosocial factors, consumption of fruits and vegetables, alcohol use, and regular physical activity. The nurse is assessing a patient's pulses and notices a difference between the patient's apical pulse and radial pulse. The apical pulse was 118 beats per minute, and the radial pulse was 105 beats per minute. What is the pulse deficit? Which of these statements is true regarding the arterial system? a. Arteries are large-diameter vessels. b. The arterial system is a high-pressure system. c. The walls of arteries are thinner than those of the veins. d. Arteries can greatly expand to accommodate a large blood volume increase. The nurse is reviewing the blood supply to the arm. The major artery supplying the arm is the _____ artery. a. Ulnar b. Radial c. Brachial d. Deep palmar The nurse is preparing to assess the dorsalis pedis artery. Where is the correct location for palpation? a. Behind the knee b. Over the lateral malleolus c. In the groove behind the medial malleolus d. Lateral to the extensor tendon of the great toe A 65-year-old patient is experiencing pain in his left calf when he exercises that disappears after resting for a few minutes. The nurse recognizes that this description is most consistent with _________ the left leg. a. Venous obstruction of b. Claudication due to venous abnormalities in c. Ischemia caused by a partial blockage of an artery supplying d. Ischemia caused by the complete blockage of an artery supplying The nurse is reviewing venous blood flow patterns. Which of these statements best describes the mechanism(s) by which venous blood returns to the heart? a. Intraluminal valves ensure unidirectional flow toward the heart. b. Contracting skeletal muscles milk blood distally toward the veins. c. High-pressure system of the heart helps facilitate venous return. d. Increased thoracic pressure and decreased abdominal pressure facilitate venous return to the heart. Which of these veins are responsible for most of the venous return in the arm? a. Deep b. Ulnar c. Subclavian d. Superficial A 70-year-old patient is scheduled for open-heart surgery. The surgeon plans to use the great saphenous vein for the coronary bypass grafts. The patient asks, "What happens to my circulation when the veins are removed?" The nurse should reply: a. "Venous insufficiency is a common problem after this type of surgery." b. "Oh, you have lots of veins—you won't even notice that it has been removed." c. "You will probably experience decreased circulation after the vein is removed." d. "This vein can be removed without harming your circulation because the deeper veins in your leg are in good condition." The nurse is reviewing risk factors for venous disease. Which of these situations best describes a person at highest risk for development of venous disease? a. Woman in her second month of pregnancy b. Person who has been on bed rest for 4 days c. Person with a 30-year, 1 pack per day smoking habit d. Older adult taking anticoagulant medication The nurse is teaching a review class on the lymphatic system. A participant shows correct understanding of the material with which statement? a. "Lymph flow is propelled by the contraction of the heart." b. "The flow of lymph is slow, compared with that of the blood." c. "One of the functions of the lymph is to absorb lipids from the biliary tract." d. "Lymph vessels have no valves; therefore, lymph fluid flows freely from the tissue spaces into the bloodstream." When performing an assessment of a patient, the nurse notices the presence of an enlarged right epitrochlear lymph node. What should the nurse do next? a. Assess the patient's abdomen, and notice any tenderness. b. Carefully assess the cervical lymph nodes, and check for any enlargement. c. Ask additional health history questions regarding any recent ear infections or sore throats. d. Examine the patient's lower arm and hand, and check for the presence of infection or lesions. A 35-year-old man is seen in the clinic for an infection in his left foot. Which of these findings should the nurse expect to see during an assessment of this patient? a. Hard and fixed cervical nodes b. Enlarged and tender inguinal nodes c. Bilateral enlargement of the popliteal nodes d. Pelletlike nodes in the supraclavicular region The nurse is examining the lymphatic system of a healthy 3-year-old child. Which finding should the nurse expect? a. Excessive swelling of the lymph nodes b. Presence of palpable lymph nodes c. No palpable nodes because of the immature immune system of a child d. Fewer numbers and a smaller size of lymph nodes compared with those of an adult During an assessment of an older adult, the nurse should expect to notice which finding as a normal physiologic change associated with the aging process? a. Hormonal changes causing vasodilation and a resulting drop in blood pressure b. Progressive atrophy of the intramuscular calf veins, causing venous insufficiency c. Peripheral blood vessels growing more rigid with age, producing a rise in systolic blood pressure d. Narrowing of the inferior vena cava, causing low blood flow and increases in venous pressure resulting in varicosities A 67-year-old patient states that he recently began to have pain in his left calf when climbing the 10 stairs to his apartment. This pain is relieved by sitting for about 2 minutes; then he is able to resume his activities. The nurse interprets that this patient is most likely experiencing: a. Claudication. b. Sore muscles. c. Muscle cramps. d. Venous insufficiency. A patient complains of leg pain that wakes him at night. He states that he "has been having problems" with his legs. He has pain in his legs when they are elevated that disappears when he dangles them. He recently noticed "a sore" on the inner aspect of the right ankle. On the basis of this history information, the nurse interprets that the patient is most likely experiencing: a. Pain related to lymphatic abnormalities. b. Problems related to arterial insufficiency. c. Problems related to venous insufficiency. d. Pain related to musculoskeletal abnormalities. Night leg pain is common in aging adults. It may indicate the ischemic rest pain of peripheral vascular disease. Alterations in arterial circulation cause pain that becomes worse with leg elevation and is eased when the extremity is dangled. During an assessment, the nurse uses the "profile sign" to detect: a. Pitting edema. b. Early clubbing. c. Symmetry of the fingers. d. Insufficient capillary refill. The nurse is performing an assessment on an adult. The adult's vital signs are normal and capillary refill time is 5 seconds. What should the nurse do next? a. Ask the patient about a history of frostbite. b. Suspect that the patient has venous insufficiency. c. Consider this a delayed capillary refill time, and investigate further. d. Consider this a normal capillary refill time that requires no further assessment. Normal capillary refill time is less than 1 to 2 seconds. The following conditions can skew the findings: a cool room, decreased body temperature, cigarette smoking, peripheral edema, and anemia. When assessing a patient the nurse notes that the left femoral pulse as diminished, 1+/4+. What should the nurse do next? a. Document the finding. b. Auscultate the site for a bruit. c. Check for calf pain. d. Check capillary refill in the toes. If a pulse is weak or diminished at the femoral site, the nurse should auscultate for a bruit. Presence of a bruit, or turbulent blood flow, indicates partial occlusion. The other responses are not correct. When performing a peripheral vascular assessment on a patient, the nurse is unable to palpate the ulnar pulses. The patient's skin is warm and capillary refill time is normal. The nurse should next: a. Check for the presence of claudication. b. Refer the individual for further evaluation. c. Consider this finding as normal, and proceed with the peripheral vascular evaluation. d. Ask the patient if he or she has experienced any unusual cramping or tingling in the arm. - It is not usually necessary to palpate the ulnar pulses. The ulnar pulses are often not palpable in the normal person. The other responses are not correct. The nurse is assessing the pulses of a patient who has been admitted for untreated hyperthyroidism. The nurse should expect to find a(n) _____ pulse. a. Normal b. Absent c. Bounding d. Weak, thready The nurse is preparing to perform a modified Allen test. Which is an appropriate reason for this test? a. To measure the rate of lymphatic drainage b. To evaluate the adequacy of capillary patency before venous blood draws c. To evaluate the adequacy of collateral circulation before cannulating the radial artery d. To evaluate the venous refill rate that occurs after the ulnar and radial arteries are temporarily occluded A modified Allen test is used to evaluate the adequacy of collateral circulation before the radial artery is cannulated. The other responses are not reasons for a modified Allen test. A patient has been diagnosed with venous stasis. Which of these findings would the nurse most likely observe? a. Unilateral cool foot b. Thin, shiny, atrophic skin c. Pallor of the toes and cyanosis of the nail beds d. Brownish discoloration to the skin of the lower leg A brown discoloration occurs with chronic venous stasis as a result of hemosiderin deposits (a by-product of red blood cell degradation). Pallor, cyanosis, atrophic skin, and unilateral coolness are all signs associated with arterial problems. The nurse is attempting to assess the femoral pulse in an obese patient. Which of these actions would be most appropriate? a. The patient is asked to assume a prone position. b. The patient is asked to bend his or her knees to the side in a froglike position. c. The nurse firmly presses against the bone with the patient in a semi-Fowler position. d. The nurse listens with a stethoscope for pulsations; palpating the pulse in an obese person is extremely difficult. To help expose the femoral area, particularly in obese people, the nurse should ask the person to bend his or her knees to the side in a froglike position. When auscultating over a patient's femoral arteries the nurse notices the presence of a bruit on the left side. The nurse knows that: a. Are often associated with venous disease. b. Occur in the presence of lymphadenopathy. c. In the femoral arteries are caused by hypermetabolic states. d. Occur with turbulent blood flow, indicating partial occlusion. - A bruit occurs with turbulent blood flow and indicates partial occlusion of the artery. The other responses are not correct. How should the nurse document mild, slight pitting edema present at the ankles of a pregnant patient? a. 1+/0-4+ b. 3+/0-4+ c. 4+/0-4+ d. Brawny edema 1+/0-4+ If pitting edema is present, then the nurse should grade it on a scale of 1+ (mild) to 4+ (severe). Brawny edema appears as nonpitting edema and feels hard to the touch. A patient has hard, nonpitting edema of the left lower leg and ankle. The right leg has no edema. Based on these findings, the nurse recalls that: a. Nonpitting, hard edema occurs with lymphatic obstruction. b. Alterations in arterial function will cause edema. c. Phlebitis of a superficial vein will cause bilateral edema. d. Long-standing arterial obstruction will cause pitting edema. nonpitting, hard edema occurs with lymphatic obstruction Unilateral edema occurs with occlusion of a deep vein and with unilateral lymphatic obstruction. With these factors, the edema is nonpitting and feels hard to the touch (brawny edema). When assessing a patient's pulse, the nurse notes that the amplitude is weaker during inspiration and stronger during expiration. When the nurse measures the blood pressure, the reading decreases 20 mm Hg during inspiration and increases with expiration. This patient is experiencing pulses: a. Alternans. b. Bisferiens. c. Bigeminus. d. Paradoxus. paradoxus. In pulsus paradoxus, beats have a weaker amplitude with inspiration and a stronger amplitude with expiration. It is best determined during blood pressure measurement; reading decreases (>10 mm Hg) during inspiration and increases with expiration. During an assessment the nurse has elevated a patient's legs 12 inches off the table and has had him wag his feet to drain off venous blood. After helping him to sit up and dangle his legs over the side of the table, the nurse should expect a normal finding at this point would be: a. Significant elevational pallor. b. Venous filling within 15 seconds. c. No change in the coloration of the skin. d. Color returning to the feet within 20 seconds of assuming a sitting position. venous filling within 15 seconds. In this test it normally takes 10 seconds or less for the color to return to the feet and 15 seconds for the veins of the feet to fill. Marked elevational pallor as well as delayed venous filling occurs with arterial insufficiency. During a clinic visit, a woman in her seventh month of pregnancy complains that her legs feel "heavy in the calf" and that she often has foot cramps at night. The nurse notices that the patient has dilated, tortuous veins in her lower legs. Which condition is reflected by these findings? a. Deep-vein thrombophlebitis b. Varicose veins c. Lymphedema d. Raynaud phenomenon Varicose veins Superficial varicose veins are caused by incompetent distant valves on veins, which results in reflux of blood and producing dilated, tortuous veins. They are more common in women, and pregnancy can also be a cause. Symptoms include aching, heaviness in the calf, easy fatigability, and night leg or foot cramps. Dilated, tortuous veins are seen on assessment. See Table 20-5 for the description of deep vein thrombophlebitis. See Table 20-2 for descriptions of Raynaud's phenomenon and lymphedema. During an assessment, the nurse notices that a patient's left arm is swollen from the shoulder down to the fingers, with nonpitting brawny edema. The right arm is normal. The patient had a left-sided mastectomy 1 year ago. The nurse suspects which problem? a. Venous stasis b. Lymphedema c. Arteriosclerosis d. Deep-vein thrombosis Lymphedema Lymphedema after breast cancer causes unilateral swelling and nonpitting brawny edema, with overlying skin indurated. It is caused by the removal of lymph nodes with breast surgery or damage to lymph nodes and channels with radiation therapy for breast cancer, and it can impede drainage of lymph. The other responses are not correct. The nurse is preparing to assess the ankle-brachial index (ABI) of a patient. Which statement about the ABI is true? a. Normal ABI indices are from 0.5 to 1.0. b. Normal ankle pressure is slightly lower than the brachial pressure. c. The ABI is a reliable measurement of peripheral vascular disease in individuals with diabetes. d. An ABI of 0.9 to 0.7 indicates the presence of peripheral vascular disease and mild claudication. n ABI of 0.90 to 0.70 indicates the presence of peripheral vascular disease and mild claudication. Use of the Doppler stethoscope is a noninvasive way to determine the extent of peripheral vascular disease. The normal ankle pressure is slightly greater than or equal to the brachial pressure. An ABI of 0.90 to 0.70 indicates the presence of peripheral vascular disease and mild claudication. The ABI is less reliable in patients with diabetes mellitus because of claudication, which makes the arteries noncompressible and may give a falsely high ankle pressure. The nurse is performing a well-child check on a 5-year-old boy. He has no current history that would lead the nurse to suspect illness. His medical history is unremarkable, and he received immunizations 1 week ago. Which of these findings should be considered normal in this situation? a. Enlarged, warm, and tender nodes b. Lymphadenopathy of the cervical nodes c. Palpable firm, small, shotty, mobile, and nontender lymph nodes d. Firm, rubbery, and large nodes, somewhat fixed to the underlying tissue Palpable firm, small, shotty, mobile, nontender lymph nodes Palpable lymph nodes are often normal in children and infants. They are small, firm, shotty, mobile, and nontender. Vaccinations can produce lymphadenopathy. Enlarged, warm, tender nodes indicate current infection. When using a Doppler ultrasonic stethoscope, the nurse recognizes venous flow when which sound is heard? a. Low humming sound b. Regular "lub, dub" pattern c. Swishing, whooshing sound d. Steady, even, flowing sound Swishing, whooshing sound When using the Doppler ultrasonic stethoscope, the pulse site is found when one hears a swishing, whooshing sound. The nurse is describing a weak, thready pulse on the documentation flow sheet. Which statement is correct? a. "Is easily palpable; pounds under the fingertips." b. "Has greater than normal force, then suddenly collapses." c. "Is hard to palpate, may fade in and out, and is easily obliterated by pressure." d. "Rhythm is regular, but force varies with alternating beats of large and small amplitude." "Hard to palpate, may fade in and out, easily obliterated by pressure." A weak, thready pulse is hard to palpate, may fade in and out, and is easily obliterated by pressure. It is associated with decreased cardiac output and peripheral arterial disease. During an assessment, a patient tells the nurse that her fingers often change color when she goes out in cold weather. She describes these episodes as her fingers first turning white, then blue, then red with a burning, throbbing pain. The nurse suspects that she is experiencing: a. Lymphedema. b. Raynaud disease. c. Deep-vein thrombosis. d. Chronic arterial insufficiency. Raynaud's disease. The condition with episodes of abrupt, progressive tricolor change of the fingers in response to cold, vibration, or stress is known as Raynaud's disease. Lymphedema is described in Table 20-2; deep vein thrombosis is described in Table 20-5; chronic arterial insufficiency is described in Table 20-4. During a routine office visit, a patient takes off his shoes and shows the nurse "this awful sore that won't heal." On inspection, the nurse notes a 3-cm round ulcer on the left great toe, with a pale ischemic base, well-defined edges, and no drainage. The nurse should assess for other signs and symptoms of: a. Varicosities. b. Venous stasis ulcer. c. Arterial ischemic ulcer. d. Deep-vein thrombophlebitis. n arterial ischemic ulcer. Arterial ischemic ulcers occur at toes, metatarsal heads, heels, and lateral ankle, and they are characterized by a pale ischemic base, well-defined edges, and no bleeding. See Table 20-5 for a description of varicose veins and deep vein thrombophlebitis. See Table 20-4 for a description of venous stasis ulcers. The nurse is reviewing an assessment of a patient's peripheral pulses and notices that the documentation states that the radial pulses are "2+." The nurse recognizes that this reading indicates what type of pulse? a. Bounding b. Normal c. Weak d. Absent Normal When documenting the force, or amplitude, of pulses, 3+ indicates an increased, full, or bounding pulse, 2+ indicates a normal pulse, 1+ indicates a weak pulse, and 0 indicates an absent pulse. A patient is recovering from several hours of orthopedic surgery. During an assessment of the patient's lower legs, the nurse will monitor for signs of acute venous symptoms. Signs of acute venous symptoms include which of the following? Select all that apply. a. Intense, sharp pain, with the deep muscle tender to the touch b. Aching, tired pain, with a feeling of fullness c. Pain that is worse at the end of the day d. Sudden onset e. Warm, red, and swollen calf f. Pain that is relieved with elevation of the leg . Intense, sharp pain, with the deep muscle tender to touch D. Sudden onset E. Warm, red, and swollen calf Signs and symptoms of acute venous problems include pain in the calf that has a sudden onset and that is intense and sharp with tenderness in the deep muscle when touched. The calf is warm, red, and swollen. The other options are symptoms of chronic venous problems. A patient has been admitted with chronic arterial symptoms. During the assessment, the nurse should expect which findings? Select all that apply. a. Patient has a history of diabetes and cigarette smoking. b. Skin of the patient is pale and cool. c. His ankles have two small, weeping ulcers. d. Patient works long hours sitting at a computer desk. e. He states that the pain gets worse when walking. f. Patient states that the pain is worse at the end of the day. . The patient has a history of diabetes and cigarette smoking. B. The patient's skin is pale and cool. E. The patient states that the pain gets worse when walking. See Table 20-3. Patients with chronic arterial symptoms often have a history of smoking and diabetes (among other risk factors). The pain has a gradual onset, with exertion, and is relieved with rest or dangling. The skin appears cool and pale. The other responses reflect chronic venous problems. The external male genital structures include the: A) testis. B) scrotum. C) epididymis. D) vas deferens. - The external male genital structures include the penis and scrotum. The testis, epididymis, and vas deferens are internal structures. An accessory glandular structure for the male genital organs is the: A) testis. B) penis. C) prostate. D) vas deferens. Glandular structures accessory to the male genital organs are the prostate, seminal vesicles, and bulbourethral glands. Which of these statements is true regarding the penis? A) The urethral meatus is located on the ventral side of the penis. B) The prepuce is the fold of foreskin covering the shaft of the penis. C) The penis is composed of two cylindrical columns of erectile tissue. D) The corpus spongiosum expands into a cone of erectile tissue called the glans. At the distal end of the shaft, the corpus spongiosum expands into a cone of erectile tissue, the glans. The penis is composed of three cylindrical columns of erectile tissue. The prepuce is skin that covers the glans of the penis. The urethral meatus forms at the tip of the glans. When performing a genital examination on a 25-year-old man, the nurse notices deeply pigmented, wrinkled scrotal skin with large sebaceous follicles. On the basis of this information the nurse would: A) squeeze the glans to check for the presence of discharge. B) consider this a normal finding and proceed with the examination. C) assess the testicles for the presence of masses or painless lumps. D) obtain a more detailed history focusing on any scrotal abnormalities the patient has noticed. - After adolescence, the scrotal skin is deeply pigmented and has large sebaceous follicles. The scrotal skin looks corrugated. Which of these statements about the testes is true? A) The lymphatics of the testes drain into the abdominal lymph nodes. B) The vas deferens is located along the inferior portion of each testis. C) The right testis is lower than the left because the right spermatic cord is longer. D) The cremaster muscle contracts in response to cold and draws the testicles closer to the body. When it is cold, the cremaster muscle contracts, which raises the scrotal sac and brings the testes closer to the body to absorb heat necessary for sperm viability. The lymphatics of the testes drain into the inguinal lymph nodes. The vas deferens is located along the upper portion of each testis. The left testis is lower than the right because the left spermatic cord is longer. A male patient with possible fertility problems asks the nurse where sperm is produced. The nurse knows that sperm production occurs in the: A) testes. B) prostate. C) epididymis. D) vas deferens. Sperm production occurs in the testes, not in the other structures listed. A 62-year-old man states that his doctor told him that he has an "inguinal hernia." He asks the nurse to explain what a hernia is. The nurse should: A) tell him not to worry and that most men his age develop hernias. B) explain that a hernia is often the result of prenatal growth abnormalities. C) refer him to his physician for additional consultation because the physician made the initial diagnosis. D) explain that a hernia is a loop of bowel protruding through a weak spot in the abdominal muscles. A hernia is a loop of bowel protruding through a weak spot in the musculature. The other options are not correct responses to the patient's question. The mother of a 10-year-old boy asks the nurse to discuss the recognition of puberty. The nurse should reply by saying: A) "Puberty usually begins about age fifteen." B) "The first sign of puberty is enlargement of the testes." C) "Penis size does not increase until about the age of sixteen." D) "The development of pubic hair precedes testicular or penis enlargement." - Puberty begins sometime between ages 9 1/2 and 13 1/2 years. The first sign is enlargement of the testes. Next, pubic hair appears and then penis size increases. During an examination of an aging male, the nurse recognizes that normal changes to expect would be: A) enlarged scrotal sac. B) increased pubic hair. C) decreased penis size. D) increased rugae over the scrotum. In the aging male the amount of pubic hair decreases, the penis size decreases, and there is a decrease in the rugae over the scrotal sac. The scrotal sac does not enlarge. An older man is concerned about his sexual performance. The nurse knows that in the absence of disease, a withdrawal from sexual activity later in life may be due to: A) side effects of medications. B) decreased libido with aging. C) decreased sperm production. D) decreased pleasure from sexual intercourse. In the absence of disease, a withdrawal from sexual activity may be due to side effects of medications such as antihypertensives, antidepressants, or sedatives. The other options are not correct. A newborn baby boy is about to have a circumcision. The nurse knows that indications for circumcision include: A) cultural and religious beliefs. B) prevention of testicular cancer. C) improving the sperm count later in life. D) preventing dysuria Indications for circumcision include cultural and religious beliefs, prevention of phimosis and inflammation of the glans penis and foreskin, decreasing the incidence of cancer of the penis, and decreasing the incidence of urinary tract infections in infancy. A 59-year-old patient has been diagnosed with prostatitis and is being seen at the clinic for complaints of burning and pain during urination. He is experiencing: A) dysuria. B) nocturia. C) polyuria. D) hematuria. Dysuria or burning with urination is common with acute cystitis, prostatitis, and urethritis. Nocturia is voiding during the night. Polyuria is voiding in excessive quantities. Hematuria is voiding with blood in the urine A 45-year-old mother of two children is seen at the clinic for complaints of "losing my urine when I sneeze." The nurse documents that she is experiencing: A) urinary frequency. B) enuresis. C) stress incontinence. D) urge incontinence. Stress incontinence is involuntary urine loss with physical strain, sneezing, or coughing that occurs due to weakness of the pelvic floor. Urinary frequency is urinating more times than usual (more than 5 to 6 times per day). Enuresis is involuntary passage of urine at night after age 5 to 6 years (bed wetting). Urge incontinence is involuntary urine loss from overactive detrusor muscle in the bladder. It contracts, causing an urgent need to void. When the nurse is conducting sexual history from a male adolescent, which statement would be most appropriate to use at the beginning of the interview? A) "Do you use condoms?" B) "You don't masturbate, do you?" C) "Have you had sex in the last 6 months?" D) "Often boys your age have questions about sexual activity." Start the interview with a permission statement. This conveys that it is normal and all right to think or feel a certain way. Avoid sounding judgmental. Which of these statements is most appropriate when the nurse is obtaining a genitourinary history from an elderly man? A) "Do you need to get up at night to urinate?" B) "Do you experience nocturnal emissions, or 'wet dreams'?" C) "Do you know how to perform a testicular self-examination?" D) "Has anyone ever touched your genitals when you did not want them to?" The elderly male patient should be asked about the presence of nocturia. This may be due to diuretic medication, fluid retention from mild heart failure or varicose veins, or fluid ingestion 3 hours before bedtime, especially coffee and alcohol. The other questions are more appropriate for younger males. When the nurse is performing a genital examination on a male patient, the patient has an erection. The nurse's most appropriate action or response is to: A) ask the patient if he would like someone else to examine him. B) continue with the examination as though nothing has happened. C) stop the examination, leave the room while stating that the examination will resume at a later time. D) reassure the patient that this is a normal response and continue with the examination. When the male patient has an erection, the nurse should reassure the patient that this is a normal physiologic response to touch and proceed with the rest of the examination. The other responses are not correct and may be perceived as judgmental. The nurse is examining the glans and knows that which of these is a normal finding for this area? A) The meatus may have a slight discharge when the glans is compressed. B) Hair is without pest inhabitants. C) The skin is wrinkled and without lesions. D) Smegma may be present under the foreskin of an uncircumcised male. The glans looks smooth and without lesions and does not have hair. The meatus should not have any discharge when the glans is compressed. Some cheesy smegma may have collected under the foreskin of an uncircumcised male. When performing a genitourinary assessment, the nurse notices that the urethral meatus is positioned ventrally. This finding is: A) called hypospadias. B) the result of phimosis. C) probably due to a stricture. D) often associated with aging. The nurse is performing a genital examination on a male patient and notices urethral drainage. When collecting urethral discharge for microscopic examination and culture, the nurse should: A) ask the patient to urinate into a sterile cup. B) ask the patient to obtain a specimen of semen. C) insert a cotton-tipped applicator into the urethra. D) compress the glans between the examiner's thumb and forefinger and collect any discharge. When assessing the scrotum of a male patient, the nurse notices the presence of multiple firm, nontender, yellow 1-cm nodules. The nurse knows that these nodules are most likely: A) from urethritis. B) sebaceous cysts. C) subcutaneous plaques. D) from inflammation of the epididymis. When performing a scrotal assessment, the nurse notices that the scrotal contents transilluminate and show a red glow. On the basis of this finding the nurse would: A) assess the patient for the presence of a hernia. B) suspect the presence of serous fluid in the scrotum. C) consider this normal and proceed with the examination. D) refer the patient for evaluation of a mass in the scrotum. When the nurse is performing a genital examination on a male patient, which of these actions is correct? A) Auscultate for the presence of a bruit over the scrotum. B) Palpate for the vertical chain of lymph nodes along the groin inferior to the inguinal ligament. C) Palpate the inguinal canal only if there is a bulge present in the inguinal region during inspection. D) Have the patient shift his weight onto the left (unexamined) leg when palpating for a hernia on the right side. The nurse is aware that which of these statements is true regarding the incidence of testicular cancer? A) Testicular cancer is the most common cancer in men aged 30 to 50 years. B) The early symptoms of testicular cancer are pain and induration. C) Men with a history of cryptorchidism are at greatest risk for development of testicular cancer. D) The cure rate for testicular cancer is low. The nurse is describing how to perform a testicular self-examination to a patient. Which of these statements is most appropriate? A) "A good time to examine your testicles is just before you take a shower." B) "If you notice an enlarged testicle or a painless lump, call your health care provider." C) "The testicle is egg shaped and movable. It feels firm and has a lumpy consistency." D) "Perform a testicular exam at least once a week to detect the early stages of testicular cancer." A 2-month-old uncircumcised infant has been brought to the clinic for a well-baby checkup. How would the nurse proceed with the genital examination? A) Elicit the cremasteric reflex. B) Assess the glans for redness or lesions. C) Avoid retracting the foreskin until the infant is 3 months old. D) Note any dirt or smegma that has collected under the foreskin. A 2-year-old boy has been diagnosed with "physiologic cryptorchidism." Given this diagnosis, during assessment the nurse will most likely observe: A) testes that are hard and painful to palpation. B) an atrophic scrotum and absence of the testis bilaterally. C) an absence of the testis in the scrotum, but the testis can be milked down. D) testes that migrate into the abdomen when the child squats or sits cross-legged. The nurse knows that a common assessment finding in a boy younger than 2 years old is: A) an inflamed and tender spermatic cord. B) the presence of a hernia in the scrotum. C) a penis that looks large in relation to the scrotum. D) the presence of a hydrocele, or fluid in the scrotum. During an examination of an aging male, the nurse recognizes that normal changes to expect would be: A) a change in scrotal color. B) a decrease in the size of the penis. C) enlargement of the testes and scrotum. D) an increase in the number of rugae over the scrotal sac. When performing a genital assessment on a middle-aged man, the nurse notices multiple soft, moist, painless papules in the shape of cauliflower-like patches scattered across the shaft of the penis. These lesions are characteristic of: A) carcinoma. B) syphilitic chancres. C) genital herpes. D) genital warts. A 15-year-old boy is seen in the clinic for complaints of "dull pain and pulling" in the scrotal area. On examination the nurse palpates a soft, irregular mass posterior to and above the testis on the left. This mass collapses when the patient is supine and refills when he is upright. This description is consistent with: A) epididymitis. B) spermatocele. C) testicular torsion. D) varicocele. When performing a genitourinary assessment on a 16-year-old boy, the nurse notices a swelling in the scrotum that increases with increased intra-abdominal pressure and decreases when he is lying down. The patient complains of pain when straining. The nurse knows that this description is most consistent with a(n) _____ hernia. A) femoral B) incisional C) direct inguinal D) indirect inguinal When the nurse is performing a testicular examination on a 25-year-old man, which of these findings is considered normal? A) Nontender subcutaneous plaques B) A scrotal area that is dry, scaly, and nodular C) Testes that feel oval and movable and are slightly sensitive to compression D) A single, hard, circumscribed, movable mass, less than 1 cm under the surface of the testes - The nurse is inspecting the scrotum and testes of a 43-year-old man. Which finding would require additional follow-up and evaluation? A) The skin on the scrotum is taut. B) The left testicle hangs lower than the right testicle. C) The scrotal skin has yellowish 1-cm nodules that are firm and nontender. D) The testes move closer to the body in response to cold temperatures. A 55-year-old man is experiencing severe pain of sudden onset in the scrotal area. It is somewhat relieved by elevation. On examination the nurse notices an enlarged, red scrotum that is very tender to palpation. It is difficult to distinguish the epididymis from the testis, and the scrotal skin is thick and edematous. This description is consistent with which of these? A) Varicocele B) Epididymitis C) Spermatocele D) Testicular torsion The nurse is performing a genitourinary assessment on a 50-year-old obese male laborer. On examination the nurse notices a painless round swelling close to the pubis in the area of the internal inguinal ring that is easily reduced when the individual is supine. These findings are most consistent with a(n) _____ hernia. A) scrotal B) femoral C) direct inguinal D) indirect inguinal The nurse is providing patient teaching about an erectile dysfunction drug. One of the drug's potential side effects is prolonged, painful erection of the penis without sexual stimulation, which is known as: A) orchitis. B) stricture. C) phimosis. D) priapism. During an examination, the nurse notices that a male patient has a red, round, superficial ulcer with a yellowish serous discharge on his penis. On palpation, the nurse finds a nontender base that feels like a small button between the thumb and fingers. At this point the nurse suspects that this patient has: A) genital warts. B) a herpes infection. C) a syphilitic chancre. D) a carcinoma lesion. During a health history, a patient tells the nurse that he has trouble in starting his urine stream. This problem is known as: A) urgency. B) dribbling. C) frequency. D) hesitancy During a genital examination, the nurse notices that a male patient has clusters of small vesicles on the glans, surrounded by erythema. The nurse recognizes that these lesions are: A) Peyronie disease. B) genital warts. C) genital herpes. D) syphilitic cancer. During a physical examination, the nurse finds that a male patient's foreskin is fixed and tight and will not retract over the glans. The nurse recognizes that this condition is: A) phimosis. B) epispadias. C) urethral stricture. D) Peyronie's disease. A 55-year-old man is in the clinic for a yearly check-up. He is worried because his father died of prostate cancer. The nurse knows that which tests should be done at this time? Select all that apply. A) Blood test for prostate-specific antigen B) Urinalysis C) Transrectal ultrasound D) Digital rectal examination E) Prostate biopsy A 16-year-old boy is brought to the clinic for a problem that he refused to let his mother see. The nurse examines him, and finds that he has scrotal swelling on the left side. He had the mumps the previous week, and the nurse suspects that he has orchitis. Which of the assessment findings below support this diagnosis? Select all that apply. A) Swollen testis B) Mass does transilluminate C) Mass does not transilluminate D) Nontender upon palpation E) Tender upon palpation F) Scrotal skin is reddened The nurse is percussing the seventh right intercostal space at the midclavicular line over the liver. Which sound should the nurse expect to hear? A) Dullness B) Tympany C) Resonance D) Hyperresonance Which structure is located in the left lower quadrant of the abdomen? A) Liver B) Duodenum C) Gallbladder D) Sigmoid colon A patient is having difficulty in swallowing medications and food. The nurse would document that this patient has: A) aphasia. B) dysphasia. C) dysphagia. D) anorexia. The nurse suspects that a patient has a distended bladder. How should the nurse assess for this condition? A) Percuss and palpate in the lumbar region. B) Inspect and palpate in the epigastric region. C) Auscultate and percuss in the inguinal region. D) Percuss and palpate the midline area above the suprapubic bone. The nurse is aware that one change that may occur in the gastrointestinal system of an aging adult is: A) increased salivation. B) increased liver size. C) increased esophageal emptying. D) decreased gastric acid secretion. A 22-year-old man comes to the clinic for an examination after falling off his motorcycle and landing on his left side on the handlebars. The nurse suspects that he may have injured his spleen. Which of these statements is true regarding assessment of the spleen in this situation? A) The spleen can be enlarged as a result of trauma. B) The spleen is normally felt upon routine palpation. C) If an enlarged spleen is noticed, then the nurse should palpate thoroughly to determine size. D) An enlarged spleen should not be palpated because it can rupture easily. - A patient's abdomen is bulging and stretched in appearance. The nurse should describe this finding as: A) obese. B) herniated. C) scaphoid. D) protuberant. - The nurse is describing a scaphoid abdomen. To the horizontal plane, a scaphoid contour of the abdomen depicts a _____ profile. A) flat B) convex C) bulging D) concave While examining a patient, the nurse observes abdominal pulsations between the xiphoid and umbilicus. The nurse would suspect that these are: A) pulsations of the renal arteries. B) pulsations of the inferior vena cava. C) normal abdominal aortic pulsations. D) increased peristalsis from a bowel obstruction. A patient has hypoactive bowel sounds. The nurse knows that a potential cause of hypoactive bowel sounds is: A) diarrhea. B) peritonitis. C) laxative use. D) gastroenteritis. . The nurse is watching a new graduate nurse perform auscultation of a patient's abdomen. Which statement by the new graduate shows a correct understanding of the reason auscultation precedes percussion and palpation of the abdomen? A) "We need to determine areas of tenderness before using percussion and palpation." B) "It prevents distortion of bowel sounds that might occur after percussion and palpation." C) "It allows the patient more time to relax and therefore be more comfortable with the physical examination." D) "This prevents distortion of vascular sounds such as bruits and hums that might occur after percussion and palpation." The nurse is listening to bowel sounds. Which of these statements is true of bowel sounds? A) They are usually loud, high-pitched, rushing, tinkling sounds. B) They are usually high-pitched, gurgling, irregular sounds. C) They sound like two pieces of leather being rubbed together. D) They originate from the movement of air and fluid through the large intestine. The physician comments that a patient has abdominal borborygmi. The nurse knows that this term refers to: A) a loud continuous hum. B) a peritoneal friction rub. C) hypoactive bowel sounds. D) hyperactive bowel sounds. During an abdominal assessment, the nurse would consider which of these findings as normal? A) The presence of a bruit in the femoral area B) A tympanic percussion note in the umbilical region C) A palpable spleen between the ninth and eleventh ribs in the left midaxillary line D) A dull percussion note in the left upper quadrant at the midclavicular line The nurse is assessing the abdomen of a pregnant woman who is complaining of having "acid indigestion" all the time. The nurse knows that esophageal reflux during pregnancy can cause: A) diarrhea. B) pyrosis. C) dysphagia. D) constipation. The nurse is performing percussion during an abdominal assessment. Percussion notes heard during the abdominal assessment may include: A) flatness, resonance, and dullness. B) resonance, dullness, and tympany. C) tympany, hyperresonance, and dullness. D) resonance, hyperresonance, and flatness. An older patient has been diagnosed with pernicious anemia. The nurse knows that this condition could be related to: A) increased gastric acid secretion. B) decreased gastric acid secretion. C) delayed gastrointestinal emptying time. D) increased gastrointestinal emptying time. - A patient is complaining of a sharp pain along the costovertebral angles. The nurse knows that this symptom is most often indicative of: A) ovary infection. B) liver enlargement. C) kidney inflammation. D) spleen enlargement. A nurse notices that a patient has ascites, which indicates the presence of: A) fluid. B) feces. C) flatus. D) fibroid tumors. The nurse knows that during an abdominal assessment, deep palpation is used to determine: A) bowel motility. B) enlarged organs. C) superficial tenderness. D) overall impression of skin surface and superficial musculature. The nurse notices that a patient has had a black, tarry stool and recalls that a possible cause would be: A) gallbladder disease. B) overuse of laxatives. C) gastrointestinal bleeding. D) localized bleeding around the anus. - During an abdominal assessment, the nurse elicits tenderness on light palpation in the right lower quadrant. The nurse interprets that this finding could indicate a disorder of which of these structures? A) Spleen B) Sigmoid C) Appendix D) Gallbladder The nurse is assessing the abdomen of an aging adult. Which of these statements regarding the aging adult and abdominal assessment is true? A) The abdominal tone is increased. B) The abdominal musculature is thinner. C) Abdominal rigidity with acute abdominal conditions is more common. D) The aging person complains of more pain with an acute abdominal condition than a younger person would. During an assessment of a newborn infant, the nurse recalls that pyloric stenosis would be manifested by: A) projectile vomiting. B) hypoactive bowel activity. C) palpable olive-sized mass in right lower quadrant. D) pronounced peristaltic waves crossing from right to left. To detect diastasis recti, the nurse should have the patient perform which of these maneuvers? A) Relax in the supine position. B) Raise the arms in the left lateral position. C) Raise the arms over the head while supine. D) Raise the head while remaining supine. The nurse is reviewing the assessment of an aortic aneurysm. Which of these statements is true regarding an aortic aneurysm? A) A bruit is absent. B) Femoral pulses are increased. C) A pulsating mass is usually present. D) Most are located below the umbilicus. During an abdominal assessment, the nurse is unable to hear bowel sounds in a patient's abdomen. Before reporting this finding as "silent bowel sounds" the nurse should listen for at least: A) 1 minute. B) 5 minutes. C) 10 minutes. D) 2 minutes in each quadrant. A patient is suspected of having inflammation of the gallbladder, or cholecystitis. The nurse should conduct which of these techniques to assess for this condition? A) Obturator test B) Test for Murphy's sign C) Assess for rebound tenderness D) Iliopsoas muscle test Just before going home, a new mother asks the nurse about the infant's umbilical cord. Which of these statements is correct? A) "It should fall off by 10 to 14 days." B) "It will soften before it falls off." C) "It contains two veins and one artery." D) "Skin will cover the area within 1 week." Which of these percussion findings would the nurse expect to find in a patient with a large amount of ascites? A) Dullness across the abdomen B) Flatness in the right upper quadrant C) Hyperresonance in the left upper quadrant D) Tympany in the right and left lower quadrants A 40-year-old man states that his physician told him that he has a hernia. He asks the nurse to explain what a hernia is. Which response by the nurse is appropriate? A) "No need to worry. Most men your age develop hernias." B) "A hernia is a loop of bowel protruding through a weak spot in the abdominal muscles." C) "This hernia is a result of prenatal growth abnormalities that are just now causing problems." D) "I'll have to have your physician explain this to you." A 45-year-old man is in the clinic for a physical examination. During the abdominal assessment, the nurse percusses the abdomen and notices an area of dullness above the right costal margin of about 10 cm. The nurse should: A) document the presence of hepatomegaly. B) ask additional history questions regarding his alcohol intake. C) describe this as an enlarged liver and refer him to a physician. D) consider this a normal finding and proceed with the examination. When palpating the abdomen of a 20-year-old patient, the nurse notices the presence of tenderness in the left upper quadrant with deep palpation. Which of these structures is most likely to be involved? A) Spleen B) Sigmoid colon C) Appendix D) Gallbladder - The nurse is reviewing statistics for lactose intolerance. In the United States, the incidence of lactose intolerance is higher in adults of which ethnic group? A) African-Americans B) Hispanics C) Whites D) Asians The nurse is assessing a patient for possible peptic ulcer disease and knows that which condition often causes this problem? A) Hypertension B) Streptococcus infections C) History of constipation and frequent laxative use D) Frequent use of nonsteroidal antiinflammatory drugs During report, the student nurse hears that a patient has "hepatomegaly" and recognizes that this term refers to: A) an enlarged liver. B) an enlarged spleen. C) distended bowel. D) excessive diarrhea. During an assessment the nurse notices that a patient's umbilicus is enlarged and everted. It is midline, and there is no change in skin color. The nurse recognizes that the patient may have which condition? A) Intra-abdominal bleeding B) Constipation C) Umbilical hernia D) An abdominal tumor During an abdominal assessment, the nurse tests for a fluid wave. A positive fluid wave test occurs with: A) splenomegaly. B) distended bladder. C) constipation. D) ascites. The nurse is preparing to examine a patient who has been complaining of right lower quadrant pain. Which technique is correct during the assessment? The nurse should: A) examine the tender area first. B) examine the tender area last. C) avoid palpating the tender area. D) palpate the tender area first and then auscultate for bowel sounds. During a health history, the patient tells the nurse, "I have pain all the time in my stomach. It's worse two hours after I eat, but it gets better if I eat again!" The nurse suspects that the patient has which condition, based on these symptoms? A) Appendicitis B) Gastric ulcer C) Duodenal ulcer D) Cholecystitis The nurse suspects that a patient has appendicitis. Which of these procedures are appropriate for use when assessing for appendicitis or a perforated appendix? Select all that apply. A) Test for Murphy's sign. B) Test for Blumberg's sign. C) Test for shifting dullness. D) Perform iliopsoas muscle test. E) Test for fluid wave. [Show More]

Last updated: 11 months ago

Preview 1 out of 365 pages

Reviews( 0 )

Recommended For You

 Applied Science> TEST BANK > Test Bank For Physics for Scientists & Engineers with Modern Physics (Volume 1) 5th Edition (Global Edition) By Douglas C. Giancoli (All)

preview
Test Bank For Physics for Scientists & Engineers with Modern Physics (Volume 1) 5th Edition (Global Edition) By Douglas C. Giancoli

This is Original 5th Edition of Test Bank From Original Author. All Other Files in the market are fake/old Edition. Other Sellers Have changed old Edition Number to new But Test Bank is old Edition....

By eBookSmTb , Uploaded: Apr 23, 2024

$20

 Finance> TEST BANK > Options, Futures, and Other Derivatives 11th Edition By John C Hull (Test Bank (MCQs Only) All Chapters, 100% Original Verified, A+ Grade) (All)

preview
Options, Futures, and Other Derivatives 11th Edition By John C Hull (Test Bank (MCQs Only) All Chapters, 100% Original Verified, A+ Grade)

Options, Futures, and Other Derivatives, 11e John C Hull (Test Bank (MCQs Only) All Chapters, 100% Original Verified, A+ Grade) Options, Futures, and Other Derivatives, 11e John C Hull (Test Bank (M...

By eBookSmTb , Uploaded: Dec 22, 2023

$25

 Accounting> TEST BANK > Introduction to Health Care Finance and Accounting 2nd Edition By Carlene Harrison, William Harrison, Carol Taylor (Test Bank All Chapters, 100% Original Verified, A+ Grade) (All)

preview
Introduction to Health Care Finance and Accounting 2nd Edition By Carlene Harrison, William Harrison, Carol Taylor (Test Bank All Chapters, 100% Original Verified, A+ Grade)

Introduction to Health Care Finance and Accounting, 2e Carlene Harrison, William Harrison, Carol Taylor (Test Bank All Chapters, 100% Original Verified, A+ Grade) Introduction to Health Care Finance...

By eBookSmTb , Uploaded: Dec 22, 2023

$25

 Economics> TEST BANK > International Economics 18th Edition By Robert Carbaugh (Test Bank All Chapters, 100% Original Verified, A+ Grade) (All)

preview
International Economics 18th Edition By Robert Carbaugh (Test Bank All Chapters, 100% Original Verified, A+ Grade)

International Economics, 18e Robert Carbaugh (Test Bank All Chapters, 100% Original Verified, A+ Grade) International Economics, 18e Robert Carbaugh (Test Bank All Chapters, 100% Original Verified,...

By eBookSmTb , Uploaded: Dec 22, 2023

$17

 Accounting> TEST BANK > Intermediate Accounting Reporting and Analysis 4th Edition By James Wahlen, Jefferson Jones, Donald Pagach (Test Bank All Chapters, 100% Original Verified, A+ Grade) (All)

preview
Intermediate Accounting Reporting and Analysis 4th Edition By James Wahlen, Jefferson Jones, Donald Pagach (Test Bank All Chapters, 100% Original Verified, A+ Grade)

Intermediate Accounting Reporting and Analysis, 4e James Wahlen, Jefferson Jones, Donald Pagach (Test Bank All Chapters, 100% Original Verified, A+ Grade) Intermediate Accounting Reporting and Analys...

By eBookSmTb , Uploaded: Dec 22, 2023

$25

 Programming> TEST BANK > Fundamentals of Python First Programs 3rd Edition By Kenneth Lambert (Test Bank All Chapters, 100% Original Verified, A+ Grade) (All)

preview
Fundamentals of Python First Programs 3rd Edition By Kenneth Lambert (Test Bank All Chapters, 100% Original Verified, A+ Grade)

Fundamentals of Python First Programs, 3e Kenneth Lambert (Test Bank All Chapters, 100% Original Verified, A+ Grade) Fundamentals of Python First Programs, 3e Kenneth Lambert (Test Bank All Chapters,...

By eBookSmTb , Uploaded: Dec 21, 2023

$25

 Medical Studies> TEST BANK > Forensic Science, An Introduction to Scientific and Investigative Techniques 4th Edition By Suzanne Bell (Test Bank All Chapters, 100% original verified, A+ Grade) (All)

preview
Forensic Science, An Introduction to Scientific and Investigative Techniques 4th Edition By Suzanne Bell (Test Bank All Chapters, 100% original verified, A+ Grade)

Forensic Science, An Introduction to Scientific and Investigative Techniques, 4e Suzanne Bell (Test Bank All Chapters, 100% original verified, A+ Grade) Forensic Science, An Introduction to Scienti...

By eBookSmTb , Uploaded: Dec 21, 2023

$17

 Chemistry> TEST BANK > Test Bank - Chemistry - An Atoms First Approach 3rd Edition by Steven S. Zumdahl, Susan A. Zumdahl & Donald J. DeCoste - Complete Elaborated and Latest Test Bank with ALL Chapters[1-21] Included and Updated. (All)

preview
Test Bank - Chemistry - An Atoms First Approach 3rd Edition by Steven S. Zumdahl, Susan A. Zumdahl & Donald J. DeCoste - Complete Elaborated and Latest Test Bank with ALL Chapters[1-21] Included and Updated.

Test Bank - Chemistry - An Atoms First Approach 3rd Edition by Steven S. Zumdahl, Susan A. Zumdahl & Donald J. DeCoste - Complete Elaborated and Latest Test Bank with ALL Chapters[1-21] Included and U...

By ProfXams , Uploaded: Apr 23, 2024

$48

 Chemistry> TEST BANK > Test Bank - Chemistry - An Atoms First Approach 3rd Edition by Steven S. Zumdahl, Susan A. Zumdahl & Donald J. DeCoste - Complete Elaborated and Latest Test Bank with ALL Chapters[1-21] Included and Updated. (All)

preview
Test Bank - Chemistry - An Atoms First Approach 3rd Edition by Steven S. Zumdahl, Susan A. Zumdahl & Donald J. DeCoste - Complete Elaborated and Latest Test Bank with ALL Chapters[1-21] Included and Updated.

Test Bank - Chemistry - An Atoms First Approach 3rd Edition by Steven S. Zumdahl, Susan A. Zumdahl & Donald J. DeCoste - Complete Elaborated and Latest Test Bank with ALL Chapters[1-21] Included and U...

By Prof.Exams , Uploaded: Apr 23, 2024

$48

 Business Analytics> TEST BANK > Test Bank - Business Analytics-Applied Modelling & Prediction 1st Edition by James Abdey - Complete Elaborated and Latest Test Bank with ALL Chapters[1-20] Included and Updated. (All)

preview
Test Bank - Business Analytics-Applied Modelling & Prediction 1st Edition by James Abdey - Complete Elaborated and Latest Test Bank with ALL Chapters[1-20] Included and Updated.

Test Bank - Business Analytics-Applied Modelling & Prediction 1st Edition by James Abdey - Complete Elaborated and Latest Test Bank with ALL Chapters[1-20] Included and Updated. Business, Analytics...

By ProfXams , Uploaded: Apr 23, 2024

$23

$15.00

Add to cart

Instant download

Can't find what you want? Try our AI powered Search

OR

GET ASSIGNMENT HELP
91
0

Document information


Connected school, study & course



About the document


Uploaded On

Aug 11, 2022

Number of pages

365

Written in

Seller


seller-icon
CoursesExams

Member since 3 years

316 Documents Sold


Additional information

This document has been written for:

Uploaded

Aug 11, 2022

Downloads

 0

Views

 91

Document Keyword Tags

THE BEST STUDY GUIDES

Avoid resits and achieve higher grades with the best study guides, textbook notes, and class notes written by your fellow students

custom preview

Avoid examination resits

Your fellow students know the appropriate material to use to deliver high quality content. With this great service and assistance from fellow students, you can become well prepared and avoid having to resits exams.

custom preview

Get the best grades

Your fellow student knows the best materials to research on and use. This guarantee you the best grades in your examination. Your fellow students use high quality materials, textbooks and notes to ensure high quality

custom preview

Earn from your notes

Get paid by selling your notes and study materials to other students. Earn alot of cash and help other students in study by providing them with appropriate and high quality study materials.


$15.00

WHAT STUDENTS SAY ABOUT US


What is Browsegrades

In Browsegrades, a student can earn by offering help to other student. Students can help other students with materials by upploading their notes and earn money.

We are here to help

We're available through e-mail, Twitter, Facebook, and live chat.
 FAQ
 Questions? Leave a message!

Follow us on
 Twitter

Copyright © Browsegrades · High quality services·